Você está na página 1de 103

UNIVERSIDADE DO ESTADO DO PAR CENTRO DE CINCIAS SOCIAIS E EDUCAO DEPARTAMENTO DE MATEMTICA, ESTATSTICA E INFORMTICA.

LICENCIATURA EM MATEMTICA MODALIDADE A DISTNCIA

DISCIPLINA: TEORIA DOS NMEROS

CONTUDO PROGRAMTICO: INTRODUO AO ESTUDO DA TEORIA DOS NMEROS: UNIDADE I NMEROS INTEIROS: 1.1 - Nmeros inteiros. 1.2 - Propiedades dos inteiros; 1.3 - Valor absoluto de um inteiro; 1.4 - Questes Resolvidas e Propostas. UNIDADE II INDUO MATAMTICA: 2.1 - Elemento mnimo de um conjunto de inteiros. 2.2 - Princpio da boa ordenao; 2.3 - Princpio de induo finita; 2.4 - Induo matemtica; 2.5 - Exemplos de demonstrao por induo matemtica e outras formas de induo matemtica; 2.6 - Questes Resolvidas e Propostas. UNIDADE III DIVISIBILIDADE: 3.1 - Relao de divisibilidade em Z. 3.2 - Conjunto dos divisores de um inteiro; 3.3 - Divisores comuns de dois inteiros; 3.4 - Algoritmo da diviso; 3.5 - Paridade de um inteiro; 3.6 - Questes Resolvidas e Propostas. UNIDADE IV MXIMO DIVISOR COMUM: 4.1 - Mximo divisor comum de dois inteiros. 4.2 - Existncia e unicidade do mdc; 4.3 - Inteiros primo entre si; 4.4 - Caracterizao do mdc de dois inteiros; 4.5 - Mdc de vrios inteiros; 4.6 - Questes Resolvidas e Propostas. UNIDADE V ALGORITMO DE EUCLIDES MNIMO MLTIPLO COMUM: 5.1 - Algoritmo de EUCLIDES. 5.2 - Mltiplos comuns de dois inteiros; 5.3 - Mnimo mltiplo comum de dois inteiros; 5.4 - Relao entre o mdc e o mmc; 5.5 - Mmc de vrios inteiros; 5.6 - Questes Resolvidas e Propostas. UNIDADE VI NMEROS PRIMOS: 6.1 - Nmeros primos e compostos. 6.2 - Teorema fundamental da Aritmtica; 6.3 - Formula que do primos; 6.4 - Crivo de ERATSTENES; 6.5 - Primos gmeos; 6.6 - Seqncias de inteiros consecutivos compostos; 6.7 - Conjectura de GOLDBACH; 6.8 - Mtodo de fatorao de FERMAT; 6.9 - Questes Resolvidas e Propostas.

UNIDADE VII EQUAES DIOFANTINAS LINEARES: 7.1 - Generalidades. 7.2 - Condio de existncia de soluo; 7.3 - Soluo da equao ax + by = c; 7.4 - Questes Resolvidas e Propostas. UNIDADE VIII CONGRUNCIAS: 8.1 - Inteiros congruentes. 8.2 - Caracterizao de Inteiros congruentes; 8.3 - Propriedades das congruncias; 8.4 - Sistemas completos de restos; 8.5 - Questes Resolvidas e Propostas. UNIDADE IX CONGRUNCIAS LINEARES: 9.1 - Generalidades. 9.2 - Condio de existncia de soluo; 9.3 - Soluo da congruncia a x b(mod. m); 9.4 - Resoluo de equaes diofantinas lineares por congruncias; 9.5 - Inverso de um inteiro; 9.6 - Questes Resolvidas e Propostas. UNIDADE X SISTEMAS DE CONGRUNCIAS LINEARES: 10.1 - Generalidades. 10.2 - Teorema do resto chinez; 10.3 - Questes Resolvidas e Propostas. UNIDADE XI TEOREMA DE FERMAT E WILSON: 11.1 - Teorema de Fermat. 11.2 - Teorema de Wilson; 11.3 - Questes Resolvidas e Propostas. BIBLIOGRAFIA:

INTRODUO AO ESTUDO DA TEORIA DOS NMEROS Embora existam diversos tipos de nmeros em Matemtica (reais, complexos, etc.), o nome "Teoria dos Nmeros" tradicionalmente reservado para o estudo dos Nmeros Inteiros, isto , -3, 2, -1, 0, 1, 2, 3, ... Tambm usado o nome Aritmtico, proveniente de arithms, que em grego significa nmero". A Teoria dos Nmeros, a mais pura disciplina dentro da mais pura das Cincias a Matemtica e tem uma longa histria, originando-se nas antigas civilizaes da humanidade. Listamos primeiro alguns nomes famosos de matemticos que contribuiro para o estudo da teoria dos nmeros: Pitgoras (569-500 a. C.) Euclides (_ 350 a. C.) Eratstenes (276-196 a. C.) Diofante (_ 250 d. C.) Plutarco (_ 100 d. C.) Marin Mersenne (1588-1648) Pierre de Fermat (1601-1665) Blaise Pascal (1623-1662) Christian Goldbach (1690-1764) Leonhard Euler (1707-1783) Joseph Louis Lagrange (1736-1813) John Wilson (1741-1793) Adrien Marie Legendre (1752-1833) Carl Friedrich Gauss (1777-1855) Augustin Louis Cauchy (1789-1857) Peter Gustav Dirichlet (1805-1859) P. L. Tchebychef (1821-1894) Frederick Nelson Cole (1861-1927) Axel Thue (1863-1922) Jacques Salomon Hadamard (1865-1963) Charles de la Vall e Poussin (1866-1962) Dentre outros.... A teoria dos nmeros veio a ocupar-se com uma classe mais vasta de problemas que surgiram naturalmente do estudo dos nmeros inteiros. A teoria dos nmeros pode ser subdividida

em vrios campos, de acordo com os mtodos que so usados e das questes que so investigadas, a saber: 1) Teoria elementar dos nmeros: utiliza somente os mtodos elementares da aritmtica para a verificao e comprovao das propriedades essenciais do conjunto dos nmeros inteiros e em particular as propriedades dos nmeros primos. 2) Teoria analtica dos nmeros: utiliza a anlise real e anlise complexa, especialmente para estudar as propriedades dos nmeros primos. 3) Teoria algbrica dos nmeros: utiliza lgebra abstrata e estuda os nmeros algbricos. 4) Teoria geomtrica dos nmeros: utiliza mtodos geomtricos, algbricos e analticos. Nesta notas faremos o estudo da primeira Teoria, um conceito chave em Teoria elementar dos Nmeros o conceito de divisibilidade. Enquanto nos nmeros reais, por exemplo, pode-se dividir qualquer nmero por outro (no nulo), obtendo como resultado um nmero real, nos inteiros diferente. Um inteiro a s divisvel pelo inteiro b quando existir um inteiro c tal que a = bc. Nesse caso, diz-se tambm que b um divisor de a, ou que b divide a, ou ainda que a mltiplo de b. Por exemplo, 8 divisvel por 2, mas no por 3. Mesmo que a no seja divisvel por b, pode-se sempre encontrar, de modo nico, inteiros c (quociente) e r (resto) tais que a = bc + r. Todo inteiro a divisvel por 1, -1, a, -a. Estes so os divisores triviais de a. Um inteiro dito primo quando s possui os divisores triviais. Um inteiro de valor absoluto maior que 1 e que no seja primo (isto , possua divisores no triviais) dito composto. Por exemplo: So primos: 2, 2, 3, -3, 17, .... So compostos 6 = 2x3, -8 = (-2) x 4, ... Os nmeros 0, 1 e 1 no so primos nem compostos. Euclides foi o primeiro a demonstrar que existe uma infinidade de nmeros primos. O mximo divisor comum dos inteiros no nulos a e b tem a propriedade de ser mltiplo de qualquer divisor comum de a e b e pode ser encontrado pelo algoritmo de Euclides. Quando o mximo divisor comum de a e b for 1, ento seus nicos divisores comuns so 1 e 1. Nesse caso, a e b so ditos primos entre si, ou relativamente primos. Por exemplo, 9 e 14 so primos entre si. As propriedades mais cruciais dos nmeros inteiros, e que no tm similares nos reais ou nos complexos, so o Princpio da Boa Ordenao, segundo o qual qualquer conjunto no vazio de inteiros limitado inferiormente possui um elemento mnimo, e o Princpio de Induo, segundo o qual se uma propriedade P(n), referente ao inteiro n, for verdadeira para n = a, e a veracidade de P(n) acarretar a veracidade de P(n + 1), ento P(n) verdadeira para todo inteiro maior que ou igual a a. A partir das propriedades usuais da adio e da multiplicao de inteiros, da relao <, e do Princpio da Boa Ordenao (ou do de Induo, que lhe equivalente), possvel construir toda a Teoria dos Nmeros. Um de seus resultados mais importantes o Teorema Fundamental da

Aritmtica, segundo o qual todo inteiro (diferente de 0, 1 e 1) pode ser escrito de modo nico como um produto de fatores primos. Uma das caractersticas da Teoria dos Nmeros que ela inclui problemas extremamente simples de enunciar e ao mesmo tempo incrivelmente difceis de resolver. Um exemplo a conjectura de Feuerbach: "todo nmero par a soma de dois nmeros primos"; ningum at hoje conseguiu decidir se isto verdadeiro ou falso. Outro exemplo o famoso ltimo Teorema de Fermat: "Dado um inteiro n maior que 2, impossvel encontrar inteiros no nulos x, y, z tais que x n + yn = zn". Este teorema, enunciado no sculo XVII por Fermat, que s foi demonstrado em 1995, por Wiles. Gauss, o "prncipe dos matemticos", dizia que a Matemtica era a rainha das cincias, e a Aritmtica, a rainha das Matemticas. Gauss desenvolveu muita a Teoria dos Nmeros. Aos 22 anos, em 1799, publicou em latim suas "Investigaes Aritmticas", onde introduziu o importante conceito de congruncia para nmeros inteiros. O matemtico ingls Hardy, grande especialista em Teoria dos Nmeros, orgulhava-se, em 1940, de que "nenhuma descoberta sua havia feito, nem provavelmente viria a fazer, direta ou indiretamente, alguma diferena para o conforto da humanidade". No entanto, 50 anos depois, um obscuro matemtico americano descobriria uma falha no recm lanado processador Pentium, ao realizar clculos "inteis" sobre primos gmeos (nmeros primos que diferem de 2). Mas j no prprio momento em que Hardy escrevia aquela frase, durante a segunda guerra mundial, trs americanos desenvolviam um sistema de cdigo secreto, chamado SRA, baseado nas dificuldades insuperveis para descobrir os fatores primos de um nmero muito grande. Criava-se um novo ramo a Criptografia, a cincia dos cdigos, fortemente baseado em Teoria dos Nmeros. Com o advento dos computadores e da computao algbrica, a Criptografia ganhou um novo impulso. Neste momento, a proliferao de senhas bancrias e de cartes de crdito, bem como a crescente necessidade de criptografar dados confidenciais que inundam a Internet, faz da Criptografia um dos ramos mais em moda da Matemtica aplicada. E um dos mais teis, para desespero pstumo de Hardy.

UNIDADE I NMEROS INTEIROS 1 - Introduo: A noo de nmero est, atravs dos tempos, associada a todos os tipos de atividades humanas. A primeira concepo de nmero data do perodo paleoltico. Poucos progressos se fizeram neste campo at se dar a transio para o perodo neoltico, durante o qual j existia uma atividade comercial importante entre diversas povoaes. Esta atividade promoveu a formao de linguagens, cujas palavras exprimiam coisas muito concretas e poucas abstraes, mas onde j havia lugar para alguns termos numricos simples. Estes termos numricos destinavam-se apenas a estabelecer a distino entre um, dois e muitos. Depois de durante milnios ter utilizado os nmeros para contar, medir, calcular, o homem comeou a especular sobre a natureza e propriedades dos prprios nmeros. Desta curiosidade nasceu a Teoria dos Nmeros, um dos ramos mais profundos da matemtica. A Teoria dos Nmeros nasceu cerca de 600 anos antes de Cristo quando Pitgoras e os seus discpulos comearam a estudar as propriedades dos nmeros inteiros. Os pitagricos rendiam verdadeiro culto mstico ao conceito de nmero, considerando-o como essncia das coisas. Acreditavam que tudo no universo estava relacionado com nmeros inteiros ou razes de nmeros inteiros (em linguagem atual, nmeros racionais). Alis, na antiguidade a designao nmero aplicava-se s aos inteiros maiores do que um. Esta crena foi profundamente abalada quando usaram o Teorema de Pitgoras para calcular a medida da diagonal de um quadrado unitrio. Com efeito, a diagonal divide o quadrado em dois tringulos retngulos issceles cujos catetos tm comprimento um e assim, pelo teorema de Pitgoras, a medida da hipotenusa igual raiz quadrada de dois, que no pode ser expresso como quociente de inteiros. Ao descobrirem que a diagonal de um quadrado de lado 1 no era uma razo entre dois inteiros (em linguagem atual, que a raiz quadrada de 2 um nmero irracional) os Pitagricos consideraram quebrada a harmonia do universo, j que no podiam aceitar a raiz quadrada de dois como um nmero, mas no podiam negar que esta raiz era a medida da diagonal de um quadrado unitrio. Convencidos de que os deuses os castigariam caso divulgassem aquilo que lhes parecia uma imperfeio divina, tentaram ocultar a sua descoberta. Segundo reza a lenda, o primeiro membro da seita pitagrica que divulgou esta descoberta morreu afogado. Este fato teve grandes repercusses na histria da cincia que se fizeram sentir at finais do sculo XIX. De cada vez que as necessidades do clculo levavam a introduzir novos entes numricos gerava-se uma enorme desconfiana sua volta, o que levava a atribuir-lhes designaes curiosas. Assim, os nmeros irracionais eram designados por nmeros inexprimveis e por nmeros incalculveis. Durante muitos sculos os nmeros reais (fracionarias ou racionais e irracionais)

foram apenas concebidos como medidas de grandezas e s nos finais do sculo XIX, principalmente por obra dos matemticos alemes Dedekind e Cantor, se construiu uma teoria dos nmeros reais independente da geometria. 1.1 - Nmeros Inteiros Noes Fundamentais: Os nmeros inteiros ou apenas os inteiros so: , 3, 2, 2, 0,1, 2,3, . Cujo conjunto representa-se pela letra , isto : destacam-se os seguintes subconjuntos: 1) Conjunto dos inteiro no nulos ( 0) . = x
z/x 0
z/x 0

, 3, 2, 2, 0,1, 2,3, , deste conjunto

1, 2, 3, 4, .
0,1, 2,3, .

2) Conjunto dos inteiro no negativos ( 0) . = x 3) Conjunto dos inteiro no positivos ( 0) . = x 4) Conjunto dos inteiro positivos ( 0) . = x 4) Conjunto dos inteiro positivos ( 0) . = x
z/x z/x

z/x 0 0

0, 1, 2, 3, . 1, 2,3, 4, . 1, 2, 3, 4, .

Os inteiros positivos so tambm denominados inteiros naturais e por isso o conjunto dos inteiros positivos habitualmente designados pela letra = * . + 1.2 - Propriedades dos Inteiros: O conjunto Z dos inteiros munidos das operaes de adio (+) e multiplicao ( ) possui as propriedades fundamentais que a seguir enumeramos, onde a, b e c so inteiros quaisquer, isto , elementos de z: 1) Lei comutativa para multiplicao e adio: a + b = b + a e ab = ba . 2) Lei associativa para multiplicao e adio: a + b + c = a + b + c
e (ab)c = a(bc) .

3) Existncia da identidade para adio e multiplicao: 0 + a = a e a 1 = 1 a = a . 4) Existncia do inverso em relao adio, -a, para todo inteiro a: a + (- a) = (-a) + a = 0 . 5) Lei distributiva: a b + c = ab + ac . 6) Lei do cancelamento da multiplicao 0 a = 0 e se ab = 0, ento a = 0 ou b = 0 . Tambm existe uma relao de ordem entre os inteiros, representada pelo sinal < (menor que) que possui as seguintes propriedades: 7) Se a 8) Se a 9) Se a 10) Se a
0, ento a < 0 ou 0 < a . b, e b < c ento, a < c . b, ento a + c < b + c . b, ento 0 < c, ento ac < bc .

11) Se a

b, ento c < 0, ento bc < ac .

12) (Lei da Tricotomia) Para quaisquer inteiros a e b, vale exatamente uma das seguintes afirmaes: a < b, a = b ou a > b . 13) Suponha que a
b , e seja c um inteiro qualquer. Ento

a + c b+c ac bc se c > 0, mas ac bc, se c < 0.

Destas propriedades podem ser deduzidas muitas outras propriedades dos inteiros. 1.3 - Valor absoluto de um Inteiro: Definio: Chama-se valor absoluto de um inteiro a, o inteiro que se indica por a , tal que:

a se a -a se a

o o
a a
2

0 a2 a . a a2

A partir da definio de a , para todo inteiro a, temos:

-a a a

Teoremas: Se a e b so dois inteiros, quaisquer ento: 1) a 2)


a 0e a a a . 0 se a = 0 .

3) ab 4) a

a b.

b.

5) a + b

ab .

Questes Resolvidas 01) Calcular a soma dos n primeiros inteiros positivos. Soluo: Vamos escrever a soma dos n primeiros nmeros inteiros positivos em ordem crescente e a mesma soma em ordem decrescente, temos: S=1 S=n + 2 + n1 + 3 + n2 + 4 + ........ + n 3 + ........ + n3 + 4 +n2 + n1 + 3 + 2 + n + 1

Somando as duas igualdades: 2S = (n + 1) + (n + 1) + (n + 1) + (n + 1) + ..............

+ (n + 1) + (n + 1) + (n + 1) + (n + 1)

Observe que sero n parcelas iguais a (n + 1). Portanto, 2S = n(n + 1) S = n(n + 1)/2. 02) Calcular o inteiro positivo n, sabendo que 3n+2 2n+3 = 2592. Soluo: Decompondo 2592, obtm-se 34.25. Portanto, n + 2 = 4 n = 2, ou 5 = n + 3 n = 2. Pois a forma de decomposio em fatores primos nica. 03) Achar um inteiro positivo de dois algarismos que seja igual ao qudruplo da soma dos seus algarismos. Soluo: Um nmero de algarismos a e b, na base 10 expresso por 10a + b. Portanto, 10a + b = 4(a + b) 6a = 3b b = 2a. Ou seja, qualquer nmero de dois algarismos, onde o algarismo das unidades o dobro do algarismo das unidades. Assim, temos: 12, 24, 36, etc. 04) Achar o menor e o maior inteiro positivo de n algarismos. Soluo: Menor: 1 algarismo igual a 1 e os demais (n 1) iguais a zero. Portanto, 1 x 10 n 1. Maior: todos os n algarismos iguais a 9, ou 1 seguido de n zeros menos 1 1.10n 1 Observao: Considerando, n = 5. Menor 10000 = 1.105 1 = 1.104 Maior 99999 = 100000 1 = 1.105 1. 05) Achar todas as solues inteiras e positivas da equao: x2 y2 = 88. Soluo: x2 y2 = 88 (x + y) (x y) = 88. Como x e y so inteiros positivos, (x + y) e (x y) so dois nmeros inteiros cujo produto 88. Assim, (1) x + y = 88 e x y = 1; (2) x + y = 44 e x y = 2; (3) x + y = 22 e x y = 4; (4) x + y = 11 e x y = 8. Cada par de duas equaes formam um sistema. Para resolver o sistema basta somar as duas equaes, o que resultaria em 2x = soma dos nmeros. Como essa soma tem que ser par (x inteiro), resulta apenas as possibilidades 2 e 3. Portanto, 2x = 46 x = 23 e y = 44 23 = 21 ou 2x = 26 x = 13 e y = 22 13 = 9. Resposta: (x = 23, y = 21) e (x = 13, y = 9). 06) O produto de um inteiro positivo de trs algarismos por 7 termina direita por 638. Achar esse inteiro.

Soluo: Para facilitar o raciocnio, construamos a tabela de multiplicao por 7. 7 x 1 = 7, 7 x 2 = 14, 7 x 3 = 21, 7 x 4 = 28, 7 x 5 = 35, 7 x 6 = 42, 7 x 7 = 49, 7 x 8 = 56, 7 x 9 = 63. Como o algarismo das unidades 8, o nico valor possvel para o algarismos das unidades do nmero 4. Ao efetuar a multiplicao do algarismo das unidades, que 4 por 8, vo duas unidades para a casa das dezenas. Assim, o algarismo das dezenas deve ser tal que ao multiplicar por 7 e somar 2, resulte num final igual a 3. Portanto, o algarismo das dezenas 3, pois 7 x 3 + 2 = 23. Da mesma forma vo duas unidades para a casa das centenas. O algarismo a deve ser de forma que, ao somar 2 (vindo das dezenas) resulte em 6. Portanto, deve ser um mltiplo de 7 terminado em 4. Isto permite concluir que o algarismo das centenas 2 , pois 2 x 7 + 2 = 16. Portanto, o nmero 234. 07) Um livro tem 1235 pginas. Determinar o nmero de vezes que o algarismo 1 aparece na numerao da pginas deste livro. Soluo: De 1 a 100, o algarismo 1 aparece 10 vezes nas unidades (1, 11, 21,... 91) e 10 vezes nas dezenas (10, 11, 12, ...19). Portanto a cada centena o algarismo 1 aparece 20 vezes. Em 1235 temos 12 centenas. Portanto o algarismo 1 aparecer 20 x 12 = 240 vezes na posio das unidades e dezenas. De 100 a 200, o algarismo 1 aparece 100 vezes na posio das centenas. Isto se repete de 1100 a 1200. Portanto, 200 vezes na posio das centenas. De 1200 a 1236, o algarismo 1 aparece 4 vezes nas unidades e 10 vezes nas dezenas. Totalizando 14 vezes. De 1000 a 1235, o algarismo 1 aparece 236 vezes na posio dos milhares. Portanto: 240 + 200 + 14 + 236 = 690 vezes. 08) Achar o inteiro que deve ser somado a cada um dos inteiros 2, 6 e 14 para que, nesta ordem, formem uma proporo contnua. Soluo: Uma proporo contnua aquela que tem os meios ou os extremos iguais. Pela definio podemos ter (a) (2 + x)/(6 + x) = (6 + x)/(14 + x) ou (2 + x)/(6 + x) = (14 + x)/(2 + x). Na situao (a), (6 + x) 6 + x) = (2 + x) (14 + x) => 36 + 12x + 2x = 28 + 16x + 2x 4x = 8 x = 2. Na situao (b) (2 + x) (2 + x) = (6 + x) (14 + x) 4 + 4x + 2x = 84 + 20x + 2x 16x = - 80 x = - 5. R: 2 ou 5. 09) Achar o menor inteiro cujo produto por 21 um inteiro formado apenas por 4 algarismo. Soluo: O nmero o menor mltiplo de 21 maior que 1000. Portanto: 1000 = 47 x 21 + 13. Portanto, o nmero 48 x 21 = 1008. 10) Escreve-se a seqncia natural dos inteiros positivos, sem separar os algarismos:

123456789101112131415... Determinar: (a) O 435 algarismo. Soluo: De 1 a 9 so escritos 9 algarismos. De 10 a 99, so dois algarismos em cada nmero 2 x 90 = 180 algarismos. Portanto, at 100 so escritos: 9 + 180 + 3 = 192. Para chegar ao algarismo que ocupa o 435 lugar sero necessrios mais 435 192 = 243 algarismos. Como a partir de 100 so usados 3 algarismos teramos 243 3 = 81 nmeros aps o 100. Portanto, o nmero 181 e o algarismo que ocupa a posio o 1. (b) O 1756 algarismo. Soluo: Da mesma forma 1756 192 = 1564 1564 3 = 521 e sobra 1 algarismo. Portanto teramos at a 100 + 521 = 621. Como sobra 1 algarismo, o prximo o 6 do nmero 622. (c) O 12387 algarismo. Soluo: At 1000 seriam 9 + 90 x 2 + 900 x 3 + 4 = 2889. 12387 2889 = 9498 9498 4 = 2374 e sobram dois algarismo. Portanto, o ltimo nmero inteiro 1000 + 2374 = 3374. A sobra de dois algarismos implica que o ltimo algarismo ser 3, o segundo algarismo de 3375. 11) Mostrar que o produto de dois fatores entre 10 e 20 o dcuplo da soma do primeiro com as unidades do segundo mais o produto das unidades dos dois. Soluo: Sejam os nmeros 10 + b e 10 + c, com 0 < b < 10 e 0 < c < 10. Nestas condies 10 + b e 10 + c estaro compreendidos entre 10 e 20 e b e c sero os algarismos das unidades. Efetuando o produto temos: (10 + b) (10 + b) = 100 + 10b + 10c + bc = 10[(10 + b) + c] + bc. (10 + b) + c a soma do primeiro com as unidades do segundo, bc o produto dos dois e 10[(10 + b) + c] o dcuplo da soma do primeiro com as unidades do segundo.

Questes Propostas 01) Calcule o inteiro positivo n, sabendo-se que: 3n + 3n+1 + 3n+2 + 3n+3 = 1080. R: n = 3. 02) Decompor o inteiro 575 numa soma de cinco inteiros mpares consecutivos. R: 109, 113, 115, 117, 119. 03) Achar todas as solues inteiras e positivas da equao (x + 1) (y + 2) = 2xy. R: os valores so (x = 2, y = 6), (x = 3, y =4) e (x = 5, y = 3). 04) Verificar se o quadrado de um inteiro pode terminar em 2, 3, 7 ou 8. R: Portanto, no pode terminar em 2, 3, 7 ou 8. 05) A soma dos quadrados de dois inteiros 3332 e um deles o qudruplo do outro. Achar os dois inteiros. R: 14 e 56. 06) A mdia aritmtica de dois inteiros positivos 5 e a mdia geomtrica 4. Achar os dois inteiros. R: 8 e 2. 07) Calcular a soma dos trs maiores nmeros inteiros de, respectivamente, trs, quatro e cinco algarismos. R: 110997. 08) Determinar a diferena entre o maior nmero inteiro com seis algarismos diferentes e o menor inteiro com cinco algarismos tambm diferentes. R: 977420. 09) Mostrar que o produto de quatro algarismos consecutivos, aumentado de 1, um quadrado perfeito. 10) Sejam a e b dois inteiros. Demonstrar: (a) Max (a, b) = (a + b + |a b|)/2. (b) Min (a, b) = (a + b |a b|)/2. 11) Achar cinco inteiros positivos consecutivos cuja soma dos quadrados igual a 2010. R: 18, 19, 20, 21 e 22. 12) Escreve-se a seqncia natural dos inteiros positivos pares, sem separar os algarismos: 24681012141618...

Determinar o 2574 algarismo que se escreve. R: algarismo o 6. 13) Os inteiros a e b so tais que 1 < a < 3 e 2 < b < 0. Mostrar que 1 < a b < 5. R: 1 < a b < 5. 14) Os inteiros a e b so tais que -2 < a < 2 e - 2 < b < 2. Mostrar que 4 < a b < 4. R: 4 < a b < 4. 15) Achar o menor inteiro positivo que multiplicado por 33 d um produto cujos algarismos so todos 7. R: 777777: 33 = 23569. 16) No planeta Mong o nmero de horas por dia igual a nmero de dias por semana, que igual ao nmero de semanas por ms, que igual ao nmero de meses por ano. Sabendo que em Mong h 4096 horas por ano, quantas semanas h por ms? R: 17) A soma de trs nmeros naturais consecutivos igual ao produto desses trs nmeros. A soma dos quadrados desses nmeros : R: 18) Sejam a, b e c inteiros e positivos. Entre as opes abaixo, a expresso que no pode representar o nmero 24 : a) ab3 b) a2b3 c) acbc d) ab2c3 e) acbccc

19) Efetuando as operaes indicadas na expresso abaixo obtemos um nmero de quatro


22007 algarismos. Qual a soma dos algarismos desse nmero: 2006 2 22005 22004 2006 .

R: 20) Qual a soma dos algarismo do nmero: R:


22 2 23 22 24 22005 2004 23 2 22006 ? 22005

21) Quantos so os possveis valores inteiros de x para que R: 20.

x 99 seja um nmero inteiro? x 19

22) Um pai tem 33 anos e seu filho, 7 anos. Depois de quantos anos a idade do pai ser o triplo da idade do filho? R: 6.

UNIDADE II INDUO MATEMTICA 2.1 - Introduo: O Princpio da Induo um eficiente instrumento para a demonstrao de fatos referentes aos nmeros naturais. Por isso deve-se adquirir prtica em sua utilizao. Por outro lado, importante tambm conhecer seu significado e sua posio dentro do arcabouo da Matemtica. Entender o Princpio da Induo praticamente o mesmo que entender os nmeros naturais. Apresentamos abaixo uma breve exposio sobre os nmeros naturais, onde o Princpio da Induo se insere adequadamente e mostra sua fora terica antes de ser utilizado na lista de exerccios propostos ao final. 2.2 - A Seqncia dos Nmeros Naturais: Os nmeros naturais constituem um modelo matemtico, uma escala padro, que nos permite a operao de contagem. A seqncia desses nmeros uma livre e antiga criao do esprito humano. Comparar conjuntos de objetos com essa escala abstrata ideal o processo que torna mais precisa a noo de quantidade; esse processo (a contagem) pressupe, portanto o conhecimento da seqncia numrica. Sabemos que os nmeros naturais so 1, 2, 3, 4, 5, A totalidade desses nmeros constitui um conjunto, que indicaremos com o smbolo N e que chamaremos de conjunto dos naturais. Portanto N = {1, 2, 3, 4, 5,}. Evidentemente, o que acabamos de dizer s faz sentido quando j se sabe o que um nmero natural. Faamos de conta que esse conceito nos desconhecido e procuremos investigar o que h de essencial na seqncia 1, 2, 3, 4, 5 . Deve-se a Giussepe Peano (1858 - 1932) a constatao de que se pode elaborar toda a teoria dos nmeros naturais a partir de quatro fatos bsicos, conhecidos atualmente como os axiomas de Peano. Noutras palavras, o conjunto N dos nmeros naturais possui quatro propriedades fundamentais, das quais resultam, como conseqncias lgicas, todas as afirmaes verdadeiras que se podem fazer sobre esses nmeros. 2.3 - Elemento Mnimo: Definio 1.1 - Seja A um conjunto de inteiros. Chama-se elemento mnimo de A um elemento a A tal que a x para todo x A. Notao: a = min A. x).

min A = a se, e somente se, (a A e ( x A) a

Teorema: Se a elemento mnimo de A, ento este elemento nico. 2.4 - Princpio da Boa Ordenao (PBO). Todo conjunto no vazio A, de inteiros no negativos, possui elemento mnimo. A Z+, A , existe min A.

2.5 - Induo Matemtica: Teorema: Seja P(n) uma proposio associada a cada inteiro positivo n e que satisfaz s duas seguintes condies: 1) P(1) verdadeira. 2) Para todo inteiro k, se P(k) verdadeira, ento P(k + 1) tambm verdadeira. Nestas condies, a proposio P(n) verdadeira para todo inteiro positivo n. 2.6 - Princpio da Induo Finita (PIF). Teorema: Seja S um subconjunto do conjunto N dos inteiros positivos (S seguintes propriedades: 1) 1 pertence a S (1 S). 2) Para todo inteiro positivo k, se k S, ento (k + 1) S; N) que satisfaz as duas

3) Nestas condies, S o conjunto N dos inteiros positivos: S = N. 2.7 - Outra Forma da Induo Matemtica: Teorema: Seja r um nmero inteiro positivo fixo e seja P(n) uma proposio associada a cada inteiro n r e que satisfaa s duas seguintes condies:

1) P(r) verdadeira. 2) Para todo inteiro k r, se P(k) verdadeiro, ento P(k + 1) verdadeiro; r.

3) Nestas condies, P(n) verdadeira para todo inteiro n Questes Resolvidas

01) Demonstrar por "induo matemtica", as questes abaixo: n(n 1)(2n 1) 1) 12 + 22 + 32 + ... + n2 = n N. 6 1.2.3 Soluo: P(1) verdadeira, pois 12 = . Suponhamos que para P(k) verdadeira: 6 k(k 1)(2k 1) 12 + 22 + 32 + ... + k2 = . Somando (k + 1)2 a ambos os membros: 6 k(k 1)(2k 1) 12 + 22 + 32 + ... + k2 + (1 + k)2 = + (k + 1)2 6 k (k 1)(2k 1) 6(k 1) 2 = 6 (k 1) k(2k 1) 6(k 1) = 6 2 (k 1)(2k k 6k 6) = 6 (k 1)(k 2)(2k 3) = Logo P(k + 1) verdadeira. 6

2) 13 + 23 + 33 + ... + n3 =

n 2 (n 1) 2 4

N.

Soluo: P(1) verdadeira, pois 13 = 13 + 23 + 33 + ... + k3 =

12 (1 1) 2 . Suponhamos que para P(k) verdadeira: 4

k 2 (k 1) 2 Somando (k + 1)3 a ambos os membros 4 k 2 (k 1) 2 3 3 3 3 3 1 + 2 + 3 + ... + k + (k + 1) = + (k + 1)3 4 2 k (k 1) 2 4(k 1) 3 = 4 2 2 (k 1) k 4(k 1) = 4 2 2 (k 1) (k 4k 4) = 4 2 (k 1) (k 2) 2 = Logo P(k + 1) verdadeira. 4 n (4n 2 1) 3) 1 + 3 + 5 + ... + (2n 1) = n N. 3 1( 4.12 1) Soluo: P(1) verdadeira, pois 12 = Suponhamos que para P(k) verdadeira: 3 k (4k 2 1) 12 + 32 + 52 + ... + (2k 1)2 = Somando (2k + 1)2 a ambos os membros 3 k (4k 2 1) 12 + 32 + 52 + ... + (2k 1)2 + (2k + 1)2 = + (2k + 1)2 3 k (2k 1)(2k 1) 3(2k 1) 2 = 3 (2k 1) k (2k 1) 3(2k 1) = 3 2 (2k 1)(2k 5k 3) = 3 (2k 1)(k 1)(2k 3) = 3 (k 1)(2k 1)(2k 3) = Logo P(k + 1) verdadeira. 3
2 2 2 2

4) 13 + 33 + 53 + ... + (2n 1)3 = n2(2n2 1) Soluo: P(1) verdadeira, pois 13 = 12(2.12 1). Suponhamos que para P(k) verdadeira: 13 + 33 + 53 + ... + (2k 1)3 = k2(2k2 1). Somando (2k + 1)3 a ambos os membros 13 + 33 + 53 + ... + (2k 1)3 + (2k + 1)3 = k2(2k2 1) + (2k + 1)3 = 2k4 k2 + 8k3 + 12k2 + 6k + 1 = 2k4 + 8k3 + 11k2 + 6k + 1 = (k + 1)(2k3 + 6k2 + 5k + 1) = (k + 1)(k + 1)(2k2 + 4k + 1) = (k + 1)2(2(k + 1)2 1) Logo P(k + 1) verdadeira.

n(n 1)(n 2) . 3 Soluo: P(1) verdadeira, pois 1(1 + 1)= 1(1 + 1)(1 + 2)/3
5) 1.2 + 2.3 + 3.4 + ... + n(n + 1) = Suponhamos que para P(k) verdadeira: 1.2 + 2.3 + 3.4 + ... + k(k + 1) = Somando (k + 1)(k + 2) a ambos os membros: 1.2 + 2.3 + 3.4 + ... + k(k + 1) + (k + 1)(k + 2) =

k(k 1)(k 2) 3

k(k 1)(k 2) + (k + 1)(k + 2) 3 k (k 1)(k 2) 3(k 1)(k 2) = 3 (k 1)(k 2)(k 3) = Logo P(k + 1) verdadeira. 3

a (q n 1 1) 6) a + aq + aq + ... + aq = ,q q 1
2 n

1.

Soluo: P(1) verdadeira, pois a + aq = a + aq + aq2 + ... + aqk =

a (q 1 1 1) Suponhamos que para P(k) verdadeira. q 1

a (q k 1 1) Somando aqk+1 a ambos os membros q 1 a (q k 1 1) a + aq + aq2 + ... + aqk + aqk+1 = + aqk+1 q 1 a (q k 1 1) aq k 1 (q 1) = q 1 k 1 aq 1 q k 1 (q 1) = q 1 k 1 a (q 1 qk 2 qk 1) = q 1 k 2 a (q 1) = Logo P(k + 1) verdadeira. q 1
02) Demonstrar por induo matemtica: 1) 2n < 2n+1 n N. Soluo: P(1) verdadeira, pois 21 < 21+1. Suponhamos que para P(k) verdadeira: 2k < 2k+1 Multiplicando por 2 ambos os membros 2.2k < 2.2k+1 ou 2k+1 < 2k+2 logo, P(k + 1) verdadeira. 2) n ! > n2 n 4. Soluo: P(4) verdadeira, pois 4! > 42 ou 24 > 16. Suponhamos que para P(k) verdadeira: k! > k2 Multiplicando por k + 1 ambos os membros k!(k + 1) > k2(k + 1) ou (k + 1)! > k3 + k2 como k3 > k2 e k2 > 2k + 1 temos (k + 1)! > k2 + 2k + 1 ou (k + 1)! > (k + 1)2 Logo P(k + 1) verdadeira. 3) 2n > n2 n 5. Soluo: P(5) verdadeira, pois 25 > 52. Suponhamos que para P(k) verdadeira: 2k > k2 Multiplicando por 2 ambos os membros 2.2k > 2.k2 ou 2k+1 > k2 + k2 > k2 + 2k + 1 (pois k2 > 2k + 1) ou 2k+1 > (k + 1)2. Logo P(k + 1) verdadeira.

4) 24 | (52n 1) n N. Soluo: P(1) verdadeira, pois 24 | (52.1 1) Suponhamos que para P(k) verdadeira, 24 | (52k 1). Logo 52k 1 = 24t ou 52k = 24t + 1 Multiplicando ambos os membros por 52.52.52k = 52 (24t + 1) = 52.24t + 25 = 52.24t + 24 + 1 52k+2 = 24(25t + 1) + 1 ou 52(k+1) 1 = 24q onde q = 24t + 1 ou 24 | (52(k+1)1). 5) 5 | (8n 3n) n N. Soluo: P(1) verdadeira, pois 5 | (81 31) Suponhamos que para P(k) verdadeira 5 | (8k 3k) Logo 8k 3k = 5t, onde t um nmero inteiro. Vamos mostrar que P(k + 1) verdadeira: 8k+1 3k+1 = 8.8k 3.3k = 8.8k (8.3k 5.3k) = 8.8k 8.3k + 5.3k = 8(8k 3k) + 5.3k = 8.5t + 5.3k = 5(8t + 3k) fazendo 8t + 3k = q = 5q Logo 5 | (8k+1 3k+1). 6) 4n > n4 n 5. Soluo: P(5) verdadeira, pois 45 > 54 Suponhamos que para P(k) verdadeira, k > 5 4k > k4 Multiplicando por 4 ambos os membros 4.4k > 4k4 ou 4k+1 > k4 + 3k4 Vamos usar a desigualdade n4 > 4n3 + 6n2 + 4n + 1 4k+1 > k4 + 3k4 > k4 + 4k3 + 6k2 + 4k + 1 ou 4k+1 > (k + 1)4 Logo P(k + 1) verdadeira 7) Demonstrar que 10n + 1 9n 10 um mltiplo de 81 para todo inteiro positivo n. Soluo: P(1) verdadeira, pois 81 | (101+1 9 1 10) ou 81 | 81. Suponhamos que para P(k) verdadeira: 81 | (10k+1 9k 10) Logo 10k+1 9k 10 = 81t Multiplicando ambos os membros por 10. 10.10k+1 10.9k 10.10 = 10.81t 10k+2 9k 81k = 10.81t + 102 10k+2 9k 9 = 10.81t + 102 + 81k 9 10k+2 9(k + 1) 10 = 10.81t + 102 + 81k 9 10 10k+2 9(k + 1) 10 = 10.81t + 81k + 81 10k+2 9(k + 1) 10 = 81(10t + k + 1) 10k+2 9(k + 1) 10 = 81q fazendo 10t + k + 1 = q Logo 81 | [10k+2 9(k + 1) 10 }].
n 3 n 5 7n 8) Demonstrar que um inteiro positivo para todo n N. 3 5 15 n 3 n 5 7n 5n 3 3n 5 7 n , Soluo: = isto : 15 | (5n3 + 3n5 + 7n) n N. 3 5 15 15 3 P(1) verdadeira, pois 15 | (5 1 + 3 15 + 7 1) ou 15 | 15. Suponhamos que para P(k) verdadeira: 15 | (5k3 + 3k5 + 7k) ou 5k3 + 3k5 + 7k = 15t. Vamos mostrar que. P(k + 1) verdadeira. Somando a expresso 15k4 + 30k3 + 45k2 + 30k + 15 a ambos os membros da igualdade acima, temos: 5k3 + 3k5 + 7k + 15k4 + 30k3 + 45k2 + 30k + 15 = 15t + 15k4 + 30k3 + 45k2 + 30k + 15. Arranjando os termos convenientemente, temos: 5k3 + 15k2 + 15k + 5 + 3k5 + 15k4 + 30k3 + 30k2 + 15k + 3 + 7k + 7 = 15(t + k4 + 2k3 + 3k2 + 2k + 1). 5(k3 + 3k2 + 3k + 1) + 3(k5 + 5k4 + 10k3 + 10k2 + 5k + 1) + 7(k + 1) = 15(t + k4 + 2k3 +3k2 + 2k + 1). 5(k + 1)3 + 3(k + 1)5 + 7(k + 1) = 15q fazendo t + k4 + 2k3 +3k2 + 2k + 1 = q Assim. 15 | [5(k + 1)3 + 3(k + 1)5 + 7(k + 1)] o que queramos provar.

Questes Propostas 01) Demonstrar por induo matemtica, as questes abaixo: 1 1 1 1) 1 + 3 + 5 + ... + (2n 1) = n2 n . 10) 1 ... 4 9 n2 2)

1 , n n

1 21

1 1 n 2 2 2

1 2n

n .
.

11)

2 31

2 2 n 2 3 3

1 , n . 3n

3) 1 r r 2 r n

1 rn + 1 , n 1 r

12) 1 2 3 n

n(n 1) , n . 2
1

4) 2 5 8 (2 3n) 5) 12 22 32 n 2 6) 6 | n(n +1)(n + 2) 7) 2 | (n2 + n) 8)


1 1 2

(n 1)(4 + 3n) , n 2

. 13) 20 21 22 2n

2n 1, n
n(n 1) 2
2

.
.

n(n 1)(2n + 1) , n . 14) 13 23 33 n 3 6


n . 15) 13 23 33 n 3 16) (1 a)n 1 na,
n n 1 , n 17) 3 | (22n 1)
.18) 3 | (n3 + 2n)

n .

n4 , n . 4 n , a , a

1.

1 1 1 2 3 3 4 n (n+1)
1 2 1 1 1 1 3 n

n . n .

9) (1 1) 1

n 1, n

02) Usando o principio da "induo matemtica", demonstrar: 1) O nmero de diagonais de um polgono convexo de n lados : d n

n(n 3) . 2

2) A soma das medidas dos ngulos internos de um polgono convexo de n lados : Sn (n 2) 1800 . 3) Se A um conjunto finito com n elementos, ento elementos.
(A) , conjunto das partes de A, tem 2n

4) Prove que a soma de uma PG ou razo q de n termos e primeiro termo a1 dado por a1 (q n 1) . Sn q 1 5) Prove que uma P.G. de primeiro termo a1 e razo q o produto (Pn) dos n primeiros termos dado por Pn
n a1 q n(n - 1) 2

n 1.

6) Prove que numa PA. de primeiro termo a1 e razo r a soma (Sn) dos n primeiros termos dado n(n 1) por Sn na1 r. 2 7) Para r + , e n 0, mostre que r (cos + isen )
n

r n (cos n + isen n) , onde i2 = -1.

8) Sendo z um nmero complexo no-nulo e n 0 , mostre que ( z n ) ( z ) n .

9) Prove que numa o termo geral da P. A. dado pela formula a n 10) Prove que (a + b)n

a1 (n 1)r .

Ca Ca
sen cos

0 n n

1 n-1 n n

C b , para n
1.

n n n

11) Seja A

cos sen

. Determine An, para n

12) Para x e n

1, mostre que sen nx


B) '

n sen x .
A' B' , sobre n conjuntos.

13) Demonstrar a lei de De Morgan (A

UNIDADE III DIVISIBILIDADE 3.1 - Introduo: Sabemos, desde a escola bsica, que quando um nmero inteiro e dividido por um segundo nmero inteiro no nulo, o quociente pode ou no ser um numero inteiro. Esta observao nos leva a seguinte definio. 3.2 - Divisibilidade: Definio 3.2 - Sejam a e b dois inteiros, com a um inteiro q tal que b = aq. Se a divide b tambm se diz que a divisor de b, que b mltiplo de a, que a um fator de b ou que b divisvel por a. Notao: a | b (a 0 divide b e portanto, a notao a | b significa que a 0 no divide b). 0. Diz-se que a divide b se, e somente se, existe

A relao a divide b (a | b) denomina-se relao de divisibilidade em . Observao: Se a | b, ento a | b. Teorema 3.1: Quaisquer que sejam os inteiros a, b e c tem-se: (1) a | 0, a 0, 1 | a e a | a, a 0. (2) Se a | 1, ento a = 1.

(3) Se a | b e se c | d, ento ac | bd. (4) Se a | b e se b | c, ento a | c. (5) Se a | b e se b | a, ento a = (6) Se a | b com b 0, ento | a | b. | b |.

(7) Se a | b e se a | c, ento a |(bx + cy) para todos x e y em . 3.3 - Conjunto de divisores de um inteiro: D(a) = {x *| x | a}. 3.4 - Divisores comuns de dois inteiros: Definio 3.3: Chama-se divisor comum de dois inteiros a e b todo inteiro d Notao: D(a, b) = {x 0 tal que d | a e d | b.

* | x | a e x | b}
D(b).

Propriedade; D(a, b) = D(a) Obs.: D(a, b)

; D(0, 0) = *.

3.5 - Algoritmo da Diviso: Teorema 3.2: Se a e b so dois inteiros, com b > 0, ento existem e so nicos os inteiros q e r que satisfazem s condies: a = bq + r e 0 r < b. 0, existem e so nicos os inteiros q e r que

Corolrio 3.2: Se a e b so dois inteiros com b satisfazem s condies: a = bq + r, 0 r < | b |.

3.6 - Paridade de um Inteiro: Na diviso de um inteiro qualquer a por b = 2 os possveis restos so r = 1 e r = 0. Se r = 0 ento, o inteiro a = 2q denominado par; e se r = 1, ento o inteiro a = 2q + 1 denominado mpar. Questes Resolvidas 01) Mostrar que, se a um inteiro qualquer, ento um dos inteiros a, a + 2, a + 4 divisvel por 3. Soluo: Quando dividimos um inteiro a qualquer por 3 , os restos so 0, 1 ou 2. Assim a = 3q ou a = 3q + 1 ou a = 3q + 2. Se a = 3q, ento 3 | a Se a = 3q + 1, a + 2 = 3q + 1 + 2 ou a + 2 = 3(q + 1) ento 3 | (a + 2). Se a = 3q + 2, a + 4 = 3q + 2 + 4 ou a + 4 = 3(q + 2) ento 3 | (a + 4). 02) Sendo a um inteiro qualquer, mostrar: a) 2 | a(a + 1). Soluo: Como a e a + 1 so inteiros consecutivos, ento um dos dois par, sendo o outro mpar; ento a(a + 1) = 2k1(2k2 + 1) onde 2k1 representa o nmero par e 2k2 + 1 representa o nmero mpar. Assim 2 | a(a + 1). 03) 3 | a(a + 1)(a + 2). Soluo: Dividindo a por 3, temos trs hipteses. a = 3q ou a = 3q + 1 ou a = 3q + 2 Se a = 3q ento 3 | a e por conseguinte 3 | a(a + 1)(a + 2) Se a = 3q + 1 ento a + 2 = 3q + 1 + 2 ou a + 2 = 3(q + 1); logo 3 | (a + 2) e por conseguinte 3 | a(a +1)(a + 2) Se a = 3q + 2 ento a + 1 = 3q + 2 + 1 ou a + 1 = 3(q + 1); logo 3 | (a + 1) e por conseguinte 3 | a(a + 1)(a + 2) 04) Mostrar que todo inteiro mpar da forma 4k + 1 ou 4k + 3. Soluo: Quando dividimos um inteiro por 4 os restos possveis so: 0, 1, 2 ou 3. Se o nmero for mpar, os restos s podero ser 1 ou 3 e, assim, n = 4k + 1 ou n = 4k + 3 onde n o nmero mpar. 05) Mostrar que o quadrado de um inteiro qualquer da forma 3k ou 3k + 1. Soluo: Dividindo um inteiro por trs, obtemos os restos 0, 1 ou 2. Sendo assim podemos escrever: a = 3k1, a = 3k1 + 1 ou a = 3k1 + 2, onde a um inteiro qualquer Se a = 3k1, a2 = 9 k12 ou a2 = 3(3k12). Fazendo 3k12 = k, temos a = 3k Se a= 3k1 + 1, a2 = 9k12 + 6k1 + 1 ou a2 = 3(3k12 +2k1) + 1. Fazendo 3k12 +2k1 = k temos a2 = 3k + 1 Se a = 3k1 + 2, a2 = 9k12 + 12k1 + 4 ou a2 = 9k12 + 12k1 + 3 + 1; assim a2 = 3(3k12 +4k1 + 1) + 1. Fazendo 3k12 +4k1 + 1 = k temos a2 = 3k + 1.

06) Mostrar que

n( n 1)(2 n 1) um inteiro qualquer que seja o inteiro positivo n. 6

Soluo: Devemos provar que n(n + 1)(2n + 1) mltiplo de 6. Como n e n + 1 so inteiros consecutivos, ento um dos dois par, logo mltiplo de 2. Vamos mostrar que um dos nmeros n, n + 1, 2n + 1 mltiplo de 3. Se n no for mltiplo de 3, ento n = 3q + 1 ou n = 3q + 2. Se n = 3q + 1, 2n + 1 = 6q + 2 + 1 ou 2n + 1 = 3(2q + 1). Logo 2n + 1 mltiplo de 3. Se n = 3q + 2, n + 1 = 3q + 2 + 1 ou n + 1 = 3(q + 1). Logo n + 1 mltiplo de 3. Ento podemos concluir que um dos trs nmeros n, n + 1, 2n + 1 mltiplo de 3. Podemos, ento, escrever: n(n + 1)(2n + 1) = 2t13t2t3, substituindo o fator mltiplo de 2 por 2t1 e o fator mltiplo de 3 por 3t2, sendo t3 outro inteiro qualquer. Conclumos que n(n + 1)(2n + 1) = 6t1t2t3 mltiplo de 6, como queramos demonstrar. Demonstrar: 07) Se a um inteiro mpar, ento 24 | a(a2 1). Soluo: a(a2 1) = a(a 1)(a + 1). Como a mpar, a = 2t + 1, a 1 = 2t e a + 1 = 2t + 2. Ento a(a2 1) = (2t + 1)2t(2t + 2) ou a(a2 1) = (2t + 1) 2t2(t + 1) ou a(a2 1) = 4t(t + 1)(2t + 1).t(t + 1)(2t + 1) mltiplo de 6. Portanto a(a2 1) = 4.6 k ou a(a2 1) = 24 k; conclumos que 24 | a(a2 1). 08) Se a e b so inteiros mpares, ento 8 | (a2 b2). Soluo: a2 b2 = (a - b)(a + b). Como a e b so mpares a = 2q + 1 e b = 2k + 1. Onde q e k so inteiros quaisquer. Ento: a2 b2 = 2q + 1 (2k + 1) (2q + 1 + 2k + 1) a2 b2 = (2q 2k)(2q + 2k + 2) ou a2 b2 = 2(q k)2(q + k + 1) ou a2 b2 = 4(q k)(q + k + 1). Quaisquer que sejam as paridades de q e k, q k e q + k tem a mesma paridade: deste modo q - k e q + k + 1 tm paridades diferentes, isto , um par e o outro mpar. Assim (q k)(q + k + 1) = 2t(2t + 1). Substituindo na expresso acima, obtemos: a2 b2 = 4.2t(2t + 1) ou a2 b2 = 8t(2t + 1) e logo 8 | (a2 b2 ). 09) Mostrar que a diferena entre os cubos de dois inteiros consecutivos nunca divisvel por 2. Soluo: Sejam os inteiros consecutivos n e n + 1. Achemos a diferena entre os cubos destes nmeros: (n + 1)3 n3 = n3 + 3n2 + 3n + 1 n3 = 3n2 + 3n + 1 = 3n(n + 1) + 1. Como n e n + 1 so consecutivos, um dos dois par e, portanto, o produto 3n(n + 1) tambm par e 3n(n + 1) + 1 mpar. Logo no divisvel por 2.

10) Na diviso do inteiro a = 427 por um inteiro positivo b o quociente 12 e o resto r. Achar o divisor b e o resto r. Soluo:: De acordo com o algoritmo da diviso 427 = 12b + r, 0 b o divisor , 12 o quociente e r o resto. Desta igualdade tiramos: 427 12b = r e da 0 12b 427 12b < b. Somando 12b aos trs membros da desigualdade, obtemos: 427, tiramos b 35, 5 ou b 35.
r b ; onde 427 o dividendo,

427 < 13b. Da desigualdade 12b

Da desigualdade 427 < 13b tiramos b > 32,8 ou b intervalo 33,35 ou 33, 34 e 35.

33. Assim os valores para b so os inteiros no

Para b = 33, r = 427 12 x 33 = 31. Para b = 34, r = 427 12 x 34 = 19. Para b = 35, r = 427 12 x 35 = 7. 11) Achar um inteiro de quatro algarismos, quadrado perfeito, divisvel por 27 e terminado em 6. Soluo: Se a, b, c ... so fatores primos, os expoentes desses fatores devem ser par. Como 27 = 33, deve-se ter pelo menos mais um 3 como fator. Portanto, o nmero deve ser mltiplo de 27 x 3 ou de 81. Para que o nmero termine em 6, devemos multiplicar 81 por um quadrado (pois 81 j quadrado), terminado em 6, pois 81 termina em 1. Assim, temos as possibilidades 81 x 16 = 1296 e 81 x 36 = 2916. Se o nmero tivesse 6 fatores iguais a 3, ele deveria ser mltiplo de 729. Para que terminasse em 6, deveriamos ter 729 x a, com a terminado em 4. Como os menores quadrados terminados em quatro so 4 e 64, teramo: 729 x 4 = 2916 e 729 x 64 = 46656 que tem 5 algarismos. Para 8 fatores iguais a 3, o nmero deveria ser mltiplo de 6561 = 38. Para que o nmero terminasse em 6, deveriamos ter 6561 x a, com a terminado em 4. Como os menores quadrados terminados em quatro so 4 e 64, teramos 6561 x 4 = 26244 que contm cinco algarismos. Para 10 fatores iguais a 3, teramos 310 > 10000, que ter mais de 4 algarismos. Portanto, os nicos nmeros so 1296 e 2916. 12) Demonstrar que se m e n so inteiros mpares, ento 8 | (m4 + n4 2). Soluo: se m e n so mpares, podemos escrever: m = 2k + 1 e n = 2k + 1. Temos ento: m4 + n4 - 2 = (2k + 1)4 + (2k + 1)4 2 = [(2k)4 + 4(2k)3 + 6(2k)2 + 4(2k) + 1] + [(2k)4 + 4(2k')3 + 6(2k)2 + 4(2k)+1] 2 = 16(k4 + k4) + 32(k3 k3) + 24(k2 + k2) + 8(k + k) + 2 2 = 8[2(k4 + k4) + 4(k3 k3) + 3(k2 + k2) + (k + k)]. Como 2(k4 + k4) + 4(k3 k3) + 3(k2 + k2) + (k + k)]. um inteiro (multiplicao e adio de inteiros), podemos escrever: m4 + n4 - 2 = 8q, q inteiro 8 | (m4 + n4 2).

Questes Propostas 01) Mostrar que, se a | b, ento (-a) | b e a | (-b) e (-a) | (-b). 02) Sejam a, b e c inteiros. Mostrar: a) se a | b, ento a | bc. b) Se a | b e se a | c, ento a2 | bc. c) a | b se, e somente se, ac | bc, (c 0).

03) Mostrar que um inteiro qualquer da forma 6k + 5 tambm da forma 3k + 2. 04) Mostrar que o cubo de um inteiro qualquer de uma das formas: 9k, 9k + 1 e 9k + 8. 05) Mostrar que, se a | (2x 3y) e se a | (4x 5y), ento a | y. 06) Determinar os inteiros positivos que divididos por 17 deixam um resto igual ao quadrado do quociente. R: 18, 38, 60 e 84. 07) Na diviso do inteiro 525 por um inteiro positivo o resto 27. Achar os inteiros que podem ser o divisor e o quociente. R: b = 498 e q = 1; b = 249 e q = 2; b = 166 e q = 3 e b = 83 e q = 6. 08) Na diviso de dois inteiros positivos o quociente 16 e o resto o maior possvel. Achar os dois inteiros, sabendo que a sua soma 341. R: a = 322, b = 19. 09) Achar os inteiros positivos menores que 150 e que divididos por 39 deixam um resto igual ao quociente. R: q = 1, 2, 3 e a = 40, 80, 120. 10) Seja d um divisor de n (d | n). Mostrar que cd | n se, e somente se, c |

n . d

11) Mostrar que para todo inteiro n, existem inteiros k e r tais que n = 3k + r e r = 1, 0, 1. 12) Mostrar que todo inteiro mpar, quadrado perfeito, da forma 4n + 1. 13) Na diviso de 392 por 45, determinar: a) o maior inteiro que se pode somar ao dividendo sem alterar o quociente. R: 12. b) o maior inteiro que se pode subtrair do dividendo sem alterar o quociente. R: 32. 14) Numa diviso de dois inteiros, o quociente 16 e o resto 167. Determinar o maior inteiro que se pode somar ao dividendo e ao divisor sem alterar o quociente. R: 11. 15) Achar o maior inteiro de quatro algarismos divisvel por 13 e o menor inteiro de cinco algarismos divisvel por 15. R: maior 9997 e o menor 10.005.

UNIDADE IV MXIMO DIVISOR COMUM - M. D. C 4.1 - Introduo: Fazem parte do ensino fundamental, entre outras, as noes de Mximo Divisor Comum (MDC), Mnimo Mltiplo Comum (MMC), Nmeros primos e Fatorao, que compem uma parcela significativa da Teoria Elementar dos Nmeros. No caso especfico M.D.C, pretendemos mostrar a relevncia destes atravs da abordagem de exerccios para o aprofundamento terico e aps o estudo do M.M.C veremos aplicaes de forma contextualizada. 4.2 - Mximo Divisor Comum: Definio 4.1 - Dados dois ou mais nmeros inteiros no nulos denominamos mximo divisor comum destes nmeros ao maior nmero inteiro que divisor simultaneamente de todos os nmeros dados. O mdc o maior elemento da interseco dos conjuntos dos divisores positivos dos nmeros dados. Definio 4.2 - Sejam a e b dois inteiros no conjuntamente nulos (a 0 ou b 0). Chama-se

mximo divisor comum de a e b o inteiro positivo d (d > 0) que satisfaz s condies: 1) d | a e d | b. 2) Se c | a e c | b, ento c Notao: d = mdc(a, b). 4.3 - Existncia e Unicidade do MDC: Teorema 4.1 - Se a e b so inteiros no conjuntamente nulos (a 0 ou b 0), ento existe e nico d.

o mdc(a,b); alm disso, existem inteiros x e y tais que mdc(a, b) = ax + by, isto , o mdc(a, b) uma combinao linear de a e b. Teorema 4.2 - Se a e b so dois inteiros no conjuntamente nulos (a de todos os mltiplos do mdc(a, b) = d T = {ax + by | x,y 4.4 - Inteiros Primos entre si: Definio 4.3 - Sejam a e b dois inteiros no conjuntamente nulos (a so primos entre si se, e somente se, o mdc(a, b) = 1. Teorema 4.3 - Dois inteiros a e b, no conjuntamente nulos (a e somente se, existe inteiros x e y tais que ax + by = 1. Corolrio 4.1 - Se o mdc(a, b) = d, ento mdc
a b , d d

0 ou b

0), ento o conjunto

Z}.

0 ou b

0). Diz-se que a e b

0 ou b

0), so primos entre si se,

= 1.

Corolrio 4.2 - Se a | b e se mdc(b, c) = 1, ento o mdc(a, c) = 1. Corolrio 4.3 - Se a | c, se b | c e se mdc(a, b) = 1, ento ab | c. Corolrio 4.4 - Se mdc(a, b) = 1 = mdc(a, c), ento o mdc(a, bc) = 1.

Corolrio 4.5 - Se mdc(a, bc) = 1, ento mdc(a, b) = 1 = mdc(a, c). Teorema 4.4 - (de Euclides) Se a | bc e se o mdc(a, b) = 1 ento a | c. 4.5 - Caracterizao do MDC de dois Inteiros: Teorema 4.5 - Sejam a e b dois inteiros no conjuntamente nulos (a positivo d (d > 0) o mdc(a, b) se, e somente se, satisfaz s condies: 1) d | a e d | b. 2) Se c | a e c | b, ento c | d. 4.6 - Caracterizao do MDC de dois Inteiros: O conceito de mximo divisor comum, definido para dois inteiros a e b, estende-se de maneira natural a mais de dois inteiros. No caso de trs inteiros a, b e c, no todos nulos, o mdc (a, b, c) o inteiro positivo d (d > 0) que, satisfaz s condies: 1) d | a, d | b e d | c. 2) Se e | a, se e | b e se e | b, ento e 4.7 - MDC de Vrios Inteiros: Teorema 4.6 - O mdc(a,b,c) = mdc(mdc(a,b), c). Questes Resolvidas 01) Determinar: (a) mdc(11, 99). Soluo: 99: 11 = 9, resto zero mdc(11,99) = 11 R: 11. (b) mdc(-21, 14). Soluo: mdc(-21, 14) = mdc(21, 14) 21: 14 = 1 resto 7. 14:7 = 2, resto zero mdc(-21, 14) = 7 R: 7. (c) mdc(17, 18). Soluo: 18: 17 = 1, resto 1 17: 1 = 17 resto 0 mdc(17, 18) = 1. R: 1. 02) Achar o menor inteiro positivo c da forma c = 22x + 55y onde x e y so dois inteiros. Soluo: Como o menor inteiro da forma 22x + 55y o mdc(22, 55) ento c = 11. 03) Calcular mdc(n, n + 2), sendo n um inteiro par. d. 0 ou b 0). Um inteiro

Soluo: Seja d = mdc(n, n+2). Ento d | n e d | (n+2). Como d | n e d | (n+2), ento, d | 2 e portanto, d = 1 ou d = 2 e como n par a resposta ser d = 2 (o maior dos divisores). 04) Calcular mdc(n, n + 2), sendo n um inteiro mpar. Soluo: Seja d = mdc(n, n+2). Ento, usando o mesmo raciocnio anterior, d = 1 ou d = 2. Mas como n mpar, conclumos que d = 1. 05) Sendo n um inteiro qualquer, calcular o mdc(n, n + 1) Soluo: Seja d = mdc(n, n+1). Ento d | n e d | (n+1). Como d | n e d | (n+1), ento d | 1. Logo d = 1. Sejam a, b e c inteiros. Demonstrar: 06) Existem inteiros x e y tais que c = ax + by se, e somente se, mdc(a, b) | c. Soluo: Demonstraremos primeiramente, a ida: Seja d = mdc(a, b). Ento d | a e d | b e logo d | (ax + by) quaisquer que sejam os inteiros x e y. Portanto d | c, desde que c = ax + by por hiptese. Soluo: Demonstraremos agora a volta: Seja d = mdc(a, b). Como d | c, temos c = dq. Sendo d o mdc(a, b) existem inteiros x e y tais que d = ax + by. Substituindo este valor na igualdade c = dq obtemos: c = (ax + by )q. Da tiramos valor c = a(xq) + b(yq). Fazendo xq = x e yq = y temos c = ax + by. 07) Se existem inteiros x e y tais que ax + by = mdc(a, b), ento o mdc(x, y) = 1 Soluo: Seja d = mdc(a, b). Ento existem inteiros x e y tais que d = ax + by, onde d | a e d |b. Dividindo ambos os membros por d ( d > 0), obtemos:
a x d b y = 1 Como d | a e d | b as d

expresses entre parntesis so inteiras, que representaremos por k1 e k2 obtendo assim: xk1 + yk2 = 1, donde conclumos que mdc(x, y) = 1. Determinar inteiros positivos a e b sabendo: 08) a + b = 63 e o mdc(a, b) = 9. Soluo: Se 9 = mdc(a, b), temos que 9 | a e 9 | b ou a = 9q1 e b = 9q2. Substituindo estes valores na igualdade a + b = 63 temos: 9q1 + 9q2 = 63; da, dividindo por 9 obtemos: q1 + q2 = 7. Como q1 e q2 so primos entre si, devemos procurar inteiros positivos primos entre si que somem 7. Temos os seguintes valores: q1 = 1 e q2 = 6; q1 = 2 e q2 = 5 e q1 = 3 e q2 = 4. Assim obtemos para a e b os seguintes valores: a = 9 e b = 54; a = 18 e b = 45 e a = 27 e b = 36. 09) ab = 756 e o mdc(a, b) = 6.

Soluo: Se 6 = mdc(a, b) temos que 6 | a e 6 | b ou a = 6q1 e b = 6q2. Substituindo na igualdade a b = 756, temos 6q16q2 = 756. Da obtemos que q1q2 = 21. Do mesmo modo, como q1 e q2 so primos entre si, devemos encontrar inteiros positivos cujo produto d 21. Os valores so: q1 = 1 e q2 = 21 e q1 = 3 e q2 = 7. Destes valores tiramos os valores de a e b: a = 6 e b = 126 e a = 18 e b = 42. 10) Demonstrar que, se n = abc + 1, ento o mdc (n, a) = mdc(n, b) = mdc(n, c) = 1. Soluo: Provemos que mdc(n, a) = 1. O mesmo pode ser feito para b e c. Seja d = mdc(n, a). Ento d | n e d | a. Podemos dizer, portanto, que d | abc, mltiplo de a. Como d | n e d | abc, ento d | 1, pois n abc = 1. Logo d = 1. 11) O mdc(a, 4) = 2 = mdc(b, 4). Demonstrar que o mdc(a + b, 4) = 4. Soluo: Temos mdc(a, 4) = 2 e mdc(b, 4) = 2. Conclumos que a e b so nmeros pares e no so mltiplos de 4 , pois se o fossem, 2 no seria o mdc entre eles. Logo o resto da diviso de a e b por 4 2. Assim a = 4q1 + 2 e b = 4q2 + 2. Somando membro a membro estas desigualdades, temos a + b = 4q1 + 2 + 4q2 + 2 ou a + b = 4 (q1 + q2 + 1). Logo, 4 | (a + b) e por conseguinte mdc(a + b, 4) = 4. 12) Sendo a e b dois inteiros no conjuntamente nulos (a mdc(a, b) = mdc(-a, b) = mdc(a, -b) = mdc(-a, -b). Soluo: Se c | a ento a = qc. Temos que - a = (-q)c c | (-a) todo divisor de a divisor de (-a) maior divisor de a tambm o maior divisor de a. O mesmo ocorre com b e b. Portanto, podemos concluir que o maior divisor comum de (a e b), tambm de (a e b), de (a e b) e o de (a, -b). Assim, mdc(a, b) = mdc(-a, b) = mdc(a, -b) = mdc(-a, -b). 13) Demonstrar que mdc(mdc(a, b), b) = mdc(a, b). Soluo: A definio do mdc de trs nmeros mdc(a, b, c) = mdc(mdc(a, b), c), quaisquer que sejam a, b e c. Fazendo c = b, temos mdc(mdc(a, b), b) = mdc(a, b, b) = mdc(a, mdc(b, b)) = mdc(a, b) pois mdc(b, b) = b. 14) Demonstrar que o mdc(n + k, k) = 1 se e somente se o mdc(n, k) = 1. Soluo: Se mdc(n + k, k) = 1, ento existem os inteiros x e y, tais que (n + k)x+ ky = 1 nx + k(a + b) = 1 (n, k) = 1. Por outro lado, se mdc(n, k) = 1, ento existem a e b tais que na + kb = 1. Fazendo a = x e b = x + y, teremos nx + k(x + y) = 1 (n + k)x + ky = 1 mdc(n + k), k) = 1. 15) Calcular o mdc(a + b, a b) sabendo que a e b so inteiros primos entre si. Soluo: Se a e b so primos entre si, no podem ser ambos pares, pois o mdc seria 2 ou mltiplo de 2. Portanto, a e b so ambos mpares ou so de paridades diferentes. 0 ou b 0), mostrar:

(1 caso) a e b com paridades diferentes (a = 2k + 1 b = 2k) Temos ento: a + b = 2k + 1 + 2k = 2(k + k) + 1 = 2n + 1 a + b mpar. a b = 2k + 1 2k = 2(k k) + 1 = 2m + 1 a b mpar. Portanto, o mdc(a + b, a b) um nmero mpar. Seja ento mdc(a + b, a b) = 2k + 1 existem x e y tais que (a + b)x + (a b)y = 2k + 1 [(a + b)/(2k+1)]x + [(a b)/(2k + 1)]y = 1 (a + b)/(2k + 1) e (a b)/(k + 1) so primos entre si. Fazendo r = (a + b)/(2k + 1) e s = (a b)/(2k + 1), resulta: a + b = r(2k + 1) (i) e a b = s(2k + 1) (ii).Como (a + b), (a b) e (2k + 1) so mpares, r e s tambm so mpares. Alm disso r e s mpares, r + s e r s so pares. Somando membro a membro as igualdades (i) e (ii), resulta: (1) 2a = (2k + 1)(r + s) a = (2k + 1)[(r + s)/2] pois s + r par (inteiro), portanto 2 | (r + s). Assim, existe o inteiro (r + s)/2, tal que a = (2k + 1)[(r + s)/2) 2k + 1 | a. Subtraindo membro a membro as igualdades (i) e (ii), (2) 2b = (2k + 1)(r s) b = (2k + 1)[(r s)/2]. (r s) par. Portanto, (r s)/2 inteiro. Assim, existe o inteiro (r s)/2, tal que b = (2k + 1)[(r s)/2] 2k + 1 | b. Ora, a e b so primos entre si. Portanto, o nico divisor comum 2k + 1. Disto permite-se escrever 2k + 1 = 1 1 = mdc(a + b, a b). (2 caso) a e b so mpares a = 2k + 1 e b = 2k + 1. Temos, ento: (a + b) = 2k + 1 + 2k + 1 (a + b) = 2(k + k + 1) (a b) = 2k + 1 2k 1 = 2(k k) Das igualdades acima, conclumos que (a + b) e (a b) so pares. Portanto, o mdc da forma 2k. Assim, existem x e y, tais que: (a + b)x + (a b)y = 2k r = (a + b)/2k e s = (a b)/2k so primos entre si. (a + b) = 2kr (i) e (a b) = 2ks (ii). Somando membro a membro, 2a = 2k(r + s) a = k(r + s) k | a. Subtraindo membro a membro, 2b = 2k(r s) b = k(r s) k | b. Como a e b so primos entre si, o nico divisor comum de a e b 1. Portanto, k = 1 e Mdc(a + b, a b) = 2k mdc(a + b, a b) =2.1 = 2. Portanto, se a e b so primos ento mdc(a + b, a b) 1 ou 2. 16) O mdc de dois inteiros positivos 10 e o maior deles 120. Determinar o outro inteiro. Soluo: Seja a o outro nmero. a deve ser um mltiplo de 10 que no divide 120. Como o maior dos nmeros e 120, a deve ser menor que 120. Os mltiplos de 10 que no dividem 120 so: 50, 70, 90 e 110.

17) O mdc(a, b) = p, sendo p um primo. Achar os possveis valores do: ( a ) mdc(a2, b). Soluo: Sejam a = p.a1.a2.a3 ...an, onde p, a1, a2, a3, ... an so os fatores primos de a e b = p.b1.b2.b3...bn, onde p, b1, b2, b3, ...bn so os fatores primos de b. Assim, a2 = p.p.a1.a2.a2.a3a3...an.an que a2 e b so divisveis ao mesmo tempo apenas por p. mdc(a2, b) = p. ( b ) mdc(a3, b) = p, mesma concluso acima. ( c ) mdc(a2, b3) = p2. Pois aparecem 2 fatores iguais a p em a2 e 3 fatores iguais a p em b3. 18) Sejam a e k inteiros no conjuntamente nulos. Demonstrar que mdc(a, a + k) | k. Soluo: Seja m o mdc(a, a + k). Assim, existem os inteiros x e y tais que: a = mx e a + k = my. Subtraindo primeira igualdade da segunda resulta: (a + k) a = my mx k = m(y x). Como x e y so inteiros, y - x inteiro. Portanto, existe o inteiro (y x) tal que k = m(y x) m | k ou mdc(a, a + k) | k. 19) Seja o quadrado abaixo em que cada lado mede 3 cm. Quantos quadradinhos de 1cm cabem no quadrado?

R: 9 quadradinhos. Com o mesmo quadrado acima, obter o valor de 3. R: 3 = 9. 20) De quantos cubinhos de 1cm de lado, isto , um centmetro cbico, precisaremos para construir um cubo com 3cm de comprimento, 3cm de largura e 3cm de altura?

R: 27 cubinhos.

Questes Propostas 01)Achar os elementos do conjunto A = {1, 2, 3, 4, 5} que so primos com 8. R: 1, 3 e 5. 02) Sabendo que o mdc(a, 0) = 13, achar todos os valores do inteiro a. R: 13 03) Sendo n um inteiro qualquer, calcular o mdc(n, n + 1). R: 1. 04) Sendo n um inteiro qualquer, achar os possveis valores do mximo divisor comum dos inteiros n e n + 10. R: 1, 2, 5, 10. 05) Sendo n um inteiro qualquer, calcular o mdc(n 1, n2 + n + 1). R: 1 ou 3. 06) Sejam a, b e c inteiros. Demonstrar: ( a ) se o mdc(a, b) = 1 ento o mdc(ac, b) = mdc(b, c) Portanto, todo divisor de d divisor de ac. ( b ) Se o mdc(a, b) = 1 e se c | (a + b), ento o mdc(a, c) = 1 e o mdc(b, c) = 1. ( c ) se b | c, ento o mdc(a, b) = mdc(a + c, b). ( d ) Se mdc(a, b) = 1, ento mdc(am, bn) = 1, onde m e n so inteiros positivos. 07) Achar o maior inteiro positivo pelo qual se devem dividir os inteiros 160, 198 e 370 para que os restos sejam respectivamente 7, 11 e 13. R: 17. 08) Os restos das divises dos inteiros 4933 e 4435 por um inteiro positivo n so respectivamente 37 e 19. Achar o inteiro n. R: n = 96 ou n = 48. 09) Demonstrar que, se a | bc e se mdc(a, b) = d, ento a | cd. 10) Demonstrar que, se a | c, se b | c e se o mdc(a, b) = d ento ab | cd. 11) Demonstrar que se mdc(a, b) = 1 e se mdc(a,c) = d,ento mdc(a, bc) = d. 12) O inteiro mpar d um divisor de a + b e de a b. Demontrar que d tambm um divisor do mdc(a, b). 13) Os inteiros positivos m e n so tais que o mdc(m, n) = d. Mostrar que o mdc(2m 1, 2n 1) = 2d 1. 14) Demonstrar que mdc(a, b) = mdc(a, b, a + b). 15) Demonstrar que mdc(a, b) = mdc(a, b, ax + by), quaisquer que seja os inteiros x e y. 16) Demonstrar que se o mdc(a, b) = d ento o mdc(a2, b2) = d2. 17) Demonstrar que mdc(a, b) = mdc(a, c) implica mdc(a2, b2) = mdc(a2, c2). 18) Demonstrar que mdc(a, b) = mdc(a, c) implica mdc(a, b) = mdc(a, b, c).

19) Demonstrar que mdc(a, b, c) = mdc(mdc(a, b), mdc(a, c). 20) Sejam a e b inteiros positivos tais que ab um quadrado perfeito e o mdc(a, b) = 1. Demonstrar que a e b so quadrados perfeitos. 21) Demonstrar que mdc( a + b, a b) > mdc(a, b). 22) Sejam a, b, c, d (b no um inteiro. 23) Determinar os inteiros positivos a e b, sabendo que a2 b2 = 7344 e mdc(a, b) = 12. R: a = 312 e b = 300, ou a = 120 e b = 84. 24) Dividindo-se dois inteiros positivos pelo seu mdc, a soma dos quocientes 8. Determinar os dois inteiros, sabendo-se que sua soma 384. R: 48 e 336 ou 144 e 240. 25) Trs rolos de arame farpado tm, respectivamente, 168 m, 264m e 312 m. Deseja-se cort-los em partes de comprimentos iguais, de maneira que cada parte seja a maior possvel. Qual o comprimento e o nmero de partes? R: 24 m e 31 partes. 26) Um terreno retangular de 221 m por 117 m ser cercado. Em toda a volta deste cercado sero plantadas rvores igualmente espaadas. Qual o maior espao possvel entre as rvores? R: 13 m. 27) Em uma excurso ao parque do Caraa, em Minas Gerais, viajaram dois nibus um com 42 pessoas e outro com 30. Os guias queriam organizar grupos com o mesmo nmero de pessoas, mas sem misturar as pessoas que vieram nos dias nibus. Eles queriam tambm que esse nmero de pessoas por grupo fosse o maior possvel. Quantos grupos e de pessoas, foram formadas em cada nibus? R: Foram formados 12 grupos de 6 pessoas sendo 7 grupos no 1 nibus e 5 grupos no 2 nibus. 28) Uma tecelagem fabrica peas de tecidos em trs metragens diferentes: 45m, 60m e 105m. Desejando cortar as peas em partes de mesmo comprimento e que esteja e que este seja o maior possvel, qual dever ser o comprimento de cada parte? Em quantas partes cada pea ser cortada? R: Cada parte dever ter 15m de comprimento. A pea 45m ser cortada em 3 partes, a pea de 60m ser cortada em 4 partes e a pea de 105m em 7 partes. 29) De um aeroporto, partem todos os dias, 3 avies que fazem rotas internacionais. O primeiro avio faz a rota de ida e volta em 4 dias, o segundo em 5 dias e o terceiro em 10 dias. Se num certo dia os trs avies partem simultaneamente, depois de quantos dias esses avies partiro novamente no mesmo dia? R: 20 dias. d) inteiros tais que mdc(a, b) = mdc(c, d) = 1. Mostrar que a soma a/b + c/d

UNIDADE V ALGORITMO DE EUCLIDES: MNIMO MLTIMPLO COMUM M.M.C 5.1 - Introduo: Fazem parte do ensino fundamental, entre outras, as noes de Mximo Divisor Comum (MDC), Mnimo Mltiplo Comum (MMC), Nmeros primos e Fatorao, que compem uma parcela significativa da Teoria Elementar dos Nmeros. No caso especfico M.M.C, pretende-se mostrar a relevncia destes atravs da abordagem de temas atuais onde aparecem e sua conexo com outras reas do conhecimento. Com isso, visamos a contextualizao e a interdisciplinaridade, ambas importantes para que o aluno veja a matemtica como uma aliada na vida prtica e sua relao com outras disciplinas. Neste sentido, busca-se que o aluno perceba que os nmeros e a lgebra formam um sistema de cdigos ligados especialmente a diversas aplicaes. Definio 5.1 - Dados dois ou mais nmeros inteiros no nulos denominamos mnimo mltiplo comum destes nmeros dados ao menor nmero inteiro positivo que mltiplo simultaneamente te todos os nmeros dados. O mmc o menor elemento da interseco dos conjuntos dos mltiplos positivos dos nmeros dados. Lema: 5.2 - Se a = bq + r, ento mdc(a, b) = mdc(b, r): 5.3 - Algoritmo de Euclides: Sejam a e b dois inteiros no conjuntamente nulos (a cujo mximo divisor comum se deseja determinar. 1) Se a 2) Se a 0, ento mdc(a,0) = |a|. 0, ento mdc(a,a) = |a|. 0 ou b 0)

3) Se b | a, ento o mdc(a,b) = |b|. 4) Se a no divide b e b no divide a, ento a = bq + r e mdc(a, b) = mdc(b, r). Dispositivo prtico para o clculo do Mximo divisor comum de dois inteiros positivos a e b denominado Algoritmo de Euclides. a r1 q1 b r2 q2 r1 r2 q3 r2 r4

qn rn - 1 0

qn - 1 rn

Teorema 5.1 - Se k > 0, ento o mdc(ka, kb) = k mdc(a, b). Corolrio 5.1 - Para todo k 0, o mdc(ka, kb) = |k| mdc(a, b).

5.4 - Mltiplos Comuns de dois Inteiros: M(a) = {x Z | a | x} = {aq | q Z}.

M(1) = M(1) = Z. Definio 5.2 - Sejam a e b dois inteiros diferentes de zero (a comum de a e b todo inteiro x tal que a | x e b | x. M(a, b) = M(a) 0eb M(b). 0). Chama-se mltiplo

5.5 - Mnimo Mltiplo Comum: Definio 5.3 - Sejam a e b dois inteiros diferentes de zero (a 0eb 0). Chama-se mnimo

mltiplo comum de a e b o inteiro positivo m (m > 0) que satisfaz s condies: 1. a | m e b | m. 2. se a | c e b | c, com c > 0, ento m Notao: m = mmc(a, b). Observao: a) mmc(a, b) ab. b) Se a | b, ento mmc(a, b) = |b|. c.

5.6 - MMC de Vrios inteiros: O conceito de mnimo mltiplo comum, definido para dois inteiros a e b, estende-se de maneira natural a mais de dois inteiros. No caso de trs inteiros a, b e c, diferentes de zero, o m.m.c(a, b, c) o inteiro positivo m(m > 0) que satisfaz s condies: 1) a | m, b | m e c | m. 2) Se a | e, b | e, e se c | e, com e > 0, ento m 5.7 - Relao Entre o MDC e o MMC: Teorema 5.2 - Para todo par de inteiros positivos a e b subsiste a relao mdc(a, b) mmc(a, b) = ab. Corolrio 5.2 - Para todo par de inteiros positivos a e b, o mmc(a, b) = ab se, e somente se mdc(a, b) = 1. 5.7 - Regra Prtica para obteno do MDC e MMC de Vrios inteiros: Podemos determinar o mdc e o mmc de dois ou mais nmeros inteiros positivos procedendo do seguinte modo: 1) Fatoramos os nmeros, decompondo-se em fatores primos positivos; 2) Calculamos o mdc multiplicando os fatores comuns aos nmeros, tomando cada fator uma s vez e com o menor expoente que ele apresenta nas decomposies dos nmeros dados; 3) Calculamos o mmc multiplicando os fatores comuns e os no comuns aos nmeros, tomando cada fator uma s vez e com o maior expoente que ele apresenta nas decomposies dos nmeros dados; Exemplo: Dados as seguintes decomposies de inteiros a = 32.19.712, b = 2.35.19.61 e c = 24.192.71, determine: a) MDC(a, b) = 32.19 b) MMC(a, b) = 24. 35.192.61.712 c) MMC(a, c) = 24. 32.192.712 d) MDC(a, b, c) = 19 e) MDC(b, c) = 2.19 e.

Questes Resolvidas 01) Usando o algoritmo de Euclides, achar os inteiros x e y que verifiquem cada uma das seguintes igualdades: a) mdc(56, 72) = 56x + 72y mdc(56, 72) = 8 8 = 56x + 72y 72 = 56.1 + 16 56 = 16.3 + 8 16 = 8.2 + 0 b) mdc(24, 138) = 24x + 138y 138 = 24.5 + 18 24 = 18.1 + 6 18 = 6.3 + 0 (mdc = 6) Tomando a penltima igualdade; 8 = 56 16.3. Tirando o valor de 16 na primeira igualdade e substituindo na penltima: 8 = 56 (72 56.1).3 8 = 56 + 56.3 72.3 8 = 56.4 + 72(-3). Portanto, x = 4 e y = -3. 6 = 24 18.1 6 = 24 (138 24.5).1 6 = 24 + 24.5 138.1 6 = 24.6 + 138(-1) x = 6 e y = -1

02) Achar inteiros x, y e z que verifiquem as seguintes igualdades: 1) 11x + 19y + 3z = 1 2) 56x + 6y + 32z = 2 3) 6x + 3y + 15z = 9 4) 14x + 7y + 21z = 4 Soluo: 01) Achando o mdc(11, 19) 1 1 2 19 11 8 3 8 3 2 1 Usando o algoritmo da diviso, podemos escrever: 19 = 11 x 1 + 8 11 = 8 x 1 + 3 8=3x2+2 3=2x1+1 2=1x2 Desprezando a ltima igualdade, eliminemos os restos, a partir da penltima igualdade: 1=32 1 = 3 (8 3 x 2) 1=3x38 1 = (11 8) x 3 8 1 = 11 x 3 8 x 4 1 = 11 x 3 (19 11) x 4 1 = 11 x 7 19 x 4 1 2 0 2 1

Achemos agora o mdc(3, 1). Como mdc(3, 1) = 1, vamos escrever este mdc em funo de 3 e 1: 1 = 3 x 1 + 1 x (-2). Agora substituamos o valor de 1, dado na igualdade acima, nesta ltima igualdade: 1 = 3 x 1 + (11 x 7 19 x 4) (2) 1 = 3 x 1 + 11 (14) + 19 x 8 ou 1 = 11 (14) + 19(8) + 3(1). Logo x = 14, y = 8 e z = 1. 02) 56x + 6y + 32z = 2. Achando o mdc(56, 32) 56 24 Usando o algoritmo da diviso, podemos escrever: 56 = 32.1 + 24 32 = 24.1 + 8 24 = 8.3 Desprezando a ltima igualdade e eliminando os restos a partir da penltima: 8 = 32 24 8 = 32 (56 32) 8 = 32.2 + 56(1) (1) Achemos o mdc(8, 6) 1 8 6 2 0 Usando o algoritmo da diviso, podemos escrever: 8=6+2 2 = 8 + 6(1) Agora, substituamos o valor de 8 na expresso (1) 2 = 32(2) + 56(1) + 6(1) 2 = 56(1) + 6(1) + 32(2) .Logo, x = 1, y = 1 e z = 2. 03) 6x + 3y + 15z = 9 Achando o mdc(15, 6) 3 2 1 32 8 1 24 0 3 8

15 3

2 6 0

2 3

Usando o algoritmo da diviso, podemos escrever: 15 = 6.2 + 3 3 = 15(1) + 6(2) (1) Achemos o mdc(3, 3).Como o mdc(3, 3) = 3,vamos escrever 3 como combinao de 3 e 3: 3 = 3(2) + 3(1) Substituamos o valor de 3 encontrado na igualdade (1) nesta ltima igualdade: 3 = 3(2) + [15(1) + 6(2)](1) 3 = 3(2) + 15(1) + 6(2). Como escrever 9 como combinao linear de x, y e z, devemos multiplicar por 3 esta ltima igualdade, obtendo:

9 = 3(6) + 15(3) + 6(6) 9 = 6(6) + 3(6) + 15(3). Logo, x = 6, y = 6 e z = 3 04) 14x + 7y + 21z = 4 Como o mdc(14, 7, 21) = 7 e 7 no divide 4, ento a equao no tem soluo inteira. Calcular: 5) mmc (45, 21). 6) mmc (83, 68). 7) mmc (120, 110). Soluo: 5) Como o mdc(45, 21) = 3, ento, 3.mmc(45, 21) = 45.21. Logo, mmc(45, 21) = 315. 6) Como mdc(83, 68) = 1, ento, 1.MMC(83, 68) = 8.8. Logo, mmc(83, 68) = 5644. 7) Como mdc(120, 110) = 10 , ento, 10.mmc(210, 110) 210.110. Logo, mmc(210, 110) = 1320 08) O mdc de dois inteiros positivos a e b 8 e na sua determinao pelo algoritmo de Euclides os quocientes sucessivamente obtidos foram 2, 1, 1 e 4. Calcular a e b. Resoluo: Temos o seguinte esquema: 4 8 Sabemos que se o mdc 8, o ltimo resto zero e o penltimo 8. Assim, temos: 2 1 1 4 8 8 0 Como 8 o divisor, 4 o quociente e zero o resto, achamos o dividendo desta diviso: 4 x 8 + 0 = 32. Logo o nmero anterior a oito 32. Deste modo 32 ser o outro resto Temos o seguinte esquema: 1 4 32 8 32 8 0 Tendo 32 para divisor, 1 para quociente e 8 para resto, o prximo dividendo ser: 32 x 1 + 8 = 40. De modo semelhante, encontramos os outros nmeros: 184 40 Logo a = 184 e b = 72. 09) Usando o algoritmo de Euclides, determinar: a) mdc(624, 504, 90). Soluo: 2 72 32 1 40 8 1 32 0 4 8 2 1 2 1 1

Pelo processo anterior acha-se o mdc(624, 504) que 24. A seguir acha-se o mdc(24, 90) que 6. R: 6. Determinar os inteiros positivos a e b sabendo: 10) ab = 4032 e o mmc(a, b) = 336. 11) mdc(a, b) = 8 e o mmc(a, b) = 560. Solues: 10) Como mmc(a, b) = 336, temos 336 = a k1 e 336 = b k2 .Multiplicando membro a membro estas duas igualdades, temos: 336 x 336 = a b k1 k2. Substituindo o valor de a b = 4032 nesta ltima igualdade, temos: 112896 = 4032 k1 k2 ou k1 k2 = 28. Assim, como k1 e k2 so primos entre si, devemos procurar dois inteiros primos entre si, cujo produto 28. Encontramos k1 = 1 e k2 = 28, k1 = 4 e k2 = 7. Com estes valores temos a = 336 e b = 12 e a = 84 e b = 48. 11) Temos: mdc(a, b) mmc(a, b) = a b. Ento a b = 8 560. Temos, portanto um problema j resolvido sobre mdc. A resposta ser: a = 8, b = 560; a = 16, b = 280; a = 40, b = 112; a = 56, b = 80. 12) Se a soma de dois nmeros 320 e o mnimo mltiplo comum entre eles 600, quais so esses nmeros? Qual o mximo divisor comum entre eles? Soluo: Se X e Y so os nmeros procurados, eles devem ser divisores de 600, logo devem pertencer ao conjunto D(600): R: {1, 2, 3, 4, 5, 6, 8, 10, 12, 15, 20, 24, 25, 30, 75, 100, 120, 150, 200, 300, 600}. Pares de nmeros deste conjunto que somam 320, so: 300 e 20 ou 200 e 120. O primeiro par no serve, pois MMC (300, 20) = 300. Os nmeros que servem so X = 200 e Y = 120, pois MMC (200, 120) = 600 e MDC (200, 120) = 40. 13) Se a diferena entre dois nmeros naturais 126 e o mximo divisor comum entre eles 18, quais so esses nmeros? Soluo: Se X e Y so os nmeros procurados, eles devem ser mltiplos de 18 e podem ser escritos na forma X = 18a e Y = 18b onde a e b devem ser determinados. Assim: 18a - 18b = 126, de onde segue que 18(a - b) = 187, o que equivalente a: a - b = 7. Tomando a = 8 e b = 1 teremos X = 144 e Y = 18. 14) Se a soma de dois nmeros naturais 420 e o mximo divisor comum entre eles 60, quais so esses nmeros? Soluo: Sejam X e Y os nmeros procurados. Se MDC(X, Y)=60, os nmeros X e Y devem ser mltiplos de 60, logo podem ser escritos na forma X = 60a e Y = 60b onde a e b so nmeros

inteiros positivos. Assim: 60a + 60b = 420, o que garante que a + b = 7. Devemos escolher nmeros naturais tal que a + b = 7, e assim, temos vrias opes. Se a = 6 e b = 1 ento X =360 e Y = 60 Se a = 4 e b = 3 ento X = 240 e Y = 180 Se a = 2 e b = 5 ento X = 120 e Y = 300 Se a = 5 e b = 2 ento X = 300 e Y = 120 Se a = 3 e b = 4 ento X = 180 e Y = 240 Se a = 1 e b = 6 ento X = 60 e Y = 360

Questes Propostas 01) Usando o algoritmo de Euclides, determinar o mdc (306, 657). 02) Usando o algoritmo de Euclides, determinar: a) mdc(285, 675, 405). R: 5. b) mdc(209, 299, 102). R:- 1. c) mdc(69, 398, 253). R: 23. 03) Usando o algoritmo de Euclides, achar inteiros x e y que verifiquem a seguinte igualdade: mdc(56, 72) = 56x + 72y. 04) Achar inteiros x e y que verifiquem a seguinte igualdade: a) 78x + 32y = 2 b) 104x + 91y = 13 c) 31x + 19y = 7 d) 42x + 26y = 16 e) 238x + 51y = 3 f) 52x + 13y = 1 g) 145x + 58y = 87 h) 17x + 5y = -2

05) Achar inteiros x, y e z que verifiquem a igualdade 198x + 288y + 512z = mdc(198, 288, 512). R: x = -5, y = -217, z = 124. 06) Calcular as solues de todos os itens abaixo podendo ser obtidas a partir da propriedade mdc(a,b).mmc(a, b) = a.b. a) mmc(83, 68) c) mmc(86, 71) e) mmc(1287, 507) g) mmc(306, 657) 07) Determinar a e b se, a + b = 589 e R: a = 57 e b = 532; a = 217 e b = 372. 08) Demonstrar que se a e b so inteiros positivos tais que o mdc(a, b) = mmc(a, b) ento a = b. 09) Sendo a e b inteiros positivos, demonstrar quo o mdc(a, b) sempre divide o mmc(a, b). R: 5644 R: 6106 R: 16731 R: 22338
mmc(a , b) mdc(a , b) 84 .

b) mmc( 120, 110) d) mmc(224, 192) f) mmc(143, 227)

R: 1320 R: 1344 R: 32461

10) Quais os dois menores nmeros pelos quais devemos dividir 252 e 234 para que os quocientes obtidos sejam iguais? R: 7 e 9. 11) Quais os nmeros compreendidos entre 100 e 300 divisiveis ao mesmo tempo por 6, 9 e 15? R: 180 e 270. 12) Quais os dois nmeros de trs algarismo divisiveis ao mesmo tempo por 8, 9 e 10? R: 360 e 720. 13) Quais os dois menores nmeros pelos quais devemos multiplicar 30 e 54 para que os produtos obtidos seja iguais? R: 9 e 5. 14) Calcular o nmero que, dividido por 12, 40 e 60 deixa sempre o mesmo resto 5? R: 125. 15) A editora do livro Matemtica recebeu pedidos de trs livrarias sendo que um pedido de 1300 livros, o segundo pedido de 1950 livros e o terceiro pedido de 3900 livros. A editora deseja remeter em n pacotes iguais de tal forma que n seja o menor possvel. Calcule o valor de n. R: 650 livros em cada pacote, num total de 11 pacotes. 16) Trs peas de tecido medem respectivamente, 180m, 252m e 324m. Pretende-se dividir em retalhos de igual comprimento. Qual dever ser esse comprimento de modo que o nmero de retalhos seja o menor possvel? Em quantos pedaos as peas sero dividas? R: O comprimento de 36 m e o nmero de peas sero de 5, 7 e 9 pedaos. 17) Duas rodas dentadas se engrenam uma a outra, a primeira tem 48 dentes e demora 4 segundos em cada volta, a segunda tem 104 dentes. Colocam-se em movimento e se pergunta ao cabo de quanto tempo, se encontram na mesma posio inicial? R: 52 segundos. 18) Dois ciclistas correm sobre uma pista circular, partindo ao mesmo tempo de uma mesma linha. O primeiro realiza uma volta completa, em 30 minutos e o segundo em 36 minutos. Quantas voltas devero dar cada um, para que tornem a encontrar-se, sobre a linha de partida? R: 6 e 5. 19) Um remdio deve ser tomado diariamente em intervalos regulares. O fabricante quer que a durao desses intervalos seja um nmero inteiro de horas (como 3 horas, por exemplo, e nunca trs horas e meia). Alm disso, o fabricante quer que os horrios em que se deve tomar o remdio no mudem de um dia para outro. Existem vrias possibilidades para a durao dos intervalos que satisfazem essas exigncias do fabricante. Quais so elas? R: D(24)
1, 2, 3, 4, 6, 8, 12, 24 .

20) Os planetas Jpiter, Saturno e Urano tm perodo de translao em torno do Sol de aproximadamente 12, 30 e 84 anos, respectivamente. Quanto tempo decorrer, depois de uma observao, para que eles voltem a ocupar simultaneamente as mesmas posies em que se encontram no momento de observao? R: 420 anos. 21) Duas pessoas fazendo seus exerccios dirios partem de um mesmo ponto e contornam, andando, uma pista oval que circula um jardim. Uma dessas pessoas andando de forma mais acelerada d uma volta completa na pista em 12 min, enquanto a outra, andando mais devagar, leva 20 min para completar a volta. Depois de quantos minutos essas duas pessoas voltaro a se encontrar no ponto de partida? R: 60 minutos ou 1 hora. 22) Em um certo pais as eleies para presidente ocorrem de 6 em 6 anos e para senador de 4 em 4 anos. Em 1992 essas eleies coincidiram. D os anos das quatro prximas vezes em que elas voltaram a coincidir. R: 2004, 2016, 2028, 2040. 23) Jos daquelas pessoas que gostam de complicar as coisas. Quando lhe perguntam a sua idade, ele responde Tenho mais de 40 anos, menos de 50 e minha idade um mltiplo de 3 e de 8. Qual a idade do Jos? R: 48 anos. 24) De uma rodoviria, parte um nibus da empresa X a cada 20 minutos e um da empresa Y a cada 45 minutos. Supondo que esses dois nibus partem juntos s 8 horas da manh, depois de quanto tempo os nibus das duas empresas partiram juntos novamente? R: 180 minutos ou 3 horas. 25) Numa estao rodoviria, os nibus para a cidade A partem de 6 em 6 horas, e para a cidade B, de 8 em 8 horas. Numa ocasio, um nibus para a cidade A partiu junto com outro para cidade B. Quanto tempo depois isso acontecer de novo? R: 24 horas. 26) Da Praa da Repblica partem, s 6 horas da manh, dois bondes das linhas X e Y, iniciando o servio do transporte de passageiros. Sabendo-se que o bonde X volta ao ponto de partida ao cabo de 50 minutos, e o Y, ao cabo de 45 minutos, pergunta-se a que horas os dois bondes partiro novamente juntos da praa da Repblica? 27) Tenho trs rguas divididas em partes iguais. Cada parte da primeira tem 3 mm, da segunda, 5 mm, e da terceira, 12 mm. Coloco as trs rguas uma do lado da outra, de modo que as suas extremidades coincidam. Quais so os traos de diviso das trs rguas que coincidem?

Questes Propostas Envolvendo M.D.C e M.M.C 01) Determine s e t inteiros tais que MDC (a, b) = sa + tb para os seguintes pares de inteiros: a) a = 145; b = 72 b) a = 896; b = 143 c) a = -123; b = 32 d) a = -75; b = -15 e) a = 102; b = 49 f) a = 138; b = 24 R: s = 1 e t = -2. R: s = 64 e t = -401. R: s = 13 e t = 50. R: s = 0 e t = 1. R: s = -12 e t = 25. R: s = -1 e t = 6.

02) Classifique cada afirmao abaixo em Verdadeira ou Falsa, justificando: a) MDC de dois nmeros naturais expressos por n e 2 + 1 sempre 1, para qualquer natural n. (V). b) Considere a e b nmeros naturais. Ento MDC(a, ab + 1) = 2. (F). c) MDC de dois nmeros naturais sempre um divisor do MMC destes mesmos nmeros. Se a e b so relativamente primos, MMC(a, b) = |a.b|. (V). 03) Seja a
N:

Determine o mdc (a, a 1) . Quais as possibilidades para o mdc (a, a 2) ? Quais as possibilidades para o mdc (a, a 6) ? Quais as possibilidades para o mdc (a, 3a 5) ?

R: d = 1. R: d = (1, 2). R: d = (1, 2, 3, 6). R: d = (1, 5).

04) Determine todos os nmeros de trs algarismos divisveis por 8, 11 e 12, simultaneamente. R: 264, 528 e 792. 05) Encontre todos os possveis pares de nmeros naturais cujo produto 3600 e cujo mmc 1200. R: a = 3 e b = 1200 ou a = 48 e b = 75. 06) Determine dois nmeros cuja soma 120 e o mmc 144. R: a = 12 e b = 108 ou a = 24 e b = 96. 07) Achar o menor nmero natural que satisfaa simultaneamente as condies: a) quando dividido por 2 tem resto 1. b) quando dividido por 3 tem resto 2. c) quando dividido por 4 tem resto 3. d) quando dividido por 5 tem resto 4. e) quando dividido por 6 tem resto 5. f) quando dividido por 7 tem resto 6. g) quando dividido por 8 tem resto 7. R: 3. R: 5. R: 7. R: 9. R: 11. R: 13. R: 15.

h) quando dividido por 9 tem resto 8. 08) Determine todos os possveis nmeros naturais n tais que: a) mmc(n, 54) = 54 b) mmc(n, 26) = 26 R: D(54) R: D(26)

R: 17.

1, 2, 3, 6, 9, 18, 27, 54 1, 2, 13, 26

09) O mmc dois nmeros naturais a e b igual a 1260 e quando dividimos este mmc pelos nmeros a e b o produto dos quocientes obtidos igual a 90. Determine todos os nmeros naturais a e b satisfazendo esta condio. R: a = 1260 e b = 14, a = 630 e b = 28 e a = 252 e b = 70. 10) O mmc dois nmeros naturais 300. Dividimos este mmc por a e b, os quocientes obtidos so tais que o seu produto vale 50. Determinem todos os pares de nmeros a e b que satisfazem estas condies. R: a = 300 e b = 6 e a = 150 e b = 12. 11) Prove que o produto de trs nmeros consecutivos divisvel por 6. 12) Se o resto da diviso de um nmero primo por 3 1, mostre que na diviso deste nmero por 6 o resto tambm 1. 13) Prove: se o resto da diviso de um nmero inteiro n por 6 5, ento o resto da diviso de n por 3 2. 14) Considere a e b nmeros naturais no primos entre si, cujo produto 420. Determine mdc(a, b). R: 2. 15) Sejam m = 26.33.52, n = 2r.3s.5t e p = 25.54.73. Escreva as condies que devem satisfazer r, s e t para que n seja divisor comum de m e p. R: r = 5, s = 0 e t = 2. 16) Seja x
2 5 3 17 4 41 , y

3 4 17 6 312 e z

2 3 5 41 6 47 2 . Determine:

a) mdc (x,y) . b) mdc (x,z) . c) mdc (x,y,z) . d) mmc (x,y,z) . e) mmc (x,y) . f) mmc (x,z) .

R: 17 4 R: 2.5 R: 2.41 R: 23.34.53.176.312.41.472 R: 2.34.53.176.312.41 R: 23.53.174.416.472

17) Encontre mdc (a,b) e mmc (a,b) , atravs da decomposio em fatores primos: 1) a = 20.600, b = 3.300. 2) a = 147.875, b = 166.725. R: mdc (a, b) = 22.52 e mmc (a, b) = 23.3.52.11.103. R: mdc (a, b) = 52.13 e mmc (a, b) = 53.7.132.19.

18) Encontre os valores de x para os quais mdc(20 + x, x) = 4. R: os valores de x devero ser divisvel por 4. 19) Um professor d aulas numa 7 srie, de 30 alunos, e numa 8 srie, de 18 alunos. Em cada sala, ele formou grupos, e de todos os grupos (tanto na 7 como na 8) tinham o mesmo nmero de alunos. Qual o maior nmero de alunos que cada grupo pode ter? R: Cada grupo pode ter no mximo 6 alunos. 20) Na minha escola, h 180 alunos na 5 srie, 168 na 6 srie, 144 na 7 srie e 120 na 8. Para uma feira de cincias, todas esses alunos sero organizados em grupos com o mesmo nmero de elementos, sem misturar alunos de srie diferentes. a) Qual o nmero mximo de alunos que pode haver em cada grupo? R: Pode ter no mximo 12 pessoas. b) Quantos grupos sero formadas em cada uma das sries? R: 15 grupos na 5 srie; 14 grupos na 6 srie; 12 grupos na 7 srie e 10 grupos na 8 srie. 21) Um pas tem eleies para presidente de 5 em 5 anos, e para governador de 4 em 4 anos. Em 1998, essas duas eleies coincidiram. D os anos das trs prximas vezes em que elas voltaro a coincidir. R: 2018, 2038 e 2058. 22) Um pas tem eleies para presidente de 4 em 4 anos, e para senador de 6 em 6 anos. Em 1997, houve eleies para presidente, e em 1988 para senador. As eleies podero cair alguma vez no mesmo ano? Explique sua resposta. R: no. 23) Muitos cometas nos visitam de tempos em tempos. Um certo cometa passa pela terra de 12 em 12 anos. Outro passa de 32 em 32 anos. Em 1913, os dois passaram por aqui. Qual a prxima ocasio em que os dois passaro pela Terra no mesmo ano? R: 2009. 24) Um cometa A passa pela terra de 26 em 26 anos. O cometa B passa de 32 em 32 anos. Ambos visitaram a Terra em 1930. Pergunta-se: a) Qual ser a prxima ocasio em que os dois visitaro a Terra no mesmo ano? R: 2346. b) Depois de 1930, quantas sero as passagens do cometa a at que os dois visitem a Terra ao mesmo ano? R: 16 passagens o cometa A e 13 passagens o cometa B.

25) Alguns cometam visitam a Terra periodicamente. Um cometa A visita terra de 12 em 12 anos. O cometa B passa de 32 em 32 anos. Ambos visitaram a Terra em 1910. Qual a prxima ocasio em que os dois passaro pela Terra no mesmo ano? R: 2006. 26) Uma rvore de Natal tem trs tipos de luzes. As vermelhas acendem a cada 8 segundos, as verdes a cada 10 segundos e as amarelas a cada 12 segundos. Se elas acenderem todas juntas num determinado momento, depois de quantos segundos ascendero juntas novamente? R: 120 segundos. 27) Numa competio, partiram juntos dois ciclistas. O primeiro leva 20 segundos para dar uma volta completa na pista e o segundo leva 18 segundos. Eles estaro juntos novamente depois de quantos segundos? R: 180 segundos. 28) Uma certa Irm recebe periodicamente a visita de seus trs filhos, Srgio a visita a cada 15 dias, Marta a cada 20 dias e Rodrigo a cada 24 dias. Como hoje dia de seu aniversrio, os trs filhos foram v-la. Daqui a quantos dias coincidira a visita dos trs filhos? R: 120 dias. 29) No terminal de nibus ABCD, chegam nibus da Vila Romana a cada 30 minutos e da Vila Inglesa a cada 40 minutos. De quanto em quanto tempo os horrios dos nibus coincidem? R: 120 minutos. 30) Uma avenida mede 4500 metros. A partir do inicio da avenida, a cada 250 metros h uma parada de nibus, e a cada 225 metros uma para de bonde. Pergunta-se: a) A que distncia do inicio da avenida ocorre a primeira coincidncia das paradas de nibus e de bonde? R: 2250m. b) Quantos so os pontos comuns de paradas de nibus e de bonde? R: 2 paradas comuns. 31) Durante um evento, o organizador pretende distribuir, como brindes, a alguns dos participantes, caixas (kits), com o mesmo contedo, formado de camisetas e chaveiros. Sabe-se que ele possui exatamente 200 camisetas e 120 chaveiros. a) Decomponha os nmeros 200 e 120 em fatores primos? R: 200 = 23.52 e 120 = 2.3.5 b) Determine os nmeros mximos de caixas, com o mesmo contedo, que o organizador conseguir formar utilizando todos os chaveiros e camisetas disponveis? R: 40. 32) (UnB) Quatro pessoas saem de uma praa a caminhar numa mesma hora. Elas repetiro vrias vezes o mesmo percurso, e seus percursos duram respectivamente, 5 min, 9 min, 10 min e 15 min. Aps quantos minutos elas estaro juntas na praa pela primeira vez? R: 90. 33) (UFRJ) Uma escola deseja distribuir cadernos entre os seus 480 alunos, de forma que cada um deles receba o mesmo nmero de cadernos e no haja sobras. Os cadernos so adquiridos pela

escola em pacotes de uma dzia e meia cada. Determine o nmero de pacotes que a escola deve adquirir para que cada aluno receba a menor quantidade possvel de cadernos. R: 80. 34) (UNICAMP) Trs lquidos diferentes, A, B e C, devem ser distribudos em barris iguais. H 108 litros do lquido A, 96 litros do B e 72 litros do C. Para que o nmero de barris seja o menor possvel, qual deve ser a capacidade de cada barril? Quantos barris sero necessrios para conter cada um dos lquidos? R: 12 litros, nmeros de barris: 9, 8 ou 6. 35) (PUC) Dois livros, um dos quais tem 256 pginas e o outro com 160 pginas, so formados por fascculos com o mesmo nmero de pginas (superior a 10 e inferior a 50). Cada fascculo pode ter? R: Pode ter 32 pginas. 36) (PUC) Um lojista dispe de trs peas de um mesmo tecido, cujos comprimentos so 48 m, 60 m e 80 m. Nas trs peas, o tecido tem a mesma largura. Ele deseja vender o tecido em retalhos iguais, cada um tendo a largura das peas e o maior comprimento possvel, de modo a utilizar todo a tecido das peas. Quantos retalhos ele dever obter? R: 47. 37) (UNESP) Trs cidades brasileiras, A, B e C, realizam grandes festas; de 5 em 5 meses em A, de 8 em 8 em B e 12 em 12 meses em C. Essas festas coincidiram em setembro de 1982. Coincidiro novamente em? R: Setembro 1992. 38) (UFES) Trs vergalhes, medindo 400 cm, 480 cm e 720 cm, devem ser cortados em pedaos iguais, de maior tamanho possvel, de modo que cada um deles tenha, por medida, um nmero inteiro de centmetros. Desse modo, sero obtidos. Quantos pedaos? R: 20 pedaos. 39) (UFMG) As medidas tomadas sobre as divisas de um campo de formato triangular so 595 m, 459 m e 340 m. O proprietrio deseja plantar cajueiros nessas divisas, de tal modo que as distancia entre eles sejam iguais e as maiores possveis. Se h um cajueiro em cada canto do campo, a quantidade de cajueiros necessria ao plantio ? R: 82. 40) (UFRN) Para as festas natalinas, uma fbrica de doces lanar uma caixa de chocolates. O nmero de chocolates poder ser dividido igualmente (sem fracion-las) entre 2, 3, 4, 5 e 6 pessoas, no havendo sobra. O menor nmero de chocolates que essa caixa dever conter ser: R: 60. 41) (UFCE) Dois relgios tocam uma msica periodicamente, um deles a cada 60 segundos e o outro a cada 62 segundos. Se ambos tocassem (simultaneamente) s 10 horas, que horas estaro marcando os relgios quando voltaro a tocar juntos (simultaneamente) pela primeira vez aps as 10 horas? R: 10 horas e 31 minutos.

42) (UFMG) Numa repblica hipottica, o presidente deve permanecer 4 anos em seu cargo; os senadores, 6 anos e os deputados, 3 anos. Nessa repblica, houve eleio para os trs cargos em 1989. A prxima eleio simultnea para esses trs cargos ocorrer, novamente, em: R: 2001. 43) (VUNESP) Uma concessionria vendeu no ms de outubro n carros do tipo A e m carros do tipo B, totalizando 216 carros. Sabendo-se que o nmero de carros vendidos de cada tipo foi maior do que 20, que foram vendidos menos carros do tipo A do que do tipo B, isto , n < m, e que MDC(n, m) = 18, os valores de n e m so, respectivamente: R: 90, 126. 44) (UFMG) De uma praa partem, s 6 horas da manha, dois nibus A e B. Sabe-se que o nibus A volta a ponto de partida a cada 50 minutos, e o nibus B, a cada 45 minutos. O primeiro horrio, aps as 6 horas, em que os nibus partiro juntos : R: 13 horas e 30 minutos. 45) (U. E. Londrina - PR) Existem para doao a escolas, 2000 ingressos de um espetculo e 1575 de outro. Cada escola deve receber ingressos para somente um dos espetculos e todas as escolas devem receber a mesma quantidade de ingressos. Distribuindo-se todos os ingressos, o nmero mnimo de escolas que podem ser contemplados nessa doao : R: 143. 46) (U. E. Londrina - PR) Para levar os alunos de certa escola a um museu, pretende-se formar grupos que tenham iguais quantidades de alunos e de modo que em cada grupo todos sejam do mesmo sexo. Se nessa estudam 1350 rapazes e 1224 garotas e cada grupo dever ser acompanhado de um nico professor, o nmero mnimo de professores necessrios para acompanhar todos os grupos nessa visita : R: 143. 47) (UFMG) Entre algumas famlias de um bairro, foi distribudos um total de 144 cadernos, 192 lpis e 216 borrachas. Essa distribuio foi feita de modo que o maior nmero possvel de famlias fosse contemplado e todas recebessem o mesmo nmero de cadernos, o mesmo nmero de lpis e o mesmo nmero de borrachas, sem haver sobra de qualquer material. Nesse caso, o nmero de cadernos que cada famlia ganhou foi de: R: 6. 48) (UE-RJ) Dois sinais luminosos fecham juntos num determinado instante. Um deles permanece 10 segundos fechado e 40 segundos aberto, enquanto o outro permanece 10 segundos fechado e 30 segundos abertos. O nmero mnimo de segundos necessrios, a partir daquele instante, para que os dois sinais voltem a fechar juntos outra vez de: R: 200. 49) (UE-RJ) o nmero de fitas de vdeo que Marcela possui est compreendido entre 100 e 150. Grupando-as de 12 em 12, de 15 em 15 ou de 20 em 20, sempre resta uma fita. A soma dos trs algarismo do nmero total de fitas que ela possui igual a: R: 4.

50) (UFMG) Trs atletas correm numa pista circular e gastam, respectivamente, 2,4 min, 2,0 min e 1,6 min, para completar uma volta na pista. Eles partem do mesmo local e no mesmo instante. Aps algum tempo, os trs atletas se encontram, pela primeira vez, no local da largada. Nesse momento, o atleta mais veloz estar completando: R: 15 voltas. 51) (Cesgranrio-RJ) Certo botnico desenvolveu em laboratrio 3 variedades de uma mesma planta V1, V2 e V3, que se desenvolvem cada uma a seu tempo, de acordo com a tabela abaixo. Plantandose as 3 variedades no mesmo dia, confiando-se na exatido, no ocorrendo nenhum fato que modifique os critrios da experincia tabulada e levando-se em conta que, a cada dia de colheita, outra semente da mesma variedade ser plantada, o nmero mnimo de sementes necessrio para que a colheita das trs variedades ocorra simultaneamente ser:
Variedades Tempo de germinao semanas aps o plantio) (em Tempo de florao (em semanas aps a germinao) Tempo para nica colheita (em semanas aps a florao)

V1 V2 V3 R: 24

4 2 1

3 3 2

1 1 1

52) Numa escola pretende-se distribuir, em partes iguais, 36 puzzles e 90 livros pelas bibliotecas das varias turmas. Qual o nmero Maximo de turmas que a escola pode ter, para que essa distribuio possa ser feita? Para esse numero Maximo, quantos puzzles e quantos livros recebero cada biblioteca de turma? R: 53) Trs amigas, a Ana, a Patrcia e a Lena tiveram folga dos respectivos empregos no sbado passado. Sabendo que a Ana tem folga a um sbado de 6 em 6 semanas, a Patrcia de 3 em 3 semanas e a Lena de 4 em 4 semanas, quantas semanas vo passar at que as trs amigas estejam de folga, em simultneo, a um sbado? R: 54) Duas rodas gigantes comeam girar, num mesmo instante, com uma pessoa na posio mais baixa em cada uma. A primeira d uma volta em 30 segundos e a segunda d uma volta em 35 segundos. As duas pessoas estaro ambas novamente na posio mais baixa aps: R: 55) Trs cidades A, B e C, realizam grandes festas: de 5 em 5 meses em A, de 8 em 8 meses em B e de 12 em 12 meses em C. Essas festas coincidiram em setembro de 1982. Coincidiro novamente em: R:

UNIDADE VI NMEROS PRIMOS 6.1 - Introduo: No ano de 2002 trs matemticos indianos descobriram um algoritmo de primariedade, que informa se um dado nmero primo ou no. Essa descoberta divulgada pela imprensa causou uma preocupao mundial devido os cdigos criptogrficos que utilizam os nmeros primos. J tinha lido sobre criptografia e sabia da sua importncia para a proteo das informaes, o que despertou o meu interesse sobre o alcance desta descoberta, e na medida que a pesquisa se desenvolvia outros movimentos em torno dos nmeros primos apareciam, envolvendo desde matemticos e tcnicos de computao profissionais at usurios de computadores domsticos. O nmero a entidade mais importante da Matemtica estando na origem de diversos ramos desta cincia. Entre os seres vivos, o homem um dos poucos que possui senso numrico. Por isso, desde os primrdios da raa humana os nmeros j estavam presentes, tendo surgido para auxiliar o homem a controlar quantidades a partir do contraste entre pouco e muito, resultando na criao dos primeiros sistemas de contagem. Juntamente com a linguagem, a escrita e outras habilidades, o nmero est no conjunto das criaes humanas em que se baseou o desenvolvimento das nossas sociedades. Nesta unidade, falaremos sobre nmeros, mas de um tipo especial os nmeros primos. um assunto em que muitos especialistas em segurana eletrnica de dados tem conhecimento, e a grande maioria das pessoas no sabem que a inviolabilidade dos seus dados pessoais depende em parte destes nmeros. Os nmeros primos, um conhecimento sem aplicao desde as civilizaes mais antigas, so a base dos cdigos de segurana de informao para computadores. Como estamos vivendo, segundo alguns historiadores e socilogos na "Era da Informao" pode-se perceber sua importncia para a nossa vida diria, embora no apaream de forma explcita. A propsito, podemos citar a frase do matemtico Nicolai Lobachevsky (1793-1856) "No h ramo da Matemtica, por abstrato que seja, que no possa um dia vir a ser aplicado nos fenmenos do mundo real". Os primos so apresentados pela primeira vez aos alunos na 5a srie e depois so quase esquecidos. No nvel mdio, apesar do aluno estar mais amadurecido para a Matemtica, eles no reaparecem, embora pudessem ajudar na fixao do contedo especfico, assim como devido ao fascnio que exercem por conta das curiosidades e mistrios que os envolvem, despertar no aluno o gosto por problemas da Teoria dos Nmeros. Cabe ressaltar, que os nmeros primos tem ganhado importncia por causa das aplicaes na criptografia, deixando de ser uma mera curiosidade. Desta forma, um papel de destaque est reservado para o conhecimento matemtico, j que ele a "porta de entrada" para o mundo tecnolgico. Segundo Ubiratan D' Ambrosio (1996)" a

educao para a cidadania, que um dos grandes objetivos da educao de hoje, exige uma apreciao do conhecimento moderno, impregnado de cincia e tecnologia". Os nmeros primos so um exemplo para os alunos, de como podemos a partir de uma definio antiga e relativamente simples, construir uma teoria que foi sendo enriquecida ao longo do tempo de outros conhecimentos, culminando no seu aproveitamento em aplicaes tecnolgicas de ltima gerao. 6.2 - Nmeros Primos: Definio 6.1 - Dizemos que um nmero inteiro positivo p maior que 1 primo, se, e somente se, p possui exatamente dois divisores positivos distintos, ou seja, 1, p . Exemplo: O nmero 2 primo, pois os divisores positivos de 2 so
1, 2 . E mais, 2 o nico

nmero primo par, pois se existe primo par maior que 2, seria da forma N = 2q (q 1). Portanto, 1, 2 e q so divisores de N, o que torna absurdo, pois N primo. Um inteiro maior que 1 e que no primo diz-se composto. Teorema 6.1: Se um nmero primo p no divide um inteiro a, ento a e p so primos entre si. Corolrio 6.1: Se p um primo tal que p | ab, ento p | a ou p | b. Corolrio 6.2: Se p um primo tal que p | a1a2a3 ... an, ento existe um ndice k, com 1 que p | ak.. Corolrio 6.3: Se os inteiros p, q1,q2 ,..., qn so todos primos e se p | q1q2 ... qn, ento existe um ndice k, com 1 k n tal que p = qk.. k n tal

Teorema 6.2: - Todo inteiro composto possui um divisor primo. 6.3 - Teorema Fundamental da Aritmtica: Teorema 6.3 Todo inteiro positivo n > 1 igual a um produto de fatores primos. Corolrio 6.4: A decomposio de um inteiro positivo n > 1 como produto de fatores primos nica, a menos da ordem dos fatores. Corolrio 6.5: Todo inteiro positivo b > 1 admite uma nica decomposio da forma n =
k p1 1 p k 2 ... p k r onde, para i =1,2,..., r cada ki um inteiro positivo e cada pi um primo, com p1 < p2 < 2 r

... < pr, denominada decomposio cannica do inteiro positivo n > 1. Exemplo 6.1: Definir mdc e mmc dos nmeros 588 e 936 pela decomposio cannica. Teorema 6.4 - (de Euclides) - H um nmero infinito de primos. Teorema 6.5 - Se um inteiro positivo a > 1 composto, ento a possui um divisor primo p 6.4 - Crivo de Eraststenes:

a.

A construo de uma tabela de primos que no um dado inteiro n faz-se usando o processo conhecido pelo nome de crivo de Eraststenes, e que consiste no seguinte: escrevem-se na ordem natural todos os inteiros desde 2 at n e, em seguida, eliminam-se todos os inteiros composto que so mltiplos dos primos p tais que p 6.5 - Primo Gmeos: Definio 6.2 Chama-se primos gmeos de dois inteiros positivos mpares e consecutivos que so ambos primos. Exemplos: 3 e 5; 5 e 7; 11 e 13; 17 e 19; 29 e 31. No se sabe at hoje se h um nmero infinito de pares de primos gmeos muito grandes, tais como: 140.737.488.353.507 140.737.488.353.699 e e 140.737.488.353.509 140.737.488.353.701

n , isto , 2p, 3p, 4p .

Um fato interessante a existncia de apenas um terno de inteiros positivos mpares e consecutivos que so todos primos: 3, 5 e 7. 6.4 - Seqncia de Inteiros Consecutivos Compostos: Teorema 6.5 - Existem seqncias de n inteiros positivos consecutivos e compostos, qualquer que seja o inteiro positivo n. A seqncia a seguinte: (n + 1)! + 2, (n + 1)! + 3, ... , (n + 1)! + (n + 1). Os seus n termos so inteiros positivos consecutivos, e cada um deles composto, porque (n + 1)! + J. divisvel por j se 2 Exemplo: Suponha n = 4, obtemos a seqncia. 5! + 2, 5! + 3, 5! + 4, 5! + 5
j n 1.

Cujos termos so 4 inteiros positivos consecutivos, cada um dos quais composto, pois, temos: 5! + 2 = 122 = 2.61 5! + 4 = 124 = 4.31 5! + 3 = 123 = 3.41 5! + 5 = 125 = 5.25

Outra seqncias de 4 inteiros positivos consecutivos e composto existem, tais como: 24, 25, 26, 27 54, 55, 56, 57 32, 33, 34, 35 74, 75, 76, 77

Nota: Chama-se: Nmeros primos: primeiros ou indecomponveis e Nmeros no primos: secundrios ou compostos.

Questes Resolvidos 01) Achar todos os primos que so iguais a um quadrado perfeito menos 1. Soluo: Seja p este primo. Ento, p = n2 1 ou p = (n 1)(n + 1). Como p primo seus fatores s podem ser 1 e p. Assim temos: n 1 = 1 e n + 1 = p. Da, conclumos que n = 2 e p = 3 que o primo pedido. 02) Achar todos os primos que so iguais a um cubo perfeito menos 1. Soluo: Seja p este nmero primo. Ento p = n3 1 ou p = (n 1)(n2 + n + 1). Como p primo seus nicos fatores so 1 e p. Temos n 1 = 1 e n2 + n + 1 = p. Da tiramos n = 2 e p = 7. 03) Determinar todos os inteiros positivos tais que n, n + 2 e n + 4 so todos primos. Soluo: que se n um inteiro qualquer, um dos inteiros n, n + 2 e n + 4 divisvel por 3. Sendo assim s existe a possibilidade da seqncia ser 3, 5 e 7. 04) Determinar todos os primos p tais que 3p + 1 um quadrado perfeito. Soluo: Temos 3p + 1 = n2 ou 3p = n2 1 ou 3p = (n 1)(n + 1). Como p primo a decomposio do primeiro membro deve ser igual a do segundo. Assim 3 = n 1 e p = n + 1. Da, n = 4 e p = 5. 05) Achar uma seqncia de 100 inteiros positivos consecutivos e compostos. Soluo: (100 + 1)! + 2, (100 + 1)! + 3, ... , (100 + 1)! + 101. 06) Mostrar que todo primo, exceto 2 e 3, da forma 6k 1 ou 6k + 1, onde k um inteiro positivo. Soluo: Como o nmero primo no divisvel por 6, e esto excludos os primos 2 e 3, os restos da diviso deste primo por 6, s podero ser 1 ou 5. Assim p = 6 q + 1 ou p = 6q + 5. A primeira igualdade j est na forma exigida, fazendo q = k; e se p = 6q + 5, basta somarmos e subtrairmos um ao segundo membro, obtendo: p = 6q + 5 + 1 1 ou p = 6q + 6 1 ou p = 6(q + 1) 1 o que d p = 6k 1, fazendo q + 1 = k. 07) Achar o menor inteiro positivo pelo qual se deve dividir 3720, para se obter um quadrado perfeito. Soluo: Achando a decomposio cannica de 3720 temos: 3720 = 23 x 3 x 5 x 31. Para que um nmero seja um quadrado perfeito os expoentes de seus fatores primos, tm de ser par. Ento o menor nmero que deve dividir o 3720 ser 2 x 3 x 5 x 31 = 930. 08) Achar todos os primos que so divisores de 50! Soluo: So todos os primos menores que 50, pois 50! = 1 x 2 x 3 x ... x 49 x 50 Logo 2, 3, 5, 7, ... , 47. 09) Mostrar que todo inteiro da forma n4 + 4, com n > 1 composto.

Soluo: Vamos decompor o nmero em um produto de fatores maiores que um n4 + 4 = n4 + 4 + 4n2 4n2 (somando e subtraindo 4n2 para formar um quadrado perfeito) n4 + 4 = (n2 + 2)2 4n2 (fatoremos a diferena entre dois quadrados) n4 + 4 = (n2 + 2 + 2n)(n2 + 2 2n). Sendo n > 1 os dois fatores so maiores que 1. 10) Mostrar que, se n > 4, composto, ento n divide (n 1)!. Soluo: Como n composto, ele pode ser decomposto como produto de dois inteiros a e b: n = a b , com 1 < a < n e 1 < b < n. Suponhamos que a b e consideremos a < b. Temos: 1 < a < b < n, ou 1 <a<b n 1. Logo (n 1)! = 1.2.a.b. (n1) sendo a e b fatores de (n1)!. Deste modo ab = n divide (n1)!. Se a = b, n = a2 e como n > 4, temos a2 > 4 ou a > 2 e a2 > 2 a ou 2 a < n = a2. Assim 2 a n 1 e como a < 2 a, a e 2 a so fatores de (n1)! Logo: (n1)! = 1.2.3...a ...2 a ... (n1). Portanto a2 um fator de (n1)!. E assim, a2 = n divide (n1)! 11) Demonstrar que o inteiro positivo a > 1 um quadrado perfeito se e somente se todos os expoentes dos fatores primos da sua decomposio cannica so inteiros pares. Soluo: Seja a um quadrado perfeito ento a tem a forma n2. Se n = n1k1.n2k2.n3k3. ... nnkn, onde n1k1.n2k2.n3k3. ... nnkn a decomposio cannica em fatores primos de n, teremos n2 = (n1k1.n2k2.n3k3. ... nnkn)2 = n12k1.n22k2.n32k3. ... nn2kn. Como todos os expoentes so da forma 2k, conclui-se que todos os expoentes so pares. Seja ento ni um fator primo de n cujo expoente no seja par. Neste caso, o fator teria expoente da forma 2k + 1. Ora, ni2k + 1 = ni2k . ni. ni2k tem expoente para, portanto est de acordo com o que foi dito anteriormente. Para que fosse quadrado, ni deveria ter dois fatores primos iguais. Como ni primo isto no possvel. Portanto, todo nmero quadrado perfeito, se e somente se, todos os expoentes dos fatores primos na decomposio cannica for par. 12) Demonstrar que, se o inteiro n composto, ento 2n 1 tambm composto. Soluo: Evidentemente se trata de n positivo, pois se n < 0, n 1 negativo e 2n 1 no ser um inteiro. Assim, para n > 0 e n composto, n > 3. Teremos ento n 1 > 2 2n 1 = 2k, k = n 1 > 2 inteiro: k 2 > 0 (k 1) > 0. Ora, 2n 1 = 2k = 2(2k 1) que mltiplo de 2 (no se esquea que k 1 > 0) 2n 1 composto. 13) Mostrar que so primos gmeos: ( a ) 1949 e 1951 Soluo: Primos gmeos so dois primos que so mpares consecutivos. 1949 e 1951 so dois mpares consecutivos. Devemos verificar se ambos so primos. 1949 e 1951. Como ambos no so divisveis por 2, 3, 5, 7, 11, 13, 17, 19, 23, 29, 31, 37, 41 e 43, os dois so primos. Portanto so primos gmeos.

Questes Propostas 01) Achar os cinco menores primos da forma n2 2. R: 2, 7, 23, 47 e 79. 02) Achar trs primos mpares cuja soma seja: ( a ) 81. R: (3, 5, 73) ( b ) 125. R: (5, 13, 107) 03) Achar todos os pares de primos p e q, tais que p q = 3. R: p = 5 e q = 2. 04) Determinar se so primos os nmeros. ( a ) 169 R: composto. ( b ) 197 R: primo ( c ) 239 R: primo ( d ) 473 R: composto 05) Achar a decomposio cannica do inteiro 5040. R: 24 . 32. 5 . 7 06) Achar o mdc(a, b) e mmc(a, b) sabendo a = 230 . 521 . 19 . 233 e b = 26 . 3 . 74 . 112 . 195. 237. R: Mdc(a, b) = 26 . 19. 233 e (a, b) = 230.3.521.74.112.195.237. 07) Mostrar que so primos gmeos: ( a ) 1997 e 1999. R: so primos gmeos. 08) Achar todos os primos gmeos entre 400 e 500. R:- 491 e 421, 461 e 463. 09) Achar uma sequncia de quatro inteiros positivos consecutivos e compostos. R: 5! + 2, 5! + 3, 5! + 4, 5! + 5 10) Verificar a conjectura de Goldbach para os seguintes inteiros pares: ( a ) 32 ( b ) 100 ( c ) 456 ( d ) 1024

11) Verificar que todo par entre 4 e 100 a soma de dois primos. 12) Achar o menor inteiro positivo n tal que 2n2 + 29 um inteiro composto. R: 1 13) Mostar que a soma de inteiros positivos mpares e consecutivos sempre um inteiro composto. 14) Usando a decomposio cannica dos inteiros 507 e 1287, achar o mdc(507, 1287) e o mmc(507, 1287). R: mdc = 39 e mmc = 16731. 15) Mostrar que o nico primo da forma n3 1 7. R: o nico n3 1 primo 7. 16) Mostrar que todo inteiro da forma 8n + 1, com n > 1, composto. 17) Mostrar que se n2 + 2 primo ento 3 | n. 18) Mostrar, mediante um exemplo, que a seguinte conjectura falsa: Todo inteiro positivo pode-se escrever sob a forma a2 + p, onde o inteiro a > 0 e p um inteiro primo ou 1. 19) Demonstrar as seguintes propriedades: ( a ) Todo primo da forma 3n + 1 tambm da forma 6m + 1. ( b ) Todo inteiro da forma 3n + 2 tem um fator primo desta forma. ( c ) Se p > 5 um primo, ento p2 + 2 composto. ( d ) Se p um primo e se p | an, ento pn | an. ( e ) Todo inteiro n > 11 pode ser expresso como a soma de dois inteiros compostos. ( f ) Se p > q > 5 e se p e q so ambos primos, ento 24 | p2 q2. ( g ) Se p 5 um primo mpar, ento p2 1 ou p2 + 1 divisvel por 10.

20) Verificar que todo inteiro pode escrever-se sob a forma 2km, onde o inteiro k > 0 e m um inteiro mpar. 21) Demonstrar que, se o inteiro n > 2, ento existe um primo p tal que n < p < n!. 22) Demonstrar que todo primo mpar da forma 4k + 1 ou 4k 1, onde k um inteiro positivo.

UNIDADE VII EQUAES DIOFANTINAS 7.1 - Introduo: Diofante foi um grande matemtico que dedicou - se resoluo de problemas. Sua mais importante obra foi a Aritmtica, uma coleo de 13 livros nos quais o autor reuniu cerca de 150 problemas resolvidos atravs de operaes numricas, nas quais demonstra seu alto grau de habilidade e engenho. Tambm chamado de pai da lgebra, devido a sua contribuio na introduo de notaes algbricas, Diofante utilizou abreviaes para a subtrao, a igualdade e a incgnita. Epitfio de Diofante: Bastante curioso o epitfio de Diofante, matemtico grego da Antiguidade, que viveu 200 anos a.C. Encontramos na Antologia Grega um problema que apresentado sob a forma de epitfio: Eis o tmulo que encerra Diofante, maravilha de contemplar. Com um artifcio aritmtico a pedra ensina a sua idade. Deus concedeu-lhe passar a sexta parte de sua vida na juventude; um duodcimo na adolescncia; um stimo em seguida, foi passado num casamento estril. Decorreram mais cinco anos, depois do que lhe nasceu um filho. Mas este filho desgraado e, no entanto, bem amado! apenas tinha atingido a metade da idade que viveu seu pai, morreu. Quatro anos ainda, mitigando sua prpria dor com o estudo da cincia dos nmeros, passou-os Diofante, antes de chegar ao termo de sua existncia. Em linguagem algbrica o epigrama da Antologia seria traduzido pela equao: Diofante. Resolvendo essa equao, achamos x = 84. Trata-se, afinal, de uma equao muito simples do 1 grau com uma incgnita vamos verificar alguns exemplos: Considere o seguinte problema (1): Se um trabalhador recebe 510 reais em tquetes de alimentao, com valores de 20 reais ou 50 reais cada tquetes, de quantas formais pode ser formado o carn de tquetes desse trabalhador. Soluo: Se x denota a quantidade de tquetes de 20 reais e y denota a quantidade de tquete de 50 reais ento a equao 20x 50y 510 , deve ser satisfeita e o problema resolvido determinando-se todas as solues todas as solues inteiras no negativas desta equao. Considere o seguinte problema (2): Se o custo de postagem de uma encomenda e de 85 centavos e devemos usar selos de 6 centavos e de 15 centavos, como devemos combinar os selos na postagem. Soluo:

x x x x + + +5+ + 4 x , na qual x representa o nmero de anos que viveu 6 12 7 2

Se x denota a quantidade de selos de 6 centavos e se y denota a quantidade de 15 centavos ento a equao 6x 15y 85 , deve ser satisfeita e o problema resolvido determinando-se todas as solues todas as solues inteiras no negativas desta equao. Desta forma passamos estudar nesta unidade a resoluo das equaes diofantinas e suas aplicaes. 7.2 - Equaes Diofantinas: Definio 7.1: Equao diofantina linear, uma equao da forma ax + by = c em que a, b, c, so nmeros inteiros. Uma soluo de uma equao diofantina linear um par de inteiros x 0, y0 que satisfaz a equao. Exemplos 7.1: 3x + 6y = 18 solues 4 e 1, 6 e 6, 10 e 2 etc... 2x + 4y = 7 no tem soluo, pois o primeiro membro ser sempre par e o segundo membro mpar. 7.3 - Condio de Existncia de Soluo: Teorema 7.1 - A equao diofantina linear ax + by = c tem soluo se, e somente se, d = mdc (a, b) divide c. 7.4 - Solues da Equao ax + by = c: Teorema 7.2 - Se d = mdc(a, b) divide c (d | c), e se o par de inteiros x0, y0 uma soluo particular da equao diofantina linear ax + by = c, ento todas as outras solues desta equao so dadas pela frmula: x = x0 +
b d

t e y = y0

a d

t onde t um inteiro arbitrrio.

Corolrio 7.1 - Se o mdc(a, b) = 1 e se (x0, y0) uma soluo particular da equao diofantina ax + by = c, ento todas as outras solues desta equao so dadas pelas frmulas: x = x0 + bt e y = y0 at onde t um inteiro arbitrrio.

Questes Resolvidas Determinar todas as solues inteiras das seguintes equaes diofantinas lineares: 01) 56x + 72y = 40 Soluo: Determinemos o mdc (72, 56).

Como o mdc (56, 72) = 8 divide 40, a equao possui soluo. Usando o algoritmo da diviso, temos: 72 = 56 x 1 + 16 56 = 16 x 3 + 8 16 = 8 x 2 Vamos escrever o mdc 8 como combinao linear de 56 e 72. 8 = 56 16 x 3 = 56 (72 56) x 3 = 56 x 4 72 x 3 = 56(4) + 72 (3) Como queremos resolver a equao 56x + 72y = 40 multipliquemos a ltima igualdade acima por 5 40 = 56(20) + 72(15).Logo xo = 20 e yo = 15, dando a soluo geral: x = 20 + (72/8)t e y = 15 (56/8)t ou x = 20 + 9t e y = 15 7t. 02) 24x + 138y = 18 Soluo: Determinemos o mdc (138, 24).

Usando o algoritmo da diviso, temos: 138 = 24 x 5 + 18 24 = 18 x 1 + 6 18 = 6 x 3 Vamos escrever o mdc 6, como combinao linear de 24 e 138 6 = 24 18 = 24 (138 24 x 5) = 24 x 6 138 = 24(6) + 138(1) Como queremos resolver a equao 24x + 138y = 18, multipliquemos a igualdade acima por 3 18 = 24(18) + 138(3): Logo xo = 18 e yo = 3, dando a soluo geral: x = 18 + (138/6)t e y = 3 (24/6)t ou x = 18 + 23t e y = 3 4t. 3) 221x + 91y = 117

Soluo: Determinemos o mdc (221, 91).

Como o mdc (221, 91) = 13 divide 117, a equao possui soluo. Usando o algoritmo da diviso, temos: 221 = 91 x 2 + 39 91 = 39 x 2 + 13 39 = 13 x 3 Vamos escrever o mdc 13 como combinao linear de 221 e 91 13 = 91 39 x 2 = 91 (221 91 x 2) x 2 = 91 x 5 221 x 2 = 221(2) + 91(5) Como queremos resolver a equao 221x + 91y = 117, multipliquemos a igualdade acima por 9. 117 = 221(18) + 91(45). Logo xo = 18 e yo = 45 dando a soluo geral: x = 18 + (91/13)t e y = 45 (221/13)t ou x = 18 + 7t e y = 45 17t. 04) 84x 438y = 156 Soluo: Determinemos o mdc (438, 84).

Como o mdc (438, 84) = 6 divide 156, a equao possui soluo. Usando o algoritmo da diviso, temos: 438 = 84 x 5 + 18 84 = 18 x 4 + 12 18 = 12 x 1 + 6 12 = 6 x 2 Vamos escrever o mdc 6 como combinao linear de 84 e 438 6 = 18 12 = 18 (84 18 x 4) = 18 x 5 84 = (438 84 x 5) x 5 84 = 438 x 5 84 x 26 ou 6 = 84(26) 438(5) Como queremos resolver a equao 84x 438y = 156 multipliquemos a igualdade acima por 26. 156 = 84(676) - 438(130). Logo xo = 676 e yo = 130, dando a soluo geral: x = 676 (438/6)t e y = 130 (84/6)t ou x = 676 73t e y = 130 14t. 05) 57x 99y = 77 Soluo: Determinemos o mdc (57, 99).

Como o mdc (57, 99) = 3 no divide 77, a equao no possui soluo. 06) 11x + 30y = 31 Soluo: Determinemos o mdc (11, 30).

Como o mdc (11, 30) = 1 divide 31, a equao possui soluo. Usando o algoritmo da diviso, temos: 30 = 11 x 2 + 8 11 = 8 x 1 + 3 8=3x2+2 3=2x1+1 2=1x2 Vamos escrever o mdc 1 como combinao linear de 11 e 30. 1 = 3 2 = 3 (8 3 x 2) = 3 x 3 8 = (11 8) x 3 8 =11 x 3 8 x 4 = = 11 x 3 (30 11 x 2) x 4 1 = 11 x 11 30 x 4 = 11(11) + 30(4) Como queremos resolver a equao 11x + 30y = 31 multipliquemos a igualdade acima por 31 31 = 11(341) + 30(124). Logo xo = 341 e yo = 124, dando a soluo geral: x = 341 + 30t e y = 124 11t. 07) 27x 18y = 54 Soluo: Determinemos o mdc (27, 18).

Como o mdc.(27, 18) = 9 divide 54, a equao possui soluo. Usando o algoritmo da diviso, temos: 27 = 18 x 1 + 9 18 = 9 x 2 Vamos escrever o mdc 9 como combinao linear de 27 e 18.

9 = 27 18 = 27(1) 18(1) Como queremos resolver a equao 27x 18y = 54 multipliquemos a igualdade acima por 6 54 = 27(6) 18(6). Logo xo = 6 e yo = 6 dando a soluo geral: x = 6 (18/9)t e y = 6 (27/9)t ou x = 6 2t e y = 6 3t. 08) 13x 7y = 21 Soluo: Determinemos o mdc (13, 7).

Como o mdc (13, 7) = 1 divide 21, a equao possui soluo. Usando o algoritmo da diviso, temos: 13 = 7 x 1 + 6 7=6x1+1 6=1x6 Vamos escrever o mdc 6 como combinao linear de 13 e -7. 1= 7 6 = 7 (13 7) = 7 x 2 13 = 13(1) 7(2) Como queremos resolver a equao 13x 7y = 21 multipliquemos a igualdade acima por 21 21 = 13(21) 7(42). Logo xo = 21 e yo = 42, dando a soluo geral: x = 21 7t e y = 42 13t 09) 44x + 66y = 11 Soluo: Determinemos o mdc (66, 44).

Como o mdc (44, 66) = 22 no divide 11, a equao no possui soluo. 10) 21x 12y = 72 Soluo: Determinemos o mdc (21, 12).

Como o mdc (21, 12) = 3 divide 72, a equao possui soluo. Usando o algoritmo da diviso, temos: 21 = 12 x 1 + 9 12 = 9 x 1 + 3

9=3x3 Vamos escrever o mdc 3 como combinao linear de 21 e -12. 3 = 12 9 = 12 (21 12) = 12 x 2 21 = 21 (1) 12(2) Como queremos resolver a equao 21x 12y = 72 multipliquemos a ltima igualdade acima por 24 72 = 21 (24) 12(48). Logo xo = 24 e yo = 48, dando a soluo geral: x = 24 (12/3)t e y = 48 (21/3)t ou x = 24 4t e y = 48 7t. 02) Determinar todas as solues inteiras e positivas das seguintes equaes diofantinas lineares 01) 5x 11y = 29 Soluo: A equao geral desta equao : x = 58 11t e y = 29 5t Como queremos as solues positivas, devemos ter: 58 11t > 0 e 29 5t > 0 Da 1a. desigualdade, tiramos: t < 5,27 ou t ser t 6 e da 2a, t < 5,8 ou t 6. Sendo assim, a soluo 6, dando uma infinidade de solues positivas.

02) 32x + 55y = 771 Soluo: A soluo geral desta equao : x = 9252 + 55t e y = 5397 32t Como queremos as solues positivas, devemos ter: 9252 + 55t > 0 e 5397 32t > 0 Da primeira desigualdade, tiramos: t > 168,21 ou t Da segunda desigualdade, tiramos: t < 168,5 ou t 03) 58x 87y = 290 Soluo: A soluo geral desta equao : x = 10 3t e y = 10 2t Como queremos as solues positivas, devemos ter: 10 3t > 0 e 10 2t > 0 Da primeira desigualdade, tiramos: t < 3,33 ou t Da segunda desigualdade, tiramos: t < 5 ou t 6, satisfazem o problema. 04) 62x + 11y = 788 Soluo: A soluo geral desta equao : x = 2364 + 11t e y = 13396 62t Como queremos as solues positivas, devemos ter: 2364 + 11t > 0 e 13396 62t > 0 Da primeira desigualdade tiramos: t > 214,9 ou t Da segunda desigualdade tiramos: t < 216,06 ou t duas condies so: t = 215 e t = 216. Quando t = 215, x = 1 e y = 66 e quando t = 216, x = 12 e y = 4. 05) 30x + 17y = 300 Soluo: A soluo geral desta equao : x = 1200 + 17t e y = 2100 30t Como queremos as solues positivas, devemos ter: 1200 + 17t > 0 e 2100 30t > 0 215 216. Logo os valores de t que satisfazem as 4 6. Logo todos os valores de t no intervalo t 169 168. Ento, no h solues positivas.

Da primeira desigualdade tiramos: t > 70,58 ou t Da segunda desigualdade tiramos: t < 70 ou t 06) 54x + 21y = 906

70

71. Portanto no h solues positivas.

Soluo: A soluo geral desta equao : x = 604 + 7t e y = 1510 18t Como queremos as solues positivas, devemos ter: 604 + 7t > 0 e1510 18t > 0 Da primeira desigualdade tiramos: t > 86,28 ou t Da segunda desigualdade tiramos: t < 83,88 ou t t = 86, t = 85 e t = 84 dando os valores: Para t = 86, x = 2 e y = 38 Para t = 85, x = 9 e y = 20 Para t = 84, x = 16 e y = 2 07) 123x + 360y = 99 Soluo: A soluo geral desta equao : x = 1353 + 120t e y = 462 41t Como queremos as solues positivas, devemos ter: 1353 + 120t > 0 e 462 41t > 0 Da primeira desigualdade tiramos: t > 11,27 ou t Da segunda desigualdade tiramos: t < 11, 26 ou t positivas. 08) 158x 57y = 7 Soluo: A soluo geral desta equao : x = 154 57t e y = 427 158t Como queremos as solues positivas, devemos ter: 154 57t > 0 e 427 158t > 0 Da primeira desigualdade tiramos: t < 2,7 ou t Da segunda desigualdade tiramos: t < - 2,7 ou t problema, dando infinitas solues positivas. Questes Propostas 01) Determinar todas as solues inteiras e positivas das seguintes equaes diofantinas lineares: 01) 90x - 28y = 22 02) 40x - 65y = 135 03) 18x - 20y = -8 04) 8x - 13y = 23 05) 3x + 4y = 20 06) 50x - 56y = 74 07) 8x - 13y = 23 08) 5x - 2y = 2 R: x = 13, y = 41. R: x = 5, y = 1. R: x = 4, y = 4. R: No existem solues positivas. R: x = 4, y = 2. R: x = 25, y = 21. R: x = 11, y = 5. R: x = 2, y = 4. 3 - 3. Ento todos os valores de t - 3 satisfazem o 11 -12. Portanto a equao no possui solues 86 84, o que nos d o intervalo 86,84 ou

09) 24x + 138y = 18 10) 93x + 81y = 3 11) 43x + 128y = 1 12) 16x + 7y = 601 13) 47x + 29y = 1288 14) 30x + 17y = 201 15) 17x + 13y = 100 16) 12x + 18y = 50 17) 60x + 18y = 67 18) 1402x + 1969y = 1 19) 102x + 1001y = 533 20) 33x + 25y = 0 21) 56x + 634y = 168 22) 5x + 7y = 14 25) 172x + 20y = 1000 26) 18x + 5y = 48 27) 39x + 26y = 105 28) 14x + 22y = 50

R: No existem solues positivas. R: No existem solues positivas. R: No existem solues positivas. R: x = 3, y = 79 e x = 31, y = 15. R: x = 20, y = 12. R: x = 5, y = 3. R: No existem solues positivas. R: No admite soluo. R: No admite soluo. R: No existem solues positivas. R: No existem solues positivas. R: x = 0, y = 0. R: x = 0, y = 2. R: R: R: R: R:

02) Encontre as solues das seguintes equaes diofantinas: 01) 2x - 10y + 35z = 0 02) 2x + 3y + 5z = 7 03) 1521x + 1955y + 455z = 221 04) 101x - 102y + 103z = 1 05) 12x + 21y + 9z + 15w = 9 R: x = 0, y = 0 e z = 0. R: x = 7, y = -14 e z = 7. R: x = 7956, y = -84.442 e z = 336.141. R: x = 1, y = -100 e z = -100. R: x = 2, y = -1, z = -1 e w = 1.

03) Determinar o menor inteiro positivo que dividido por 8 e por 15 deixa os restos 6 e 13, respectivamente. R: 188. 04) Exprimir 100 como soma de dois inteiros positivos de modo que o primeiro seja divisvel por 7 e o segundo seja divisvel por 11. R: 56 e 44. 05) Determinar as duas menores fraes positivas que tenham 13 e 17 para denominadores e cuja soma seja igual a 305|221. R: 8|13 e 13|17.

06) Determine duas fraes cujos denominadores sejam 12 e 16 e cuja soma seja 10|48. R: 1|12 e 2|16.

07) Demonstrar que se a e b so inteiros positivos primos entre si, ento a equao diofantina ax by = c, tm um nmero infinito de solues inteiras e positivas. 08) Um grupo de pessoas gastou 1000 dlares num hotel. Sabendo-se que apenas alguns dos homens estavam acompanhados pelas esposas e que cada homem gastou 19 dlares e cada mulher gastou 13 dlares, pede-se determinar quantas mulheres e quantos homens estavam no hotel? R: 41 homens e 17 mulheres. 09) Ache o inteiro estritamente positivo com a seguinte propriedade da resto 6 quando dividido por 11 e resto 3 quando dividido por 7?. R: 17. 10) Determine todos os mltiplos de 11 e de 9 cuja a soma igual a: a) 79. R: no existe. b) 80. R: 36 e 44. c) 270. R: 99 e 171; 72 e 198.

11) Determine o menor inteiro positivo que tem restos 11 e 35 quando dividido respectivamente por 37 e 48. R: 899. 12) O valor da entrada de um cinema e R$ 8,00 e da meia entrada R$ 5,00. Qual o menor nmero de pessoas que pode assistir a uma sesso de maneira que a arrecadao da bilheteria seja R$ 500,00. R: Este problema no tem uma nica soluo. As solues possveis so: x = 5 e y = 92 ou x = 60 e y = 4. 13) Ao entrar num bosque alguns viajantes avistaram 37 montes de maa. Aps serem retiradas 17 frutas, o restante foi dividido igualmente entre 79 pessoas. Quantas frutas coube a cada pessoa? R: 4 para cada pessoa. 14) Dispondo de 100 reais, quais so as quantias que se podem gastar comprando selos de R$ 5,00 e de R$ 7,00. R: x = 6 e y = 10. 15) Numa criao de coelhos e galinhas contaram-se 400 ps. Quantas so as galinhas e quantos so os coelhos, sabendo que a diferena entre esses dois nmeros seja a menor possvel? R: 99 coelhos e 2 galinhas. 16) Um grupo de pessoas gastou 690 dlares num hotel. Sabendo-se que apenas alguns dos homens estavam acompanhados pelas esposas e que cada homem gastou 18 dlares e cada mulher gastou 15 dlares, pede-se determinar quantas mulheres e quantos homens estavam no hotel. R: Este problema no tem uma nica soluo. As solues possveis so: 25 homens e 16 mulheres ou 30 homens e 10 mulheres ou 35 homens e 4 mulheres.

17) Um teatro vende ingressos e cobra R$ 18,00 por adulto e R$ 7,50 por criana. Numa noite, arrecada R$ 900,00. Quantos adultos e crianas assistiram ao espetculo, sabendo que eram mais adultos do que criana? R: Este problema no tem uma nica soluo. As solues possveis so: x = 0 e y = 50 ou x = 12 e y = 45 ou x = 24 e y = 40. 18) Uma agncia de correios possui apenas selos de 14 centavos e de 21 centavos. Determine as combinaes desses selos que podem ser feitas para postar cartas dos seguintes valores postais: (a) R$ 3,50 R: x 1 y 16 x 4 y 14 x 7 y 12 x 10 y 10 x 13 y 8 x 16 y 6 x 19 y 4 x 22 y 2 x 25 y 0

(b) R$ 4,00 R: No admite soluo (c) R$ 7,77 R: Varias solues por ter o valor de entre 37 t 55 . 19) Com R$ 5,49 podemos comprar maas, a 18 centavos cada, e perras, a 33 centavos cada. Qual e o numero mnimo de frutas que podem ser compradas? R: 18 frutas. 20) Um estudante, viajando da Europa aos Estados Unidos, troca seus francos suos e francos franceses por dlares. Ele recebe US$ 17,06 tendo recebido US$ 0,19 (19 cents') por cada francos francs e US$ 0,59 por cada francos suos. Quanto de cada moeda ele possua? R: Francos Francs 7,03 e Francos Suos 17,03. 21) De que modos e possvel combinar 50 moedas, misturando moedas de 1, de 10 e de 25 centavos, de modo a totalizar 3 reais? R: x = 1, y = -3 e z = 7. 22) Temos duas balanas: uma que marca pesos mltiplos de 10 e outra que marca pesos mltiplos de 13. Como que com essas balanas podemos pesar 107 gramas? R: x = 12, y = 9. 23) Apenas com a utilizao de dois relgios que s do intervalos de tempo de 5 e de 11 minutos como podemos cozer um ovo durante 3 minutos? R: No existem solues positivas.

24) Numa papelaria vendem-se dois tipos de canetas por 110 e 70 reais respectivamente. Ao fim de um dia a importncia total recebida pela venda dessas canetas foi 6570 reais. Qual o menor numero possvel de canetas vendidas? E qual o maior? R: Qual o menor numero possvel de canetas 63. E qual o maior 91. 25) Subindo uma escada de dois em dois degraus, sobra um degrau. Subindo a mesma escada de trs em trs degraus, sobram dois degraus. Determine quantos degraus possui a escada, sabendo que o seu nmero e mltiplo de 7 e esta compreendido entre 30 e 100. R: 35 26) Se um trabalhador recebe 510 reais em tquetes de alimentao, com valores de 20 reais ou 50 reais cada tquetes, de quantas formais pode ser formado o carn de tquetes desse trabalhador. R: carn com 3 tquetes de R$ 20,00 reais e 9 tquetes de R$ 50,00, com 8 tquetes de R$ 20,00 reais e 7 tquetes de R$ 50,00, com 13 tquetes de R$ 20,00 reais e 5 tquetes de R$ 50,00, com 18 tquetes de R$ 20,00 reais e 3 tquetes de R$ 50,00, com 23 tquetes de R$ 20,00 reais e 1 tquetes de R$ 50,00.

UNIDADE VIII CONGRUNCIAS 8.1 - Introduo: O conceito de congruncia, bem como a notao atravs da qual essa noo tornou um dos instrumentos mais poderosos da teoria dos nmeros, foi introduzido por Karl Friedrich Gauss (1777 1855), em sua obra Disquisitions arithmeticae em 1801. Para dar uma idia da noo de congruncia consideremos a seguinte questo, talvez ingnua mas ilustrativa: Se hoje sexta-feira, que dia da semana ser daqui a 1520 dias? Para organizar o raciocnio, indiquemos por o dia de hoje (sexta - feira), e por 1 o dia de amanha (sbado), e assim por diante. A partir dessa escolha, podemos construir a seguinte tabela:
Sexta 0 7 Sbado 1 8 Domingo 2 9 Segunda 3 10 Tera 4 11 Quarta 5 12 Quinta 6 13

Nossa questo agora se resume em saber que coluna se encontra o nmero 1520. Para isso basta observar que dois nmeros da seqncia 0, 1, 2, , esto na mesma coluna se, e somente se, sua diferena divisvel por 7. Suponhamos que o nmero 1520 se encontre na coluna encabeada pelo nmero a (0 a
6) , logo 1520 - a = 7q. 6) , Ora, pela

Para algum inteiro positivo q. donde obtemos 1520 = 7q + a, com (0 a

unicidade do resto na diviso euclidiana, segue dessa igualdade que o resto da diviso de 1520 por 7. portanto 1520 = 217 7 + 1, desta forma conclui-se que o resto 1 e que portanto 1520 esta na segunda coluna. Logo, daqui a 1520 dias ser um sbado. 8.2 - Congruncias: Definio 8.1 - Sejam a e b inteiros quaisquer e seja m um inteiro positivo fixo. Diz-se que a congruente a b mdulo m se, e somente se, m divide a diferena a b. Em outros termos a congruente a b mdulo m se, e somente se, existe um inteiro k tal que a b = km. Notao: a b (mod m) Simbolicamente: a 24 (mod 7); 31 b (mod m) se, e somente se, m | ( a b ). 63 (mod 8).

Exemplos 8.1 - 3

11 (mod 6); 15

Definio 8.2 - Se m no divide a diferena a b, ento, diz-se que a, incongruente a b mdulo m. Notao: a Observaes: 1) Dois inteiros quaisquer so congruentes mdulo 1. 2) Dois inteiros so congruentes mdulo 2, se ambos so pares ou ambos mpares. 3) a 0 (mod m) se, e somente se, m | a. b (mod m).

Exemplos 8.2:

1) Mostrar que n 7 (mod 12) ento, n 2) Mostrar que n2 0 (mod 4) ou n2

3 (mod 4) n 1 (mod 4) n Z.

Z.

8.3 - Caracterizao de Inteiros Congruentes: Teorema 8.1 - Dois inteiros a e b so congruentes mdulo m se, e somente se, a e b deixam o mesmo resto quando divididos por m. 8.4 - Propriedades das Congruncias: Teorema 8.2- Seja m um inteiro positivo fixo (m > 0) e sejam a, b e c inteiros quaisquer. Subsistem as propriedades: 1) a a (mod m). b (mod m), ento b b (mod m) e se b a (mod m). c(mod m), ento a c (mod m)

2) Se a 3) Se a

Teorema 8.3 - Seja m um inteiro positivo fixo (m > 0) e sejam a, b dois inteiros quaisquer. Subsistem as seguintes propriedades: 1) Se a 2) Se a 3) Se a b (mod m) e se n | m, com n > 0, ento a b (mod m) e se c > 0, ento ac b (mod n).

bc (mod mc).

b (mod m) e se a, b, m so todos divisveis pelo inteiro d > 0, ento

a d

m b mod . d d

Teorema 8.4 - Seja m um inteiro positivo fixo (m > 0) e sejam a, b, c, d inteiros quaisquer. Subsistem as seguintes propriedades: 1) Se a 2) Se a 3) Se a b (mod m) e se c d (mod m), ento a + c b + c (mod m) e ac b + d (mod m) e ac bc (mod m). bd (mod m).

b (mod m), ento a + c b (mod m), ento an

bn (mod m) para todo inteiro positivo n.

Exemplos 8.3 - Mostrar que: a) Se a b (mod m), ento a c (mod m) ento a b (mod m). c b (mod m). b mod
m . d

b) Se a + b

Teorema 8.5 - Se ac Corolrio 8.1 - Se ac Corolrio 8.2 - Se ac

bc (mod m) e se o mdc(c, m) = d, ento a bc (mod m) e se o mdc (c, m) = 1, ento a

b (mod m). b (mod p).

bc (mod p), com p primo, e se p no divide c, ento a

8.4 - Sistemas Completos de Restos: Definio 8.3 - Chama-se sistema completo de restos mdulo m todo conjunto S = {r1, r2,..., rm} de m inteiros tal que um inteiro qualquer a congruente mdulo m a um nico elemento de S.

Exemplos 8.3 - {1, 2, 3}, {0, 1, 2}, { 1, 0, 1}, {1, 5, 9} so sistemas completo de restos mdulo 3. Teorema 8.6 - O conjunto S = {0, 1, 2, ..., n1 } um sistema completo de restos mdulo m. Corolrio 8.3 - Se S = {r1, r2,..., rm} um sistema completo de restos mdulo m, ento os elementos de S so congruentes mdulo m aos inteiros 0, 1, 2, ... , m 1, tomados numa certa ordem. Questes Resolvidas 01) Achar o menor inteiro positivo que represente a soma: ( a ) 5 + 3 + 2 + 1 + 8 (mod. 7). Soluo: 5 + 2 0 (mod.7), 3 + 1 4 (mod. 7) 8 5 (mod. 7) 1 (mod. 7).

Portanto 5 + 3 + 2 + 1 + 8 R: 5

0+4+1

( b ) 2 + 3 1 + 7 2 (mod.4). Soluo: 2 + 3 1 (mod. 4) e -1 + 7 2 = 4 1+0 0 (mod. 4).

Portanto: 2 + 3 1 + 7 2 R. 1 02) Sabendo que 1066

1 (mod. 4)

1776 (mod m), achar todos os possveis valores do mdulo m.

Soluo: Como m | (1066 1776) devemos achar todos os divisores positivos de 710. Logo os divisores positivos de 710 so: 1, 2, 5, 10, 71, 142, 355 e 710. 03) Exprimir que n mpar de trs outras maneiras. Soluo: 01) n 02) n 1 (mod. 2) - 1 (mod. 2)

03) n = 2k + 1 04) n = 2k 1 04) Mostrar que todo primo (exceto 2) congruente mdulo 4 a 1 ou 3. Soluo: Se p primo, diferente de 2, ento ele mpar. Dividindo p por 4 obtemos os restos 1 ou 3. Assim p = 4q + 1 ou p = 4q + 3. Na primeira igualdade p 1 = 4q, ento p segunda igualdade p 3 = 4q , dando p 05) Mostrar que 1110 Soluo: 112 1110 = 112. 118 1 (mod 100). 212 41 (mod 100); 118 412 81 (mod 100) 3 (mod 4). 1 (mod 4) e na

21 (mod 100); 114 21 . 81

1 (mod 100).

06) Mostrar que todo primo (exceto 2 e 3) congruente mdulo 6 a 1 ou 5.

Soluo: Se p um primo diferente de 2 e 3, ento ao dividirmos p por 6, obtemos os restos 1 ou 5. Assim p 1 (mod 6) ou p 5 (mod 6).

07) Mostrar que 41 divide 220 1. Soluo: Devemos mostrar que 220 26 220 23 (mod 41); 24 402 1 (mod 41). 23 . 16 40 (mod 41) 16 (mod 41); 210= 26 . 24

1 (mod 41).

08) Mostrar que 89 | (244 - 1). Soluo: Devemos mostrar que 244 27 39 (mod 89); 24 244 = (211)4 1 (mod 89) 39 . 16 1 (mod 89) 16 (mod 89); 211 = 27 . 24

1 (mod 89).

09) Mostrar que 97 | (248 1). Soluo: Devemos mostrar que 248 27 31 (mod 97); 25 248 = (212)4 Logo 248 224 (mod 97); 222 1 (mod 97). 1 (mod 97). 31 . 32 22 (mod 97) (-1)2 (mod 97) -1 (mod 97); 224 = (222)2 32 (mod 97); 212 = 27 . 25 96

10) Determinar quais dos seguintes conjuntos so sistemas completos de restos mdulo 4: a) {2, 1, 0, 1}. Soluo: 2 2; 1 3; 0 0; 1 1 (mod 4). Portanto um sistema completo de restos mdulo 4.

b) {0, 4, 8, 12}. Soluo: 0 mdulo 4. c) {13, 4, 17, 18}. Soluo: 13 4. ( d ) -5, 0, 6, 22 . Soluo: -5 = 4(-2) + 3 3, 0 0; 6 = 4.1 + 2 2; 22 = 4.5 + 2 3; 4 0; 17 1; 18 2 (mod 4). Portanto um sistema completo de restos mdulo 0; 4 0; 8 0; 12 0 (mod 4). Portanto no um sistema completo de restos

O conjunto equivalente a {0, 2, 3}. Portanto, no um conjunto completo. R: So conjuntos completos os das letras a e c. Questes Propostas 01) Achar todos os inteiros x tais que 0
x 15 e 3x

6 (mod 15).

R: x = 2; x = 7 e x = 12. 02) Achar todos os inteiros x tais que 1 x 100 e x R:7, 24, 41, 58, 75 e 92. 03) Sabendo que k 1 (mod 4), mostrar que 6k + 5 3 (mod 4). 7 (mod 17).

04) Achar os restos das divises de 250 e 4165 por 7. R: 4 e 6. Demonstrar as seguintes proposies: 05) Se a um inteiro mpar, ento a2 06) Se a um inteiro qualquer, ento a3 07) Se a um inteiro qualquer, ento a3 08) Mostrar, mediante um exemplo, que a2 09) Demonstrar que, se a 1 (mod 8). 0, 1 ou 8 (mod 9). a (mod 6). b2 (mod.m) no implica a b (mod.m).

b (mod. m) ento mdc(a, m) = mdc(b, m). bk (mod. m) e k j no implica aj bj.

10) Mostrar, mediante um exemplo, que ak 11) Achar os restos das seguintes divises: 1) 245 por 7. 2) 11100 por 100. 3) 310.425+68 por 5. 4) 52.4841+285 por 3. 5) 710 por 51. 6) 2
100

R: r = 1. R: r = 1. R: r = 1. R: r = 0. R: r = 19. R: r = 1. R: r = 49. R: r = 1. R: r = 126. R: r = 3. R: r = 5. R: r = 0. R: r = 13. R: r = 28.

por 11.

7) 734 por 51. 8) 14256 por 17. 9) 521 por 127. 10) 1212 por 5. 11) (116 + 1717)21 por 8. 12) 1316-225.515 por 3. 13) 23728 por 13. 14) 563 por 29. 12) Mostrar que 47 | (223 1) : 13) Para todo n N, Mostrar que:

1) 1016n 1 divisvel por 70.

R: r = 1.

2) 198n 1 divisvel por 17.

R: r = 1.

14) Achar os restos da diviso por 7 do nmero: 1) 1010 + 1010 + 1010 +........+ 1010 . 2) 16 + 26+.......+1006. 3) 17 + 27+.......+1007. 4) 22225555 + 55552222. 15) Achar os restos da diviso por 4 do nmero: 1) 1 + 2 + 22+........+219. 2) 15 + 25+.......+1005. R: r = 3. R: r = 0.
2 3 100

R: r = 4. R: r = 2. R: r = 3. R: r = 0.

16) Determinar quais dos seguintes conjuntos so sistemas completos de restos mdulo 6. a) {1, 2, 3, 4, 5}. R: No , porque possui somente 5 elementos. b) {0, 5, 10, 15, 20, 25}. R: Portanto um sistema completo de restos mdulo 6. c) {4, 3, 2, 1, 0, 1}. R: Portanto um sistema completo de restos mdulo 6. d) {17, 4, 6, 7, 10, 3}. R: Portanto um sistema completo de restos mdulo 6. 17) Achar um sistema completo de restos {p1, p2, ... p7} mdulo 7, tal que todo pi primo. R: r = {2, 3, 5, 7, 11, 13, 28}. 18) Achar um sistema completo de restos mdulo 7 formado s de mltiplos no negativos de 4. R: r = { 36, 40, 44, 48, 52, 56, 60}. 19) Encontrar um sistema completo de restos mdulo 11 formado somente por mltiplo de 6. R: r: 0, 1, 2, 3, 4, 5, 6, 7, 8, 9, 10 . 20) Encontrar um sistema completo de restos mdulo 7 onde todos os elementos so primos: R: r: 0, 1, 2, 3, 4, 5, 6 . 21) Dado um primo p sempre possvel encontrar um sistema completo de restos mdulo p formado s por primos? Justificar. Sua resposta. R:

UNIDADE XI CONGRUNCIAS LINEARES 9.1 - Introduo: Usamos congruncias rotineiramente em nossa vida. Por exemplo, na contagem das horas, fazemos uso de congruncia modulo 12 ou modulo 24. Na contagem de minutos e segundos, usamos congruncia modulo 60. Nos calendrios, na contagem dos dias da semana fazemos uso, de certa forma, da congruncia modulo, e na contagem dos meses, empregamos congruncia modulo 12. Os hodmetros dos carros, usados para marcar quilmetros rodados, geralmente trabalham modulo 100.000. Veremos nesta unidade como resolve congruncias lineares e aplicando na resoluo de equaes diofantinas. 9.2 - Congruncias Lineares: Definio 9.1: Chama-se congruncia linear toda equao da forma ax inteiros quaisquer e m um inteiro positivo. Todo inteiro x0 tal que ax0 soluo da congruncia linear. Observao 9.1 - Se x0 uma soluo da congruncia linear ax b (mod m), ento todos os b (mod m), onde a e b so b (mod m) diz-se uma

inteiros x0 + km, onde k um inteiro arbitrrio, tambm so solues da congruncia linear. Observao 9.2 - Duas solues x0 e x1 da congruncia ax , x0 b (mod m) congruente mdulo m isto

x1 (mod m) no so consideradas solues distintas. O nmero de solues da congruncia

dado pelo nmero de solues mutuamente incongruente mdulo m. 9.3 - Condio de Existncia de Solues: Teorema 9.1 - A congruncia linear ax b (mod m) tem soluo se, e somente se, d divide b, sendo

d = mdc(a, m). logo ax 0 b my0 ou ax 0 my0 = b . 9.4 - Solues da Congruncia ax b (mod m): b (mod m) tem

Teorema 9.2 - Se d divide b, sendo d = mdc(a, m), ento a congruncia linear ax precisamente d solues mutuamente incongruentes mdulo m, dada pela frmula: x = xo +
m t, 0 d

d 1. b (mod m) tem uma nica

Corolrio 9.1 - Se o mdc(a, m) = 1, ento a congruncia linear ax soluo mdulo m.

9.5 - Resoluo de Equaes Diofantinas Lineares por Congruncias: Uma equao diofantina linear, uma equao da forma ax + by = c em que a, b, c, so nmeros inteiros. e de acordo com o Teorema 7.1 A equao diofantina linear ax + by = c tem soluo se, e

somente se, d = mdc (a, b) divide c. Uma soluo de uma equao diofantina linear um par de inteiros x0, y0 que satisfaz a equao, ento: ax0 + by0 = c o que implica: ax0 e ax0 - c = - by

b (mod m), Assim sendo, para obter uma soluo particular da equao c (mod

diofantina linear, basta determinar uma soluo qualquer x = x0 da congruncia linear ax

b), e substituir este valor x0 de x na equao ax + by = c afim de encontrar o valor correspondente y0 de y, isto , tal que ax0 + by0 = c. Obviamente, tambm se pode obter uma soluo particular da equao diofantina linear, determinando uma soluo qualquer y = y0 da congruncia linear. bx (mod a). Questes Resolvidas 01) Resolver as seguintes congruncias lineares: a) 2x 1 (mod 17). c

Soluo: O mdc(2, 17) = 1 e 1 | 1, logo a congruncia possui uma soluo. 2x 1 (mod 17); 1 2.9 (mod 17); 2x 2.9 (mod 17); x 9 (mod 17) b) 3x 1 (mod 17). Soluo: O mdc(3, 17) = 1 e 1 | 1, logo a congruncia possui uma soluo. 3x 1 (mod 17); 1 3.6 (mod 17); 3x 3.6 (mod 17); x 6 (mod 17) c) 3x 6 (mod 18). Soluo: O mdc(3, 18) = 3, e 3 | 6, logo a congruncia possui trs solues. Dividindo por 3 a congruncia dada, obtemos: x 2, x = 8 e x = 14 d) 25x 15 (mod 29). Soluo: O mdc(25, 29) = 1 e 1 | 15, logo a congruncia possui uma soluo 25x 15 (mod 29) dividindo por 5, temos: 5x 3 (mod 29); 3 5.18 (mod 29) 5x 5.18 (mod 29); x 18 (mod 29) e) 5x 2 (mod 26). Soluo: O mdc(5, 26) = 1, logo a congruncia possui uma soluo 5x 2 (mod 26); 2 5.16 (mod 26); 5x 5.16 (mod 26); x 16 (mod 26) f) 6x 15 (mod 21). Soluo: O mdc(6, 21) = 3 e 3 | 15, logo a congruncia possui trs solues 6x 15 (mod 21). Dividindo por 3, temos : 2x 5 (mod 7); 5 2.6 (mod 7) 2x 2.6 (mod 7); x 6 (mod 7). A soluo geral : x = 6 + 7t, t = 0, 1 e 2, dando: 2 (mod 6) Assim a soluo geral x = 2 + 6t, t = 0, 1 e 2 dando x =

x = 6, x= 13 e x = 20 g) 36x 8 (mod 102). Soluo: O mdc(36, 102) = 6 e como 6 no divide 8, a congruncia no possui soluo. 02) Resolver por congruncia as seguintes equaes diofantinas lineares. a) 12x + 25y = 331. Soluo: O mdc(12, 25) = 1 e 1 | 331, logo a equao diofantina possui soluo 12x 331 = 25(-y) da, 12x 331 (mod 25); 331 12.13 (mod 25) 12x 12.13 (mod 25); x 13 (mod 25). Assim x0 = 13 uma soluo particular da equao diofantina linear. Substituindo este valor na equao diofantina, obtemos: 12.13 + 25y = 331; 25y = 331 - 156; 25y = 175; y0 = 7 A soluo geral : x = 13 + 25t, y = 7 12t. b) 5x 53y = 17. Soluo: O mdc(5, 53) = 1 e 1 | 17, logo a equao diofantina possui soluo 5x 17 = 53y da, 5x 17 (mod 53); 17 5.14 (mod 53); 5x 5.14 (mod 53) x 14 (mod 53). Assim x0 = 14 uma soluo particular da equao diofantina linear. Substituindo este valor na equao diofantina, obtemos: 5.14 53y = 17; 53y = 17 - 70; 53y = 53; y0 = 1. A soluo geral : x = 14 53t, y = 1 5t. c) 7x + 6y = 9. Soluo: O mdc(7, 6) = 1 e 1 | 9, logo a equao diofantina possui soluo 7x - 9 = 6(y) da, 7x 9 (mod 6); 9 7.3 (mod 6); 7x 7.3 (mod 6)

x 3 (mod 6). Assim x0 = 3 uma soluo particular da equao diofantina linear. Substituindo este valor na equao diofantina, obtemos: 7.3 + 6y = 9. 6y = 9 21; 6y = 12; y0 = 2. A soluo geral : x = 3 + 6t, y = 2 7t. 03) Determinar o nmero de solues de cada uma das seguintes congruncias lineares: (a ) 3x 6 (mod. 15)

Mdc(3, 15) = 3. Como 6 mltiplo de 3, a equao tem duas solues. ( b ) 4x 8 (mod. 15)

Mdc(4, 15) = 1. 8 mltiplo de 1. Portanto, a equao tem 1 soluo. ( c ) 5x 10 (mod. 15)

mdc(5, 15) = 5. Como 10 mltiplo de 5, a equao tem 2 solues. ( d ) 6x = 11 (mod. 15)

mdc(6, 15) = 3. Como 11 no mltiplo de 3, a equao no tem soluo. Questes Propostas 01) Resolver as seguintes congruncias lineares: 1) 34x 2) 8x 3) 14x 4) 5x 5) 3x 6) 23x 7) 7x 60(mod 98) 16(mod 12) 36(mod 48)
3(mod 24) 5(mod 7) 7(mod19)

R: x R: x R: x R: x R: x R: x R: x R: x R: x R: x R: x

45 e 94 (mod 98). 2, 5, 8, e 11 (mod. 12). 6 ou 30 (mod. 48). 4 (mod. 24). 4 (mod. 7). 16 (mod. 19). 13 (mod. 18). 15, 39, 63, 87 e 111 (mod. 120). 30 e 61 (mod. 48). 3 (mod. 12). 3, 11, e 19 (mod. 24).

5(mod18)

8) 25x 15(mod120) 9) 14x


36(mod 48)

10) 5x 15(mod12) 11) 3x


9(mod 24)

02) Resolver por congruncia as seguintes equaes diofantinas lineares. ( a ) 4x + 51y = 9 ( b ) 7x + 6y = 9 ( c ) 11x + 27y = 4 ( d ) 79x 131y = 6 ( e ) 39x + 26y = 104 ( f ) 61x 11y = 81 ( g ) 65x + 77y = 200 ( h ) 51x + 85y = 1037 R: x = 15 + 51t e y = -1 4t. R: x = 3 + 6t e y = - 2 7t R: x = 20 + 27t e y = -8 11t R: x = 42 - 131t e y = 24 - 75t R: x = 2t e y = 37 + 61t R: x = 8 - 11t e y = 37 - 61t R: x = 9 + 77t e y = -5 65t R: x = 2 + 5t e y = 11 3t

03) Determinar o nmero de solues que pode ter uma congruncia linear cujo mdulo 20. R: 1, 2, 4, 5, 10 ou 20 solues pois estes so os divisores de 20. 04) Demonstrar que se d = mdc (a, m) e se d | b, ento as congruncias lineares ax (a|d)x (b|d) (mod. m|d) tm precisamente as mesmas solues. b (mod. m) e

05) Encontrar todas as solues de cada uma das seguintes congruncias lineares: 1) 5x
3(mod 7)

R: x = 15 + 51t e y = -1 4t R: R:

2) 13x 14(mod 29) 3) 15x


9(mod 25)

4) 37x 16(mod 19) 5) 5x


20(mod 15)

R: R: R: R: R: R: R: R: R:

6) 3x 1(mod 25) 7) 9x 1(mod 65) 8) 6x 10 (mod 22) 9) 14x 1(mod 77) 10) 15x
9 (mod 12)

11) 6x 10 (mod 22) 12) 9x 12 (mod 21)

UNIDADE X SISTEMAS CONGRUNCIAS LINEARES 10.1 - Introduo: No sculo um, o matemtico chins chamado Sun-Tsu se perguntou? Que nmero ser a esse de forma que quando dividido por 3, o resto 2; quando dividido por 5, o resto 3, e quando dividido por 7, o resto 2?. A pergunta Qual a soluo para o seguinte sistema de congruncias?
x x x 2 (mod 3) 3 (mod 5) 2 (mod 7)

Definio 10.1 - Um sistema de congruncias lineares um sistema da forma abaixo:


A1x A2x Ar x B1 (mod m1 ) B2 (mod m 2 ) Br (mod m r )

Arx

Br (mod mr) onde Ai, i = 1,2, ..., r so inteiros supostamente no nulos. Uma soluo

do sistema um inteiro x0 que soluo de cada uma das congruncias que dele fazem partes. Exemplo 10.1: 3x 1 (mod 5) 2x 3 (mod 9). a2 (mod m2) admite soluo se, e somente se, a1

Teorema 10.1 - Um sistema x

a1 (mod m1); x

a2 divisvel por d = mdc(m1, m2). Neste caso, se x0 uma soluo particular do sistema e se m = mmc(m1, m2) ento x = x0 (mod m) sua soluo geral. Corolrio 10.1 - Um sistema de congruncias lineares x a1 (mod m1); x a2 (mod m2); ... ; x ar

(mod mr) admite soluo se, e somente se, ai aj divisvel por dij = mdc(mi, mj). Nesse caso se x0 uma soluo particular, ento a soluo geral do sistema dada por x mmc(m1, m2, ... , mr). Exemplo 10.2: x 1 (mod 5); x 3 (mod 4); x 9 (mod 6). x0 (mod m) onde m =

Teorema 10.2 - (do Resto Chins): Sejam m1, m2,..., mr nmeros inteiros maiores que zero e tais que mdc(mi, mj) = 1, i j. Faamos m = m1m2...mr e sejam b1, b2, ... , br, respectivamente, solues

das congruncias lineares ;x

m y mj

1 (mod mj). Ento o sistema x

a1 (mod m1); x

a2 (mod m2); ...

ar (mod mr) admite solues para quaisquer a1, a2, ... , ar

e sua soluo geral dada por:


M (mod m). mr

a1b1

M m1

a 2 b2

M m2

... a r br

Este algoritmo, utilizado para resolver sistemas de congruncias lineares, muito antigo e foi inventado, independentemente, pelos chineses e pelos gregos, para resolver problemas de

astronomia. O algoritmo chins do resto tem este nome porque um dos primeiros lugares em que aparece foi no livro Manual de aritmtica do mestre Sun-Tsu, escrito entre 287 d.C. e 473 d.C. Exemplo: x 1 (mod 2); x 2 (mod 3); x 3 (mod 5).

Questes Resolvidas 01) Resolver os seguintes sistemas de congruncias lineares: a) x 1 (mod 2), x 1 (mod 3). Soluo: Como o mdc(2, 3) = 1 o sistema possui soluo. A soluo geral da 1a. x = 1 + 2a, substituindo este valor na 2a, obtemos 1 + 2 a 1 (mod 3); 2 a 0 (mod 3); a 0 (mod 3). Logo a = 3b; substituindo este valor em x = 1 + 2a, temos x = 1 + 2(3b), dando x = 1 + 6b que a soluo geral do sistema ou x 1(mod 6) . b) x 5 (mod 12), x 7 (mod 19). Soluo: Como mdc(12, 19) = 1 o sistema possui soluo. A soluo geral da 1a. x = 5 + 12a; substituindo este valor na 2a, obtemos: 5 + 12 a 7 (mod 19); 12 a 2 (mod 19); 6 a 1 (mod 19); 1 6.16 (mod 19) 6; a 6.16 (mod 19); a 16 (mod 19). A soluo geral a = 16 + 19b; substituindo este valor em x = 5 + 12 a, temos x = 5 + 12(16 + 19b), dando x = 197 + 228b, que a soluo do sistema ou x 197(mod 228) . 02) Resolver os seguintes sistemas de congruncias lineares: a) x 3 (mod 5), x 5 (mod 7), x 7 (mod 11). Soluo: Como mdc(5, 7) = mdc(5, 11) = mdc(7, 11) = 1 o sistema possui soluo. A soluo geral da 1a. x = 3 + 5 a, substituindo este valor na 2a, obtemos 3 + 5 a 5 (mod 7); 5 a 2 (mod 7); 2 5.6 (mod 7); 5 a 5.6 (mod 7) a 6 (mod 7) dando a soluo geral a = 6 + 7b; substituindo este valor em x = 3 + 5 a temos: x = 3 + 5(6 + 7b) = 33 + 35b. Substituindo este valor na 3a equao, temos: 33 + 35b 7 (mod 11); 35b 26 (mod 11); 35b 2b (mod 11); 2b 26 (mod 11). b 13 (mod 11) dando a soluo b = 13 + 11c. Substituindo este valor na expresso x = 33 + 35 b, temos x = 33 + 35(13 + 11c) = 422 + 385c que a soluo geral do sistema
x 348(mod 385) .

b) x 1 (mod 5), x 5 (mod 7), x 7 (mod 11). Soluo: Como o mdc(5, 7) = mdc(5, 11) = mdc(7, 11) = 1 o sistema possui soluo. A soluo geral da 1a congruncia : x = 1 + 5 a.; substituindo este valor na 2a., obtemos 1 + 5 a 5 (mod 7); 5 a 4 (mod 7); 4 5.5 (mod 7); 5 a 5.5 (mod 7) a 5 (mod 7), dando a soluo geral a = 5 + 7b; substituindo este valor em x = 1 + 5 a, obtemos x = 1 + 5(5 + 7b) = 26 + 35b; substituindo este valor na 3a congruncia, temos:

26 + 35b 7 (mod 11); 35b 19 (mod 11); 19 35.7 (mod 11). 35b 35.7 (mod 11); b 7 (mod 11), dando a soluo geral b = 7 + 11c; substituindo este valor em x = 26 + 35b, x = 26 + 35(7 + 11c) = 271 + 385c que a soluo geral do sistema ou
x 271(mod 385) .

c) x 5 (mod 6), x 4 (mod 11), x 3 (mod 17). Soluo: Como o mdc(6, 11) = mdc(6, 17) = mdc(11, 17) = 1 o sistema possui soluo. A soluo geral da 1a congruncia : x = 5 + 6 a, substituindo este valor na 2a congruncia, obtemos: 5 + 6 a 4 (mod 11); 6 a 1 (mod 11); 1 6.9 (mod 11). 6 a 6.9 (mod 11); a 9 (mod 11), dando a soluo geral a = 9 + 11b; substituindo este valor em x = 5 + 6 a, obtemos: x = 5 + 6(9 + 11b) = 59 + 66b; substituindo este valor na 3a congruncia, temos: 59 + 66b 3 (mod 17); 66b 56 (mod 17); dividindo por 2, temos: 33b 28 (mod 17); 28 33.11 (mod 17); 33b 33.11 (mod 17); b 11 (mod 17), dando a soluo geral b = 11 + 17c, substituindo este valor em x = 59 + 66b, temos: x = 59 + 66(11 + 17c) = 785 + 1122c que a soluo geral do sistema ou x
785(mod 1122) .

d) x 5 (mod 11), x 14 (mod 29), x 15 (mod 31). Soluo: Como o mdc(11, 29) = mdc(11, 31) = mdc(29, 31) = 1 o sistema possui soluo. A soluo geral da 1a congruncia : x = 5 + 11 a, substituindo este valor na 2a congruncia, obtemos 5 + 11 a 14 (mod 29); 11 a 9 (mod 29); 9 11.14 (mod 29); 11 a 11.14 (mod 29); a 14 (mod 29), dando a soluo geral a = 14 + 29b substituindo este valor em x = 5 + 11 a, temos: x = 5 + 11(14 + 29b); x = 159 + 319b, substituindo este valor na 3a congruncia, temos: 159 + 319b 15 (mod 31); 319b 144 (mod 31); 144 319.15 (mod 31); 319b 319.15 (mod 31); b 15 (mod 31), dando a soluo geral b = 15 + 31c; substituindo este valor em x = 159 + 319b, temos: x = 159 + 319(15 + 31c); x = 4944 + 9889c que a soluo geral do sistema ou
x 4944(mod 9889) .

e) x 7 (mod 9), x 10 (mod 4), x 1 (mod 7). Soluo: Como o mdc(9, 4) = mdc(9, 7) = mdc(4, 7) = 1 o sistema possui soluo. A soluo geral da 1a congruncia : x = 7 + 9a, substituindo este valor na 2a congruncia, obtemos 7 + 9a 10 (mod 4); 9a 3 (mod 4); 3a 1 (mod 4); 1 3.3 (mod 4); 3 a 3.3 (mod 4); a 3 (mod

4), dando a soluo geral a = 3 + 4b; substituindo este valor em x =7 + 9 a, temos x = 7 + 9(3 + 4b) = 34 + 36b; substituindo este valor na 3a congruncia, temos: 34 + 36b 1 (mod 7); 36b 33 (mod 7); 33 36.2 (mod 7); 36b 36.2 (mod 7); b 2 (mod 7), dando a soluo geral b = 2 + 7c, substituindo este valor em x = 34 + 36b, temos x = 34 + 36(2 + 7c); x = 106 + 252c, que a soluo geral do sistema ou x 106(mod 252) .

f) x 28 (mod 29), x 30 (mod 31), x 10 (mod 11). Soluo: Como o mdc( 29, 31) = mdc(29, 11) = mdc(31, 11) = 1 o sistema possui soluo. A soluo geral da 1a congruncia : x = 28 + 29 a, substituindo este valor na 2a congruncia, obtemos: 28 + 29 a 30 (mod 31); 29 a 2 (mod 31); 2 29 (mod 31); 29 a 29 (mod 31); a 1 (mod 31), dando a soluo geral a = 1 + 31b; substituindo este valor em x = 28 + 29 a, temos: x = 28 + 29(1 + 31b); x = 1 + 899b; substituindo este valor na 3a congruncia, temos: 1 + 899b 10 (mod 11); 899b 11 (mod 11); 11 899.11 (mod 11); 899b 899.11 (mod 11); b 11 (mod 11); dando a soluo geral b = 11 + 11c; substituindo este valor em x = 1 + 899b, temos x = 1 + 899(11 + 11c) = 9888 + 9889c, que a soluo geral do sistema ou x g) x a (mod 3), x b (mod 5), x c (mod 8). Soluo: Como o mdc(3, 5) = mdc(3, 8) = mdc(5, 8) = 1 o sistema possui soluo. Vamos resolver este sistema pelo mtodo do resto chins. Temos m1 = 3, m2 = 5 e m3 = 8, sendo m = 3.5.8 = 120. m/m1 = 5.8 = 40, m/m2 = 3.8 = 24, m/m3 = 3.5 = 15. Vamos resolver as seguintes congruncias lineares: 40b1 1 (mod 3); 24b2 1 (mod 5); 15b3 1 (mod 8); 40b1 1 (mod 3), b1 = 1; 24b2 1 (mod 5); b2 = 1; 15b3 1 (mod 8); b3 = 1. Com a1 = a, a2 = b e a3 = c, que a soluo geral do sistema x 40a 24b 15c (mod 120). 03) Resolver o seguinte sistema de congruncia: 1) 5x 11 (mod 17), 3x 19 (mod 32), 11x 6 (mod 37). Soluo: Como o mdc(17, 32) = mdc(17, 37) = mdc(32, 37) = 1 o sistema possui soluo. De 5x 11 (mod 17), x 9 (mod 17) De 3x 19 (mod 32), x 17 (mod 32) De 11x 6 (mod 37), x 14 (mod 37), assim temos o seguinte sistema: x 9 (mod 17), x 17 (mod 32), x 14 (mod 37) Usando o mtodo do resto chins: a1 = 9, a2 = 17, a3 = 14, m1 = 17, m2 = 32, m3 = 37 e m = m1m2m3 = 17.32.37 = 20128. m/m1 = 32.37 = 1184, m/m2 = 17.37 = 629, m/m3 = 17.32 = 544. Ento 1184b1 1 (mod 17), 629b2 1 (mod 32) e 544b3 1 (mod 37). De onde conclumos: b1 = 14, b2 = 29 e b3 = 10. A soluo geral ser: x 12.113 (mod 20.128). 2) 2x 1 (mod 5), 3x 9 (mod 6), 4x 1 (mod 7), 5x 9 (mod 11). Soluo: Como o mdc(5, 6) = mdc(5, 7) = mdc(5, 11) = mdc(6, 7) = mdc(6, 11) = mdc(7, 11) = 1 o sistema possui soluo. De 2x 1 (mod 5), x 3 (mod 5). De 3x 9 (mod 6), x 3 (mod 2).
-1(mod 9889) .

De 4x 1 (mod 7), x 2 (mod 7). De 5x 9 (mod 11), x 4 (mod 11). Vamos resolver pelo mtodo do resto chins: a1 = 3, a2 = 3, a3 = 2, a4 = 4, m1= 5 , m2 = 2, m3 = 7, m4 = 11. m = m1 m2 m3 m4 = 5.2.7.11 = 770.
m = 2.7.11 = 154, 154b1 1 (mod 5), b1 = - 1. m1 m = 5.7.11 = 385, 385b2 1 (mod 2), b2 = 1. m2 m = 5.2.11 = 110, 110b3 1 (mod 7), b3 = 3. m3 m = 5.2.7 = 70, 70b4 1 (mod 11), b4 = 3. m4

A soluo : x 3.(1).154 + 3.1.385 + 2.3.110 + 4.3.70 (mod 770) ou x 653 (mod 770). 4) Resolver os seguintes sistemas de congruncias lineares: 1) x 8 (mod 9), x 2 (mod 3), x 5 (mod 7). Soluo: Como o mdc(9, 3) = 3 e 3 | (8 - 2), mdc(9, 7) = mdc(3, 7) = 1 o sistema possui soluo. A soluo geral da 1a congruncia : x = 8 + 9 a , substituindo este valor na 2a congruncia, obtemos: 8 + 9 a 2 (mod 3); 9 a 6 (mod 3); 3 a - 2 (mod 1); a 1 (mod 7), dando a soluo

geral a = 1 + b; substituindo este valor em x = 8 + 9 a, temos: x = 8 + 9(1 + b) = 17 + 9b; substituindo este valor na 3a congruncia, temos: 17 + 9b 5 (mod 7); 9b 12 (mod 7); 3b 4 (mod 7); 4 3 (mod 7); 3b 3 (mod 7); b 1 (mod 7); dando a soluo geral b = 1 + 7c; substituindo este valor em x = 17 + 9b, temos x = 17 + 9(1 + 7c) = 26 + 63c que a soluo geral do sistema. 2) x 4 (mod 6), x 13 (mod 15), x 8 (mod 14), x 1 (mod 7). Soluo: Como o mdc(6, 15) = 3 e 3 | (13 - 4), mdc(6, 14) = 2 e 2 | (8 4), mdc(6, 7) = mdc(15, 14) = mdc(15, 7) = 1 e mdc(14, 7) = 7 e 7 | (8 1) o sistema possui soluo. A soluo geral da 1a congruncia : x = 4 + 6 a; substituindo este valor na 2a congruncia, obtemos: 4 + 6 a 13 (mod 15); 6 a 9 (mod 15); 2 a 3 (mod 5); 3 8 (mod 5); 2 a 8 (mod 5); a 4 (mod 5); dando a soluo geral a = 4 + 5b; substituindo este valor em x = 4 + 6 a, temos x = 4 + 6(4 + 5b) = 28 + 30b; substituindo este valor na 3a congruncia, temos: 28 + 30b 8 (mod 14); 30b 20 (mod 14); 3b 2 (mod 7); 2 12 (mod 7); 3b 12 (mod 7); b 4 (mod 7); dando a soluo geral b = 4 + 7c; substituindo este valor em x =28 + 30b, temos: x = 28 + 30(4 + 7c).

x = 148 + 210c; substituindo este valor na 4a congruncia, temos: 148 + 210c 1 (mod 7); 210c 147 (mod 7); 30c 21 (mod 1); 21 30 (mod 1); 30c 30 (mod 1); c 1 (mod 1); dando a soluo geral c = 1 + d; substituindo este valor em x =148 + 210c; temos: x = 148 + 210(1 + d) = 358 + 210c que a soluo geral do sistema. 3) x 0 (mod 3), x 1 (mod 4), 17x 9 (mod 23) Soluo: Como o mdc(3, 4) = mdc(3, 23) = mdc(4, 23) = 1 o sistema possui soluo. A congruncia 17x 9 (mod 23) pode ser transformada em x 10 (mod 23) A soluo geral da 1a congruncia : x = 3 a; substituindo este valor na 2a congruncia, obtemos 3 a 1 (mod 4); 1 9 (mod 4); 3 a 9 (mod 4); a 3 (mod 4); dando a soluo geral a = 3 + 4b; substituindo este valor em x =3 a, temos; x = 3(3 + 4b); x = 9 + 12b; substituindo este valor na 3a congruncia, temos: 9 + 12b 10 (mod 23); 12b 1 (mod 23); 1 12.2 (mod 23); 12b 12.2 (mod 23); b 2 (mod 23); dando a soluo geral b = 2 + 23c; substituindo este valor em x = 9 + 12b, temos: x = 9 + 12(2 + 23c) = 33 + 276c que a soluo geral do sistema.

Questes Propostas 01) Resolver os seguintes sistemas de congruncias lineares: 3x 1(mod 7) x 1(mod 10) 01) 5x 2(mod11) R: x 810(mod 1001) 11) x 4(mod 11) 4x 3(mod13) x 6(mod 13)
x 8(mod 9) 2(mod 3) 5(mod 7) x 2 (mod 5) 3 (mod 6) 8 (mod 9)

R: x

851(mod 1430)

02) x x

R: x

26(mod 63)

12) x x

R:

x 1(mod 3)

x 1 (mod 4)

03) x x
x

2(mod 5) 3(mod 7) 2(mod11) 4(mod12) 5(mod13) 3(mod11)

R: x

52(mod 105)

13) x x
x

3 (mod 6) 6 (mod 9) 7 (mod 15)

R:

04) x x
x

R: x

772(mod 1716)

14) x x

4 (mod 32) R: 7 (mod 11)

21x 15(mod 45)


4128(mod 6061)

05) x 5(mod19) R: x x 10(mod 29)


x 5(mod 7) 1(mod 9) 6(mod10)

15) 27x 33(mod 48) 22x 18(mod 46)


x 2(mod 3) 3(mod 5) 2(mod 7)

R:

06) x x

R: x

26(mod 630)

16) x x

x 1(mod 2)

x
R: x
5(mod 42)

8(mod 5) 5(mod 3) 2(mod 4) 2(mod 3) 3(mod 4) 4(mod 5) 5(mod 6)


R: x 119(mod 360) R: x 158(mod 420)

07) x x

2(mod 3) 5(mod 7)

17)

x x x

x 11(mod 7)

2x 1(mod 5)

08) 3x 5x

2(mod 7) 7(mod11)

R: x

283(mod 385)

18)

x x x

7(mod11) 5(mod13) R: x 4(mod 5)


227(mod 715)

x 1(mod 3)
19)

09) 3x 7x

x x x x

3(mod 5) 5(mod 8) 7(mod11) 2(mod 4) 3(mod 5) 4(mod 7) 5(mod 9)

R:

3(mod10)

10) x 11(mod13) R: x 1103(mod 2210) x 15(mod17)

20)

x x x

02) Determinar o menor inteiro positivo mltiplo de 7 que tem para resto 1, quando dividido por 2, 3, 4 e 5. R: 03) Determine os nmeros inteiros cujos restos da diviso por 3, 4 e 5 so, respectivamente, 1, 2 e 3. R: 04) Determine um certo inteiros entre 1 e 1200 tem como restos 1, 2 e 6 quando dividindo respectivamente por 9, 11 e 13. Determin-lo. R: 05) Um Coronel do Corpo de Bombeiros depois de assumir o comando da corporao quis saber qual era o efetivo do Comando Geral para esse objetivo mandou o Ajudante Geral dispor o efetivo sucessivamente em colunas de: 07 indivduos, tendo sobrado 06 indivduos; 11 indivduos, tendo sobrado 05 indivduos; 13 indivduos, tendo sobrado 03 indivduos; Sabendo que o efetivo do Comando Geral, tem menos de 1000 de militares Determine quantos militares constituem o efetivo. R = 874 militares efetivo. 06) Um Coronel depois de destacado para comandar um regimento do Exrcito quis saber qual era o efetivo desse regimento para esse objetivo mandou-os dispor sucessivamente em colunas de: 37 indivduos, tendo sobrado 01 indivduos; 32 indivduos, tendo sobrado 04 indivduos; 27 indivduos, tendo sobrado 01 indivduos; Sabendo que um regimento, tem menos de 10.000 de militares. Determine quantos militares constituem esse regimento. R = 4996 militares efetivo. 07) Um bando de 17 piratas, ao tentar dividir igualmente entre si as moedas de uma arca, verificou que haveria uma sobra de 3 moedas. Seguiu-se uma discusso, na qual um pirata foi morto. Na nova tentativa de diviso j com um pirata a menos, verificou-se que haveria uma sobra de 10 moedas. Nova confuso, e mais um pirata foi morto. Ento, por fim, eles conseguiram dividir igualmente as moedas entre si. Qual o menor nmero de moedas que arca poderia conter? R= 3930 moedas. 08) Dois satlites S1 e S2 em rbita sobre a terra passam periodicamente sobre salvador. Sabendo-se que S1 gasta 32 horas para completar sua rbita e que S2 gasta 23 horas, e que hoje S1 foi visto s 11

horas da manha e S2 foi visto s 10 horas da manha determine quando S1 e S2 sero vistos simultaneamente sobre salvador. R: s 03:00 horas sero vistos simultaneamente. 09) Trs satlites passaro sobre uma cidade esta noite. O primeiro 01 h da madrugada, o segundo s 04 hs e o terceiro s 08 hs da manh. Cada satlite tem um perodo diferente. O primeiro leva 13 hs para completar uma volta ao redor da terra; o segundo 15 h/s e o terceiro 19 h/s. Determine quantas horas decorrero a partir da meia-noite at que os trs satlites passem ao mesmo tempo sobre a cidade? R: 10 horas. 10) Uma senhora estava caminhando para um mercado quando um cavalo se bateu com a sua cesta de ovos. O cavaleiro queria pagar os danos e perguntou para a senhora quantos ovos haviam na cesta. Ela no se lembrava exatamente da quantidade, mas sabia que se tirasse os ovos da cesta de trs em trs, sobravam dois ovos. Se tirasse de 5 em 5, sobravam 3 ovos e de 7 em 7 sobravam 2. Qual seria a menor quantidade de ovos que ela poderia ter? R: Pelo menos 23 ovos. 11) Um casal resolveu ir fazer uma viagem volta do mundo. Sabendo que partiram no dia 01 de maro de um ano bissexto num domingo que chegariam no dia 6 de maro, Segunda-feira e que demoraram menos de 4 anos para fazer a viagem. Determine quantos dias demorou a viagem? R: 12) Num cesto ha mais de 200 e menos de 600 ovos. Sabe-se que, se os ovos forem retirados 2 de cada vez, no fim resta 1 ovo; se forem retirados 3 de cada vez, restam 2; se forem retirados 4 de cada vez, restam 3; se forem retirados 5 de cada vez, restam 4; se forem retirados 6 de cada vez, restam 5; e, se forem retirados 7 de cada vez, o cesto fica vazio. Quantos ovos esto no cesto? R: 13) Trs marinheiros sobreviveram a um naufrgio e foram ter a uma ilha onde vivia um macaco. Comeram alguns cocos e resolveram apanhar mais alguns para comerem nos dias seguintes. Ao anoitecer, imediatamente antes de irem dormir, constataram que tinham apanhado menos de 100 cocos. A meio da noite, um dos marinheiros acordou e, como no confiava nos companheiros, decidiu guardar a parte dele. Assim, tentou dividir o monte dos cocos em trs montes, todos com igual nmero de cocos. Ao fazer isto sobrou um coco que ele deu ao macaco. Enterrou a parte dele e foi dormir. Um segundo marinheiro acordou e, pelo mesmo motivo, decidiu tambm enterrar a parte dele. Dividiu o monte em trs montes iguais, tendo dado ao macaco um coco que, tambm neste caso, sobrou. Enterrou a parte dele e foi dormir. Passado algum tempo o terceiro marinheiro

acordou e fez exatamente aquilo que haviam feito os outros dois anteriormente. Na manha seguinte os trs marinheiros dividiram novamente o monte de cocos em trs partes e voltou a sobrar um coco que deram ao macaco. Quantos cocos havia inicialmente no monte? Com quantos cocos ficou cada um dos marinheiros? R: 14) Um grupo de 17 macacos guarda suas bananas em 11 cestas de igual contedo e em uma 12 cesta contendo 6 bananas. Eles podem dividir o total de suas bananas em 17 grupos iguais. Qual o menor nmero de bananas que eles podem possuir? R: 15) Generais Chineses contavam o nmero de soldados sobreviventes de uma batalha alinhando-os sucessivamente em filas de determinados tamanhos, contando cada vez o nmero de soldados restantes e calculando o total de sobreviventes partir desses dados. Um general tinha inicialmente 1200 soldados antes de uma batalha; aps a batalha, ao alinh-los em filas de 5 soldados, restaram 3 soldados; ao alinh-los em filas de 6 soldados, restaram tambm 3 soldados; ao alinh-los em filas 7 soldados, restou 1 soldado; finalmente, ao alinh-los em filas de 11 soldados, nenhum restou. Quantos soldados sobreviveram batalha? R:

UNIDADE XI TEOREMA DE FERMAT e WILSON 11.1 - Introduo: Desde, pelo menos, 500 anos antes de Cristo, os chineses sabiam que, se p um nmero primo, ento p| 2p-2. Coube a Pierre de Fermat, no sculo XVII, generalizar este resultado, enunciando um pequeno, mas notvel teorema que se constitui no resultado central desta unidade. Nesta unidade estudaremos algumas propriedades referentes a congruncias modulo nmeros primos que so as aplicaes do pequeno teorema de Fermat e o teorema Wilson e sua relevncia no estudo da teoria dos nmeros. 11.2 - Teorema de Fermat: Teorema 11.1 - (de Fermat) - Se p um primo e se p no divide a, ento ap-1 Corolrio 11.1 - Se p primo, ento ap Exemplo 11.2 - Mostrar que 538 1 (mod p).

a (mod p) qualquer que seja o inteiro a.

Exemplo 11.1 - Verificar o teorema de Fermat para a = 3 e p = 7. 4 (mod 11).

Exemplo 11.3 - Mostrar que 117 composto. Teorema 11.2: Se p e q so primos distintos tais que ap ap.q a (mod p.q). 1 (mod 341). a (mod q) e aq a (mod p), ento:

Exemplo 11.4 - Mostrar que 2340 11.2 - Teorema de Wilson:

Teorema 11.3 - (de Wilson)- Se p primo, ento (p 1)! Teorema 11.4 - Se (n 1)! 1 (mod n), ento n primo.

1 (mod p)

Obs: O teorema de Wilson e o seu recproco do um critrio para se reconhecer se um inteiro dado primo um inteiro n > 1 primo se e somente se, (n 1)! 1 (mod n). Entretanto, para inteiros

grandes este critrio impraticvel e por isso apenas tem utilizao terica. Exemplo 11.5 - Verificar o teorema de Wilson com p = 7. Exemplo 11.6 - Reconhecer se o inteiro 11 primo. Questes Resolvidas 01) Verificar o teorema de Fermat com a = 2 e p = 13. Soluo: Temos que mostrar que 212 24 3 (mod 13); 28 1 (mod 13). 3.9 1 (mod 13) 9 (mod 13); 212 = 24.28

02) Verificar o teorema de Wilson para p = 5. Soluo: Temos que mostrar que (5 1)! 2.3 1 (mod 5); 2.3.4 4 (mod 5); 4 1 (mod 5) 1 (mod 5) 1 (mod 5); logo 4!

03) Mostrar que 8 composto usando o teorema de Wilson: Soluo: (8 1)! = 2.3.4.5.6.7 conclumos que 8 no primo. 04) Mostrar que 19 primo usando o recproco do teorema de Wilson. Soluo: (19 1)! = 2.3.4...17.18. Vamos ver a que nmero congruente este fatorial. 2.3.4 = 5 (mod 19); 5.6 11 ((mod 19); 7.8 1 (mod 19); 9.10 5 (mod 19) 11.12 1 (mod 19); 13.14 0 (mod 8); portanto, como (8 1)! no congruente a 1

11 (mod 19); 15.16

12 (mod 19); 7.18 2 (mod 19). 2.(1) 2 (mod 19); 10! = 8!.9.10 9.11 4 (mod 1 (mod 19). 9 (mod 19); 14! = 12!.13.14 9.2 18

Assim 6! = 2.3.4.5.6 5.11 17 2 (mod 19); 8! = 6!.7.8 2.(5) 9 (mod 19); 12! = 10!.11.12 4.12 9.(1)

19). 16! = 14!.15.16

9 (mod 19); 18! = 16!.17.18

05) Reconhecer se 17 primo usando o teorema de Wilson: Soluo: Temos que verificar se (171)! 4! 7.8 7 (mod 17); 5.6 4.5 1 (mod 17). Temos: 7.13 6 (mod 17)

13 (mod (17); 6! = 4!.5.6 6.5 3 (mod 17); 11.12

5 (mod 17); 8! = 6!.7.8

4 (mod 17); 9.10 = 5 (mod 17) 13 (mod 17) 12 (mod 17) 2 (mod 17)

10! = 8!.9.10 12! = 10!.11.12 14! = 12!.13.14 16! = 14!.15.16 06) Verificar: a) 186

3.13 5 (mod 17); 13.14 5.12 8.2 8 (mod 17); 15.16 16 1 (mod 17)

1 (mod 49)

Soluo: 182 30 (mod 49); 184 30.30 18 (mod 49).

186= 182.184

30.18 1 (mod 49) Soluo: 183 = 5832 1 (mod 343); 186 1 (mod 343).

b) 186 1 (mod 343)

7) Achar o resto da diviso de 15! por 17. Soluo: 16! 16! 1 (mod 17) pelo teorema de Wilson; 1 16 (mod 17) ento,

16 (mod 17); da 15!.16 16 (mod 17). Podemos cancelar o fator 16, desde que mdc(16, 17) = 1 (mod 17). Conclumos que o resto procurado 1. 1 (mod 35).

1 e portanto 15!

8) Mostrar que, se o mdc(a, 35) = 1, ento a12 a4 1 (mod 5); a12 1 (mod 5) logo 5 | (a121)

Soluo: 35 = 5.7 e portanto mdc(a, 5) = mdc(a, 7) = 1. Aplicando o teorema de Fermat: a6 1 (mod 7); a12 1 (mod 7) logo 7 | (a12 1). Como mdc(5, 7) = 1 ento, 35 = 5.7 | (a12 1) o que demonstra a12 1 (mod 35) 9) Demonstrar que, para todo inteiro a, se tem:

a ) a13 a (mod 7). Soluo: Suponhamos que 7 | a. Ento a 0 (mod 7); a13 a6 1 (mod 7); a7 a (mod 7); a13 = a6.a7 a (mod 7) b) a37 a (mod 13). Soluo: Suponhamos que 13 | a. Ento a 0 (mod 13); a37 0 (mod 13) e portanto a37 a (mod 13). Agora suponhamos que 13 no divide a. Ento, pelo teorema de Fermat a12 1 (mod 13); a13 a (mod 13); a24 = (a12)2 a37 = a13.a24 a (mod 13). Questes Propostas 01) Verificar o teorema de Fermat com a = 2 e p = 17. 02) Demonstrar que, para todo inteiro a, se tem: a) a21 a (mod 15) b) a7 a (mod 42) 1 (mod 13) 0 (mod 7) e portanto a13 a (mod 7). Agora, suponhamos que 7 no divide a ento, pelo teorema de Fermat:

03) Demonstrar que, para todo inteiro positivo n, se tem: a) 22n 1 (mod 3) 04) Mostrar que 18! + 1 05) Mostrar que 538 06) Mostrar: a) 561 | (2561 2) 07) Verificar o teorema de Wilson p = 5. 08) Mostrar que 8 composto usando o teorema de Wilson. 09) Verificar que 186 1 (mod 49). b) 561 | (3561 3). 0 (mod 437). b) 23n 1 (mod 7)

4 (mod 11).

10) Achar o resto da diviso de 15! Por 17. 11) Usando o pequeno Teorema de Fermat, encontrar o resto da diviso de 2100.000 por 17. 12) Encontrar o digito das unidades de 3100, quando expresso na base 7. R: r = 4. 13) Mostrar que se p e q so primos distintos, ento pq - 1 q p - 1 1(mod p q) . 14) Mostre que 13 | 270 + 370, usando o teorema de Fermat. 15) Usando o teorema de Wilson encontrar o menor resduo positivo de: 6 7 8 9(mod 5) . 16) Formar com os inteiros 2, 3, 4, ... , 21 todos os pares a e b tais que ab 1 (mod 23).

O LTIMO TEOREMA DE FERMAT Desde da antiguidade o homem sentem uma fascinao por nmeros que surge quase imediatamente da prtica. Um dos problemas mais antigos o de dividir um quadrado em soma de dois quadrados e uma das solues {3, 4, 5}, i.e., escrever 25 = 16 + 9. Aparentemente os antigos egpcios j usavam estes nmeros para construir ngulos retos e reconheciam que {3, 4, 5} eram lados de um tringulo retngulo. Por volta de 500 AC os chineses tambm sabiam deste tringulo. Provavelmente tambm os babilnios sabiam o que est por trs disto o teorema de Pitgoras Para que um tringulo seja retngulo necessrio e suficiente que o quadrado de sua hipotenusa seja igual a soma dos quadrados dos catetos. Olhando de outra maneira a equao a2 = b2 + c2, admita como soluo uma terna {c, b, a} de valores inteiros, as quais chama-se ternas pitagricas. A tabela babilnica Plimton 322 apresentam uma lista de ternas pitagricas. Por volta de 250 AC o matemtico grego Diofanto escreveu o primeiro livro de titulo Aritmtica, dedicado ao que hoje chamamos de Teoria de Nmeros. Ele estuda solues inteiras de equaes que hoje chamamos de equaes diofantinas. Para construir-se ternas pitagricas basta observarmos a identidade algbrica (a+b)2 = 4ab, ajeitando a = u2 e b = v2, obtendo-se a terna {u2 - v2, 2uv, u2 + v2}. PIERE DE FERMAT Pierre de Fermat (1601-1665) foi um Juiz francs que nasceu e viveu em Toulouse, Frana. Ele possua uma cultura universal na poca, que cultivava a poesia, filosofia grega, direito e principalmente matemtica. Fermat seguia uma certa tradio da poca propor problemas tipo desafio, para outros matemticos. Ele considerado juntamente com Descartes um dos criadores da Geometria Analtica. Suas idias sobre mtodos das tangentes contm as razes do Clculo Diferencial. Raramente encontrasse uma publicao de Fermat como tambm ele raramente se encontrou pessoalmente com outros grandes matemticos de sua poca. Suas comunicaes estavam em suas correspondncias que geralmente era enviada pelo padre Marin Mersenne, e seu amigo Pierre de Carcavy, que circulavam seus manuscritos, cartas em geral. Estes ltimos foram colecionados em forma de livro por seu filho Samuel. Uma grande parte de seu trabalho e problemas foram resolvido nos 200 anos seguintes sua morte. Outra grande parte dedicada as equaes diofantinas onde ele deixou uma srie de problemas. Dentre os vrios problemas temos a representao de primos como soma de quadrados, gerao de primos, divisibilidade por primos, e soluo de x2 - dy2 = +1. a maioria deles foi resolvida no sculo seguinte, por Euler, Legendre, Dirichlet e outros. O problema conhecido como ltimo Teorema de Fermat, ou FLT, i.e., o ltimo problema de Fermat em aberto, uma generalizao do problema das ternas pitagricas. Ele foi proposto numa (ab)2

margem de uma edio do livro de Diofanto ao lado do problema 8, livro II, que o problema da decomposio de um quadrado em soma de dois outros. Ele afirma que isto impossvel para cubos e potncias superiores e afirma que possui uma demonstrao elegante para este fato e que no cabe na margem. Ele tambm propem este problema para outros matemticos da poca Frenicle, Wallis, ..... O problema mostrar que a equao F[n], an + bn = cn, n inteiro maior que 2, no possui solues no triviais no sentido de que pelo menos um dos nmeros a, b ou c zero. fcil de verificar que basta mostrar que F[n] no tem soluo para n = 4, e para primos diferentes de 2. O caso n = 4 foi resolvido por Fermat onde ele exibe seu mtodo a qual denomina mtodo da descida infinita. O problema se enuncia ento mostrar que para todo primo impar p, F[p] no possui solues no triviais. O problema pode ser dividido em dois casos: O primeiro onde se requer que nenhum dos nmeros a, b, e c sejam divisveis por p, e o segundo o caso onde precisamente um dos nmeros divisvel por p. Para n = 3, o cubo de 5 mais o de seis difere muito pouco do de sete. Este problema foi proposto em 1637, e da seguiu-se uma busca para reproduzir a demonstrao de Fermat. Como Fermat errou na soluo de outros problemas, admite-se a possibilidade dele tambm ter errado em sua afirmao, porem no sabemos qual era sua demonstrao. PRIMEIRAS TENTATIVAS As primeiras tentativas foram feitas para p = 3. Euler, no inicio do sculo passado, apresentou a soluo neste caso. Ele cometeu enganos porem depois de um estudo minucioso da divisibilidade de nmeros da forma a2+3.b2 ele obteve uma soluo correta. Gauss apresenta tambm uma demonstrao para este caso. Com Gauss, 1801 nasce a Teoria dos Nmeros. O caso p = 5, foi demonstrado por G.L.Dirichlet em 1928. O caso p = 7 de Lam (1839). Sophie Germain, em 1823, foi a primeira a tentar uma soluo geral ela mostra que se 2p+1 tambm primo ento o primeiro caso valido. Isto introduz um novo problema existe um numero infinito de primos para os quais 2p + 1 tambm primo? At hoje no se sabe. Com este resultado Legendre consegue mostrar a validade do primeiro caso para todo primo menor que 100. Lam e Cauchy apresentam demonstraes erradas. KUMMER At o fim de 1993 o nico teorema geral que se conseguiu devido ao matemtico alemo E. Kummer. Ele introduz a noo de primo regular, estes primos esto relacionados a certas condies de divisibilidade de uns nmeros chamados nmeros de Bernouilli, Kummer demonstra por volta de 1850, que F[p] vlido para todo primo regular. Infelizmente no se sabe se existem ou no infinitos primos regulares, sabe-se que existe um nmero infinito de primos no regulares. Os primos no regulares menores que 164 so 37, 59, 67, 101, 103, 131, 149, e 157. A surge com

Kummer, Dirichlet e Dedekind a Teoria de Nmeros Algbricos, teoria esta que falando grosseiramente trata de expresses contendo radicais. A noo de nmero primo e de decomposio estendida a estes nmeros. Isto sempre foi essencial em todos os casos demonstrados. Em 1816, e de novo em 1850 a Academia Francesa de Cincias ofereceu uma medalha de ouro e um prmio de 3000 Francos a quem resolve-se o problema de Fermat. A medalha foi concedida a Kummer pelo seu brilhante trabalho neste teorema. No inicio deste Sculo, um matemtico alemo Wohlfskehl a beira do suicdio descobriu o trabalho de Kummer sobre o teorema de Fermat e decidiu dedicar-se a sua leitura. Ele deixou sua fortuna na Knigliche Gesellschaft der Wissenschaften, em Gttingen, Alemanha, oferecendo um prmio de 100.000DM pelo soluo deste problema, devido a inflao este prmio est reduzido a 10.000DM. Este prmio leva a Academia a ter que selecionar milhares de trabalhos apresentados anualmente, dentre os quais a grande maioria ou no fazem sentido, ou so muito elementares. Deve-se observar que Fermat era um matemtico muito bom ele resolveu o caso n = 4 aplicando seu mtodo denominado a descente infinita, e que qualquer soluo que se Fermat tinha em mente deveria ter contedo matemtico. FERMAT COMPUTACIONAL Depois de Kummer um grande nmeros de matemticos como Wagstaff, Morishima, Inkeri, Vandiver, Eichler, Brckner, tentaram desenvolver critrios para que pudssemos verificar a validade de F[p] via computador. Estes critrios so complicados para mencionarmos aqui. Um deles porem bem simples Wiefrich (1909). Se p2 no divide 2p-1-1 ento o primeiro caso do teorema de Fermat verdadeiro. At hoje s conhecemos dois nmeros dentre todos os primos menores 3x109 para os quais isto no verdade 1093 e 3511. A temos um novo problema existe um nmero infinito destas excees? Desenvolveu-se tambm critrios para o estudo do caso II para primos no regulares, e com isto Wagstaff em 1976 mostra FLT para todo p < 125000. Em 1992 Buhlen verificou a validade de FLT para todos os primos menores que 4000000. Outra direo tomada foi de estudar o tamanho da soluo, i.e.., se x o menor valor de uma soluo de F[p] qual um estimativa de x, ou melhor qual a estimativa do nmero que deve-se tentar para exibir um contra exemplo? Usando-se o resultado de Wagstaff a concluso que deveramos tentar nmeros com mais de 18x10 11 dgitos. Isto mostra que contra-exemplos est totalmente fora de nosso alcance. TEORIA DE CURVAS Na dcada dos 20, Mordell olha para o problema do ponto de vista de geometria algbrico F[p] a equao de uma curva algbrica, e dizer que existe uma soluo no trivial significa dizer que ela tem uma soluo ou ponto com coordenadas racionais, ou simplesmente uma soluo racional. O primeiro passo seria mostrar que estas curvas teriam somente um nmero finito de

pontos racionais, desde que um dos seus invariantes, seu gnero seja maior que dois. Em 1983 o matemtico alemo G. Faltings demonstrou esta conjectura, o que lhe valeu a Field Medal prmio quadrienal ortografo nos Congressos Internacionais aos melhores trabalhos do perodo, equivalente ao prmio Nobel para matemtica. Seguindo a idia de Faltings em trazer toda maquinaria moderna da geometria algbrica para a teoria de nmeros, um grupo de matemticos associa ao problema de Fermat uma curva algbrica especial que possui muita estrutura aritmtica a curva eltica. As curvas de primeiro grau, equao linear em duas variveis representa uma reta no plano as do segundo grau representam cnicas (elipse, hiprbole, parbola ou circunferncia). Uma equao do terceiro grau representa uma cbica, ela pode ser transformada ou em y = x(x+a)(x+b) ou em y2 = x(x-a)(x+b). Esta ltima chama-se curva eltica, ser indicada por E[a, b]. Elas tem este nome por estarem associadas as integrais elticas, integrais estas que so duplamente peridicas. Elas tambm possuem uma operao natural de adio de pontos. A curva no singular se a e b so distintos e diferentes de zero. Estudar a aritmtica da curva significa estudar os pares de inteiros que satisfazem sua equao. Seja E uma curva eltica. Para cada primo p vamos chamar de n(p) o nmero de solues de sua equao modulo p. Quando os nmeros n(p) possuem uma frmula aritmtica de clculo a curva chama-se modular. Em 1955 um matemtico japons chamado Tanyama postulou que toda curva eltica onde a e b so nmeros racionais uma curva modular. Um matemtico alemo G. Frey em 1980 teve a idia de associar a cada soluo {a, b, c} de F[p] a curva eltica E[an,bn] a qual tem uma estrutura algbrica muito estranha, e da a suspeita de que tal curva no poderia existir e isto implicaria na no existncia da soluo de F[p]. Em 1986 K. Ribet mostra que a curva associada a F[p] no modular, e da a conjectura de Tanyiama implica em FLT !!!. No fundo o que Ribet fez foi mostrar que uma conjectura mais fraca, a de J P Serre, um dos grandes matemticos deste sculo, se aplica a curva de Frey. ANDREW WILES. Andrew Wiles nasceu em 1954 na Inglaterra. Ele conta que quando tinha dez anos pegou na biblioteca um livro onde viu este problema e desde ento se props a resolv-lo. Deixou depois de varias tentativas amadora, o problema de lado foi estudar matemtica em Cambridge com um grande especialista em teoria de nmeros, John Coates. Veio para Harvard onde passou dois anos e resolveu dois problemas em aberto nesta rea. Em seguida aceita uma posio de Professor em Princeton. Quando soube dos resultados de Ribet, ele ganhou confiana de que poderia resolve FLT e ao contrario do matemtico comum ele decide se isolar por sete anos ate apresentar uma soluo ao problema. Levou cinco anos para ter uma idia da soluo.

A primeira vista pode-se pensar que Wiles se trancou e ignorou o que se passa na sua rea; isto no foi o caso, pois uma boa tcnicas e construes no existiam em 86. Sua demonstrao apareceu volume de Maio do Annals of Mathematics de 1995. A soluo apresentada em 23 de Junho de 1993 tem um erro que depois foi consertado por um aluno dele. O trabalho original acertado juntamente com o material necessrio para corrigir o anterior ocupa todo este volume do Annals. O que Wiles mostra que toda curva eltica semiestavel modular e em particular a curva de eltica associada a uma possvel soluo de F[p] tambm modular o que contradiz ao teorema de Ribet. Logo a cura no existe, i.e., F[p] no admite solues no triviais. Observaes Obs1: Babilnios e certamente Diofanto j sabiam que existe um numero infinito (em potencial) de ternas Pitagricas. Obs2: Atualmente o material lecionado no tanto no secundrio como nos cursos de matemtica uma preparao para se entender mais tarde o que realmente a matemtica. A grande aventura lgica. Idias que vem da natureza. Satisfao pessoal de resolver problemas e mais ainda de criar nova matemtica. Neste sentido a Matemtica e nica, a cada minuto de envolvimento nos d grande prazer, mesmo para o profissional coisas simples como novas maneiras de apresent-la so gratificantes. Obs3: O processo de verificao e publicao de resultados longo. Por um lado a Matemtico requer lpis e papel somente (hoje em dias supercomputadores em alguns ramos como, por exemplo, teoria de nmeros) o que a torna acessvel a milhares de pessoas, os matemticos hoje em dia so da ordem de 200.000. Cada anos se produz cerca de 1.000.000 de resultados novos. Cada um ou dois anos ela dobra o que torna impossvel a matemticos (the mathematical experience) saberem toda a matemtica. H. Poincare foi o ultimo universal. nos 60 primeiros anos do sculo 20 A Weil era universal nas reas de Geometria Algbrica, lgebra, Teoria de Nmeros, e outras reas relacionadas ele tinha a reputao de ter um grande faro matemtico, uma grande intuio dos problemas que so solveis. Enunciar problemas muito fcil porem a grande maioria so de soluo muito difcil ou ate impossvel com tcnicas atuais. Muitas vezes eles requerem um amadurecimento de um ou vrios matemticos por dcadas. Atualmente j difcil dominar um das centena de seus ramos e sub-ramos. O Mathematical Review a principal revista que apresenta resenhas dos trabalhos publicados no ultimo ano, ele atualmente um volume maior que um dos volumes de qualquer enciclopdia. Apesar de alguns problemas como o de Fermat serem simples de enunci-los a probabilidade de se resolver novos problemas com tcnicas elementares bem pequena. No se faz pesquisa a nvel de graduao, nem mesmo a nvel de mestrado, pois os problemas em aberto requerem cada vez mais quantidades muito grande de conhecimento. Aquele

fato de que grande matemtica Os feita por jovens j esta passando, pois solues de grandes problemas requerem muito conhecimento de tcnicas e de certo domnio de outros ramos, o que leva tempo de aprendizado. Afinal de contas, Wiles tem quarenta anos. Obs4: Ningum nunca saber qual a demonstrao que Fermat tinha em mente. Muitos matemticos acham que ele errou em algum canto. Outros porem acham que ele pode ter descoberto uma demonstrao deste teorema. Ele tinha uma autoreputao para cuidar. CONFERENCIAS SOBRE FLT. Em linhas gerais seguimos a conferencia patrocinada pelo MSRI (Mathematical Sciences Research Institute- Berkeley, California) onde que consistiu de cinco conferencias de dez minutos, seguida de uma mesa redonda. Em ordem falaram Osserman e ,Buhlen diretores do MRSI e os eminente matemticos C. Rubin , Ribet e J.Conway. Osserman fala sobre a pr-histria do problema e acrescenta dois pontos no apresentados acima. O primeiro um problema pratico Voc entra numa Pizzaria e voc tem duas escolhas: Comer uma pizza grande, ou uma pequena e uma media. Queremos saber qual escolha tem mais pizza. Se cortamos as pizzas ao meio e com seus dimetros formarmos um tringulo, se este for obtusngulo a primeira escolha a melhor. Se o tringulo for acutngulo a segunda escolha a melhor. Os existe uma posio onde as duas escolhas so iguais quando o tringulo retngulo, e isto uma conseqncia imediata do teorema de Pitgoras e sua recproca a lei dos cossenos. O segundo ponto refere-se a Pitgoras. Na realidade ele era o lder de uma escola filosfica de iniciao que se dedicava ao estudo da perfeio do universo. Eles descobriram que a natureza traz um grande numero de fenmenos que podem ser descritos via matemtica. A seguir ele descreve a teoria musical de Pitgoras: a busca por notas harmnicas. O fato o seguinte: se tivermos uma corda vibrante de extremos AB e a subdividimos prendendo-a em um ponto C, e tocarmos as duas sub-cordas sucessivamente observaremos que razes como 3:4 ou 2:3 soam harmonicamente. Se considerarmos cordas sobre os lados de um tringulo retngulo teremos tringulos harmnicos onde qualquer pares de lados so harmnicos. Os tringulos de lados inteiros so harmnicos!. Dai o interesse por ternas Pitagricas. Convm mencionar que no caso do tringulo retngulo issceles a razo 1:2. Esta razo foi conhecida na idade media como musica diablica. O problema das construes das ternas Pitagricas esta resolvido no livro de Diofanto. Buhlen discute o desenvolvimento do problema, os resultados obtidos nos 349 anos seguinte. Alem do que foi dito acima ela fala um pouco mais sobre a vida de Sophie Germain, antes deste sculo uma das trs matemticas. Ela estudou matemtica as escondidas, seu trabalho foi apresentado com outro nome M. Leblach pois a Academia Francesa no admitia mulheres, e por fim teve ser trabalho

reconhecido por Gauss. O livro de Gauss sobre aritmtica, publicado em 1801 quando ele tinha 21 anos d a teoria de nmeros um status de rea da matemtica. Antes ela era tratada como recreao. Gauss tambm no da grande importncia ao problema de Fermat. A primeira pergunta que surge quando nos enfrentamos com um problema matemtico saber como verificar a validade de nossas afirmaes, ou tentamos tambm mostrar que nossa afirmao no valida, e para isto basta exibir um exemplo onde o teorema no valido. Tendo isto em mente durante pelo menos um sculo ou mais tentou-se encontrar contra exemplos procedimento este que foi abandonado nos meados deste sculo quando se teve conscincia que o valor da menor soluo muito grande (Milnor). O caso n = 4 um dos poucos onde Fermat apresenta sua demonstrao na margem de uma pagina do livro de Diofanto usando a descente infinita que consiste em assumir a existncia de uma soluo, achar o menor valor da terna soluo e por algum processo fabricar uma outra soluo que tenha menor valor que o da primeira terna. Qualquer demonstrao de FLT feita pelo mtodo de reduo ao absurdo: parte-se de uma suposta soluo e obtm-se uma contradio. Outro mtodo o usado por Diofanto a analise dos divisores primos de xn = yn - zn. J que este ultimo lado direito pode ser decomposto em fatores que em geral so nmeros complexos. Os erros de demonstrao foram na analise de divisibilidade destes fatores. Uma analise mais critica destes fatores levou Kummer a criao da noo de (numero) ideal, e dai a criao de uma teoria destes objetos anloga a de nmeros. Rubin comea tratando de uma curva eltica: o grfico da equao y2 = x3 - x. Ela foi tratada por Fermat que mostrou no ter soluo inteira. Esta equao aparece na determinao de todos os tringulos retngulos de lados inteiros cuja rea um quadrado perfeito. Seu objetivo a noo de modularidade. Ele explicita um procedimento muito comum na teoria de nmeros mais especificamente quando se quer saber se uma equao tem ou no solues inteiras o que em geral um problema bastante difcil. Reduz-se a equao modulo um primo dado observando-se que se ela tem soluo inteira ela ter soluo modulo p para todo primo p, o que sempre fcil de se verificar pois ficaremos com um problema de contagem. O prximo passo verificar se ela tem soluo modulo pn para todo n. E dai tenta-se levantar esta soluo para uma soluo inteira este o chamado principio de Hasse. No caso das curvas elticas este procedimento apresenta enormes dificuldades. No caso particular da equao acima Gauss em 1814 apresenta uma formula para este calculo. Ribet baseado na formula de Gauss apresenta a seguinte tabela: primos: 2 3 5 7 11 13 17 19 23 29 31 n(p) 2 3 7 7 11 7 15 19 23 29 31 primos 1000003 1000033 1000037 1000039 n(p) 1000003 998207 998055 1000039

O calculo de n(p) para nmeros pequenos pode ser feito atravs de uma tabela contando-se o numero de pares onde p divide y2 - x3 + x. para p = 5 a tabela contem essencialmente 25 valores. J para 1000003 a tabela contem mais de 1012 valores o que difcil de contar. Rubin botem os valores acima da formula de Gauss. Nos casos simples n(2)=2, e se p for um primo da forma 4k-1, n(p) = p. No outro caso a formula mais complicada. Uma curva eltica em geral chama-se modular se esta seqncia de inteiros n(p) admite uma formula de calculo. A curva modular desempenha um papel fundamental na soluo de Wiles. Em 1955 Yutata Tanyama apresenta a hiptese que toda curva eltica a coeficientes inteiros modular na poca e percebeu motivos para esta hiptese. Tanyiama morreu em 58 mais tarde Goro Shimura professor em Princeton apresenta boas razes porque a hiptese deveria ser verdadeira. Dai em diante o numero de especialista s na rea que acreditavam na hiptese foi crescendo. Esta a pea principal da soluo de Wiles ele demonstra a hiptese de Tanyiama para uma classe especial de curvas que so as semi-estaveis. Ribet comea observando que um matemtico alemo G. Frey no inicio da dcada dos 80 comeou a fazer clculos, e associou curva de Fermat uma curva eltica E[an, bn]. Em 1981 Frey veio a Berkeley e trocou idias com Ribet. Na poca ele j estava percebendo a ligao de FLT e Tanyiama, porem no estava claro. Em Janeiro de 1985, em Oberwolfach ele apresentou uma curta comunicao onde delineava suas ideais, esta comunicao chegou a Paris onde Serre, um dos grandes matemticos atuais que tambm possui a medalha da rea, observou que uma pequena variao da conjectura de Tanyiama implicaria FLT. Em 1986 ele, Ribet tinha uma demonstrao da conjectura de Serre e apresenta a Mazur que diz que parece correta. Ribet passa todo o ano de 87 no MSRI onde apresenta seminrios semanais sobre sua demonstrao. Como o processo de publicao lento, passa por referres onde tem-se que negociar sugestes, e somando-se o atraso nas publicaes, seu trabalho foi publicado em 1990. A seguir ele comenta como Wiles se retirou e observa que a soluo depende na realidade de muitas outras estruturas e trabalhos. Ribet observa que a soluo de Wiles no um fato isolado. Ela depende dos trabalhos que muitos outros desenvolveram nesta ultima dcada este trabalho envolve tcnica desenvolvidas nas reas: 1. Famlias de Representaes de Galois (Hida, Mazur) 2.Teoria de Iwahori (Greenberg, Rubin) 3.Sistemas de Euler(Kolyvagen, Flach) 4.Formas Modulares(Ribet) 5.Geometria Algbrica(Faltings) 6.Teoria da Representao(Langlands, Tunnel)

TRABALHO E PROBLEMAS DE FERMAT NA TEORIA DE NMEROS Vamos a seguir apresentar uma pequena lista com comentrios sobre os problemas enunciados por Fermat. 1. O primeiro de nossa lista o pequeno teorema de Fermat Seja p um primo dado. Ento para todo inteiro a p divide ap-a. Este foi generalizado por Euler e sua verso geral que para todo n inteiro positivo e todo inteiro a n divide af(n)-a, onde f(n) o numero de inteiros entre 0 e n-1 que so relativamente primos a n. 2. Fermat estudou formulas para primos a primeira quando M(n) = 2n -1 primo. Isto leva ao problema de saber para que valores de n, primo, M(n) primo. Estes primos chamam-se primos de Mersenne pois foi ele que testou todos os nmeros primos menores que 100, com alguns erros. No se sabe se existe um numero infinito de primos de Mersenne. Outro caso estudado por Fermat foi a primalidade de F(n) = 2n -1. Aqui necessrio que n seja uma potncia de dois. Fermat afirmou que todos estes nmeros so primos. Euler mostrou que F(5) divisvel por 641. Ate hoje no se conseguiu outro valor maior que 5 que nos fornea um numero primo, tentou-se at 11, e alguns valores acima tambm. 3. Fermat estudou a representao de nmeros como soma de outros particulares. Por exemplo um primo cujo resto na diviso por 4 um uma soma de dois quadrados. Isto foi demonstrado por Euler em 1749. Em 1977, Lagrange mostra que todo nmeros soma de quatro quadrados. Em 1801 Gauss demonstra o teorema de Fermat sobre Os de trs nmeros triangulares. Em 1813, Cauchy demonstra os teoremas sobre cinco nmeros pentagonais e outros. Em 1840 Jacobi demonstra outros teoremas deixados por Fermat. 4. A equao de Pell x2 - dy2 = +1, d inteiro, assim chamada erroneamente por Euler, tambm foi estudada por Fermat. Fermat j sabia que esta equao possuiu um numero infinito de solues quando d > 0. Legendre mostra, usando fraes continuas que todas solues so potncias de uma delas mais tarde Dirichlet demonstra seu famoso teorema de unidades o qual engloba todos estes resultados.

BIBLIOGRAFIA: 1) Edgard de Alencar Filho - Teoria Elementar dos Nmeros - Livraria Nobel S.A. - 1981. 2) Hygino H. Domingues - Fundamentos de Aritmtica - Atual - 1991. 3) L.H. Jacy Monteiro - lgebra Moderna - Volume I - LPM - 1963. 4) I. Vinogradov - Fundamentos de la Teoria de los Nmeros - Editorial Mir. - 1977. 5) Jos Plnio de Oliveira Santos Introduo Teoria dos Nmeros Coleo Matemtica A. J. 6) Pettofrezzo - Teoria de los Nmeros - Editorial Prentice / Hall Intercontinental - 1972. 7) Universitria - 1998 George E. Andrews Number Theory Dover Publications, Inc. - 1994.

Você também pode gostar